RELATION Anf Function Class 12 (1) - 230613 - 235541

You might also like

Download as pdf or txt
Download as pdf or txt
You are on page 1of 287

NOTE

M A T H E M A T I C S

RELATIONS AND FUNCTIONS


INTRODUCTION TO RELATIONS

What you already know What you will learn

• Set theory • Cartesian product of sets • Types of relations


• Relations
• Domain and range of a relation

Cartesian Product of Sets

Let A and B be two non-empty sets, then the cartesian product denoted by A × B is the ordered
pair of elements. A × B = {(a, b): a ∈ A and b ∈ B}
A B
Examples
a
Find A × B for the following sets: 1
b
A = {a, b, c} 2
B = {1, 2} c

Solution
A × B = {(a, 1), (a, 2), (b, 1), (b, 2), (c, 1), (c, 2)}

Number of elements in the Cartesian product of sets

If n(A) = p and n(B) = q, then we get,


Cardinality of A × B = n(A × B) = n(A) × n(B) = pq
For A={a, b, c} and B = {1, 2}
Cardinality of A × B = n(A × B) = 3 × 2 = 6

Relations

Let A and B be two sets. Then, relation R from A to B is a subset of A × B, i.e., R ⊆ A × B


Number of relations from A to B = Number of subsets of A × B
If n(A) = p, n(B) = q
Number of relations from A to B = 2pq
Examples
n(A) = 6, n(B) = 4
⇒ n(A × B) = 6 × 4 = 24
Number of relations from A to B = 2n(A × B) = 224

© 2020, BYJU'S. All rights reserved


02

Examples
If A = {1, 2} and B = {1, 2} then find the number of relations from A to B.

Solution

Given A = {1, 2} and B = {1, 2}


Therefore, A × B = {(1, 1), (1, 2), (2, 1), (2, 2)}
We know that relation R from A to B is a subset of A × B.
R ⊆ A × B = {(1, 1), (1, 2), (2, 1), (2, 2)}
Now, number of ways to select a null set from A × B = 4C0
Number of ways to select one ordered pair from A × B = 4C1
Similarly, the number of ways to select two, three, and four ordered pairs simultaneously from
A × B is given by 4C2, 4C3, and 4C4, respectively.
∴ Total number of ways to select relations from A × B= 4C0 + 4C1 + 4C2 + 4C3 + 4C4 = 24

Note

• If a set has n elements, then the total subsets are 2n.


• The total number of subsets is 2n, and the number of non-void subsets is 2n - 1.

If x ∈ {1, 2, 3, 4} and y ∈ {1, 2, 3, 4}, then find the ordered pair in the following conditions:
(a) R = {(x, y)| x = y} (b) R = {(x, y)| y = x + 1}

Solution

(a) R = {(x, y)| x = y} = {(1, 1), (2, 2), (3, 3), (4, 4)}
(b) R = {(x, y)| y = x + 1} = {(1, 2), (2, 3), (3, 4)}

Domain and range of a relation


The domain of relation R is the set of the first element of all the ordered pairs of relation R.
The range of relation R is the set of the second element of all the ordered pairs of relation R.

© 2020, BYJU'S. All rights reserved


03

If A = {-4, 1, 2, 4, 9}, B = {-2, 0, 1, 2, 3, 5}, and R : A → B such that R = {(a, b): a = b2, a ∈ A,
b ∈ B}, then write relation R and also its domain and range.

Solution

Given, R = {(a, b): a = b2, a ∈ A, b ∈ B}


a = 1 ⇒ b2 = 12 = 1; ( a, b ) ≡ (1, 1)
a = 4 ⇒ b2 = -22 = 4; ( a, b) ≡ ( 4, -2)
a = 4 ⇒ b2 = 22 = 4; ( a, b) ≡ ( 4, 2)
a = 9 ⇒ b2 = 32 = 9; ( a, b) ≡ ( 9, 3)
Therefore,
R = {(1, 1), (4, -2), (4, 2), (9, 3)}
Domain is {1, 4, 9}
Range is {1, -2, 2, 3}

Types of Relations

Void relation

A relation R on set A is known as a void or empty relation if no element of set A is related to any
element of set A.
⇒R=φ

Example

A= {Students in a boys school}


Let R = {(a, b) : b is a sister of a where a, b ∈ A}
Since A is a set of students in a boys school, there will be no girl. Therefore, relation R will not
contain any element, and it will be a void relation.
⇒R=φ

Universal relation

It is a relation in which each element of set A is related to every element of set A.


⇒R=A×A

Example

Let A = {Set of real numbers}


R = {(a, b) : |a - b| ≥ 0, a, b ∈ A}
We know that a - b is a real number.
So, |a - b| will also be a non-negative real number.
⇒ |a - b| ≥ 0 will hold true for every value of a and b.
Therefore, R will be a universal relation.

© 2020, BYJU'S. All rights reserved


04

Identity relation

The relation on set A is an identity relation if each and every element of A is related to itself only.

Example A A

Let A = {Set of integers} -2 -2


Relation R = {(a, b) : a = b, a, b ∈ A} -1 -1
Both the elements of ordered pair (a, b) are equal. 0 0
Therefore, it is an identity relation. 1 1
⇒ R = {(a, b) : a = b where a, b ∈ A} = IA 2 2

Reflexive relation

A relation R on set A is said to be reflexive if every element of A is related to itself, i.e., relation R is
reflexive if (a, a) ∈ R ∀ a ∈ A

Note

• I ⊆ R, where I is an identity relation and R is an reflexive relation on A.


• Every identity relation is a reflexive relation but every reflexive relation is not an identity
relation.

Note

In an identity relation, every element of a set is related to itself only. However, in a reflexive
relation, every element is related to itself and to other elements as well.

Example

Let’s take A={1, 2, 3} then


R1 = {(1, 1), (2, 2), (3, 3)} is reflexive and identity.
R2 = {(1, 1), (1, 2), (2, 2), (1, 3), (3, 3)} is reflexive but not identity.

© 2020, BYJU'S. All rights reserved


05

Check whether the given relations are reflexive or not.


1. Relation R is defined on a set of natural numbers. R={(a, b) : a is divisible by b, a, b ∈ ℕ}
2. Relation R is defined on a set of straight lines in a plane, R = {(L1, L2) : L1|| L2}
3. Relation R is defined on a set of straight lines in a plane, R = {(L1, L2) : L1 ⊥ L2}

Solution

1. Relation R is defined on a set of natural numbers.


R = {(a, b): a is divisible by b, a, b ∈ ℕ}
We know that a number divides itself. So, (a, a) ∀ a ∈ ℕ will be an element of relation R.
So,
IA = {(1, 1), (2, 2), (3, 3).....} will be a subset of R, i.e., IA ⊆ R
Therefore, R is a reflexive relation.
2. Relation R is defined on a set of straight lines in a plane.
R = {(L1, L2):L1|| L2}
For a reflexive relation, L1|| L1. Here, L1 and L2 are the same, i.e., coincident. Hence, they are
parallel.
Therefore, R is a reflexive relation.
3. Relation R is defined on a set of straight lines in a plane.
R = {(L1, L2) : L1 ⊥ L2}
For a reflexive relation, L1 ⊥ L2. Since L1 and L2 are the same, i.e., coincident lines, they cannot
be perpendicular to each other.
Therefore, R is not a reflexive relation.

Symmetric relation

Relation R on set A is said to be symmetric iff (a, b) ∈ R ⇒ (b, a) ∈ R


a R b ⇒ b R a ∀ ( a, b ∈ A)
It is read as ‘a is related to b implies that b is related to a for all (a, b) ∈ A.

Example:

Consider a set, A = {1, 2, 3}. Identify the symmetric relations among the following:
(1) R1 = {(1, 1), (1, 2), (2, 1), (1, 3), (3, 1)}
(2) R2 = {(1, 1), (1, 2), (2, 1), (1, 3)}
(3) R3 = {(1, 1), (2, 2), (3, 3)}
Solution
(1)  Here, we have, (1, 2) and (2, 1);(1, 3) and (3, 1). For (1, 1), we know that on interchanging the
position of the first and the second element in the ordered pair (1, 1), we will get (1, 1). Thus, R1
is a symmetric relation.
(2) R  2 is not a symmetric relation because it does not have the corresponding ordered pair (3, 1)
for (1, 3).
3) F  or (1, 1), (2, 2), (3, 3), we know that on interchanging the position of the first and the second
element in the ordered pair (1, 1), we will get (1, 1). Similarly we can do it for (2, 2), (3, 3). Thus,
R3 is a symmetric relation.

© 2020, BYJU'S. All rights reserved


06

Check whether the given relations are symmetric or not.


1. Relation R is defined on a set of natural numbers. R={(a, b) : a is divisible by b, a, b ∈ ℕ}
2. Relation R is defined on a set of straight lines in a plane, R = {(L1, L2) : L1|| L2}
3. Relation R is defined on a set of straight lines in a plane, R = {(L1, L2) : L1 ⊥ L2}

Solution

1. Relation R is defined on a set of natural numbers.


R = {(a, b): a is divisible by b where a, b ∈ ℕ}
We know that a is divisible by b. So, if b is also divisible by a, then a R b is symmetric.
Now, (6, 3) is a part of the relation. However, (3, 6) is not a relation as 3 is not divisible by 6.
Therefore, R is not a symmetric relation.
2. Relation R is defined on a set of straight lines in a plane.
R = {(L1, L2): L1 || L2}
If L1 || L2, then L2|| L1. Hence, they are parallel to each other.
Therefore, R is a symmetric relation.
3. Relation R is defined on a set of straight lines in a plane.
R = {(L1, L2) : L1 ⊥ L2}
If L1 ⊥ L2, then L2 ⊥ L1. Hence, they are perpendicular to each other.
Therefore, R is a symmetric relation.

Transitive relation

Relation R on set A is said to be transitive, iff we have,


(a, b) ∈ R , (b, c) ∈ R ⇒ (a, c) ∈ R ∀ a, b, c ∈ A
Or a R b , b R c ⇒ a R c, ∀ a, b, c ∈ A

Example

Consider a set, A={1, 2, 3}. Identify the transitive relations among the following:
1. R1 = {(1, 2), (2, 3), (1, 3)}
2. R2= {(1, 1), (1, 3), (3, 2)}
Solution
(1) Consider (1, 2) ≡ (a, b) and (2, 3) ≡ (b, c)
⇒ (1, 3) ≡ (a, c) ∈ R1
Similarly, we can check for different combinations of a, b, and c.
Therefore, R1 is a transitive relation.
(2) Consider (1, 3) ≡ (a, b) and (3, 2) ≡ (b, c)
However, (1, 2) ≡ (a, c) ∉ R2
Therefore, R2 is not a transitive relation.

© 2020, BYJU'S. All rights reserved


07

Check whether the given relations are transitive or not.


1. Relation R is defined on a set of natural numbers. R={(a, b) : a is divisible by b, a, b ∈ ℕ}
2. Relation R is defined on a set of straight lines in a plane, R = {(L1, L2) : L1|| L2}
3. Relation R is defined on a set of straight lines in a plane, R = {(L1, L2) : L1 ⊥ L2}

Solution

1. Relation R is defined on a set of natural numbers.


R = {(a, b): a is divisible by b where a, b ∈ ℕ}
We know that if a is divisible by b i.e., b is divisible by c, then a is divisible by c.
The relation is transitive.
For example (18, 6) and (6, 3), 18 is divisible by 6 and 6 is divisible by 3.
Then, 18 is divisible by 3.
Therefore, R is a transitive relation.
2. Relation R is defined on a set of straight lines in a plane.
R = {(L1, L2) : L1 || L2}
We know that L1 is parallel to L2, L2 is parallel to L3, then L1 is parallel to L3.
The relation is transitive.
If L1 || L2 and L2|| L3, then L1 || L3.
Therefore, R is a transitive relation.
3. Relation R is defined on a set of straight lines in a plane.
R = {(L1, L2) : L1 ⊥ L2}
We know that L1 ⊥ L2, L2 ⊥ L3. Then for relation to be transitive L1 ⊥ L3. But it’s clear that L1 and
L3 are parallel to each other.
The relation is not transitive.

Equivalence Relation

Relation R on a set A is said to be an equivalence relation on Example:


A iff,
• It is reflexive, i.e., (a, a) ∈ R ∀ a ∈ A An identity relation is an
• It is symmetric, i.e., (a, b) ∈ R ⇒ (b, a) ∈ R ∀ a, b ∈ A equivalence relation.
• It is transitive, i.e., (a, b) ∈ R, (b, c) ∈ R ⇒ (a, c) ∈ R ∀ a, b, c ∈ A Since (a, a) ∈ R ∀ a ∈ A
⇒ It is an equivalence relation

Anti-Equivalence Relation

Relation R on a set A is said to be an anti-equivalence relation on A in the following condition:


When a = b, a is equally related to b and b is equally related to a.

Example:

Check whether the given relations are anti-equivalence or not.

© 2020, BYJU'S. All rights reserved


08

1. x ≥ y
x ≥ y and y ≥ x
When x = y, both are possible simultaneously.
Therefore, it is an anti-equivalence relation.

2. x is divisible by y
When x = y, x is divisible by y and y is divisible by x.
Therefore, it is an anti-equivalence relation.

Show that the relation R = {(a, b): b lies within 1 km from a} is not an equivalence relation.

Solution

1. Reflexive relation: {(a, a): a lies within 1 km from a} is correct.


Therefore, R is a reflexive relation.
2. Symmetric relation: (a, b) ∈ R, i.e., b is within 1 km to a and (b, a) ∈ R, i.e., a is within 1 km to b.
Thus, both the relations are simultaneously possible.
Therefore, R is a symmetric relation.
3. Transitive relation: (a, b) ∈ R, i.e., b is within 1 km to a and (b, c) ∈ R, i.e., c is within 1 km to b.
However, it does not imply a and c are 1 km apart.
Therefore, R is not a transitive relation.
Therefore, R is not an equivalence relation.

Concept Check

1. If A = {1, 3, 5, 7}, B={1, 4, 9, 16, 25}, and R : A → B such that R = {(a, b): b = a2, a ∈ A, b ∈ B}, then
write relation R and also its domain and range.
2. Let A and B contain 2 and 4 elements respectively. Find the number of subsets of A × B having 3
or more elements.
3. Let ℝ be the set of real numbers.
Statement 1: A = {(x, y) ℝ × ℝ: y - x is an integer} is an equivalence relation on ℝ.
Statement 2: B = {(x, y) ℝ × ℝ: x = αy for some rational number α} is an equivalence relation.
(a) S tatement 1 is true, statement 2 is true, and statement 2 is the correct explanation of
statement 1.
(b) Statement 1 is true, statement 2 is true, and statement 1 is not the correct explanation of
statement 2.
(c) Statement 1 is true and statement 2 is false.
JEE MAIN 2011
(d) Statement 1 is false and statement 2 is true.

© 2020, BYJU'S. All rights reserved


09

Summary Sheet

Key Takeaways

• A set is a well-defined collection of objects.


• Let A and B be two non-empty sets, then the cartesian product denoted by A × B is the ordered
pair of elements. A × B = {(a, b): a ∈ A and b ∈ B}
• The domain of relation R is the set of the first element of all the ordered pairs of relation R.
• The range of relation R is the set of the second element of all the ordered pairs of relation R.
• If no element of set A is related to any element of set A, then relation R on set A is known as a
void or empty relation.
• A universal relation is a relation in which each element of set A is related to every element of
set A.
• If every element of A is related to itself, i.e., relation R is reflexive if (a, a) ∈ R ∀ a ∈ A, then relation
R on set A is said to be reflexive.
• In an identity relation, every element of a set is related to itself only. However, in a reflexive
relation, every element is related to itself and to other elements as well.
• Relation R on set A is said to be symmetric iff (a, b) ∈ R ⇒ (b, a) ∈ R
a R b ⇒ b R a ∀ (a, b) ∈ A
• Relation R on set A is said to be transitive iff we have the following:
(a, b) ∈ R , (b, c) ∈ R ⇒ (a, c) ∈ R ∀ a, b, c ∈ A
Or
a R b, b R c ⇒ a R c ∀ a, b, c ∈ A
• Relation R on a set A is said to be an equivalence relation on A iff,
» It is reflexive, i.e., (a, a) ∈ R ∀ a ∈ A
» It is symmetric, i.e., (a, b) ∈ R ⇒ (b, a) ∈ R ∀ a, b ∈ A
» It is transitive, i.e., (a, b) ∈ R, (b, c) ∈ R ⇒ (a, c) ∈ R ∀ a, b, c ∈ A
• Relation R on a set A is said to be an anti-equivalence relation on A in the following condition:
» When a=b, a is equally related to b and b is equally related to a.

© 2020, BYJU'S. All rights reserved


10

Mind Map

Relation

Domain and range of


Types of relation
relation

Void Universal Identity Reflexive Symmetric Transitive Equivalence


relation relation relation relation relation relation relation

Self-Assessment

Let R = {(x, y) : 2x + y = 41, x, y ∈ ℕ} be a relation defined on the set of natural numbers. Find the
domain and range of R.

Answers

Concept Check 1

We will map the relation to find the domain and range of R.


Given, R = {(a, b): b = a2, a ∈ A, b ∈ B}
a = 1 ⇒ b = 12 = 1; ( a, b ) ≡ (1, 1)
a = 3 ⇒ b = 32 = 9; ( a, b) ≡ (3, 9)
a = 5 ⇒ b = 52 = 25; ( a, b) ≡ (5, 25)
a = 7 ⇒ b = 72 = 49; ( a, b) ≡ (7, 49)
Therefore, R = {(1, 1), (3, 9), (5, 25)}
Domain is {1, 3, 5}
Range is {1, 9, 25}

Concept Check 2

n ( A ) = 2 and n ( B ) = 4
⇒ n ( A × B) = 2 × 4 = 8

© 2020, BYJU'S. All rights reserved


11

Total number of subsets having three or more elements = 8C3 + 8C4 + . . . + 8C8
= 28 - 8C0 + 8C1 + 8C2
= 256 - 1- 8 - 28 = 219

Concept Check 3

Step 1:
Consider the following:
(x, x) ∈ A
⇒y-x=x-x=0
The expression y - x is an integer.
Thus, A is reflexive.
Step 2:
(x, y) ∈ A ⇒ The expression y - x is an integer.
(y, x) ∈ A ⇒ The expression x - y is also an integer.
Thus, A is symmetric.
Step 3:
(x, y) ∈ A ⇒ The expression y - x is an integer.
(y, z) ∈ A ⇒ The expression z - y is an integer.
(x, z) ∈ A ⇒ The expression z - x is also an integer.
Thus, A is transitive.
Hence, A is an equivalence relation.
Step 4:
Consider the following:
(x, x) ∈ B
⇒ x = αx
⇒α=1
Since α is a rational number, B is reflexive.
(x, y) ∈ B ⇒ x = αy (For some rational α)
(y, x) ∈ B ⇏ y = αx (For some rational α)
(Example: x = 0, y = 1 : x = α × y ⇒ 0 = α × 1= 0;
y = αx ⇒ 1 = α × 0 No rational α exists)
Thus, B is not symmetric.
Hence, B is not an equivalence relation.
Thus, option (c) is the correct answer.

Self-Assessment

We know that x, y ∈ ℕ
For x = 1, y = 39
For x = 2, y = 37
. .
. .
. .
For x = 20, y = 1 Domain of R = {1, 2, 3, ....., 20}
For x = 21, y = -1 ∉ ℕ Range of R = {1, 3, 5, ...., 39}

© 2020, BYJU'S. All rights reserved


NOTE
M A T H E M A T I C S

RELATIONS AND FUNCTIONS


INTRODUCTION TO FUNCTIONS

What you already know

• Set theory • Cartesian product • Relations

What you will learn

• Function • Domain, co-domain and range

Show that relation R defined on the set of real numbers such that R = {(a, b) : a > b} is
 transitive.

Solution

Let (a, b) ∈ R and (b, c) ∈ R.


⇒ a > b and b > c
⇒a>c
⇒ (a, c) ∈ R
Therefore, R is a transitive relation.

Let T be the set of all the triangles in a plane with R as a relation given by R = {(T1, T2):
 T1 is congruent to T2}. Show that R is an equivalence relation.

Solution

(1) Every triangle is congruent to itself.


Therefore, R is reflexive.
(2) (T1, T2) ∈ R : T1 is congruent to T2.
⇒ T2 is congruent to T1.
Therefore, R is symmetric.
(3) (T1, T2) ∈ R : T1 is congruent to T2.
(T2, T3) ∈ R : T2 is congruent to T3.
⇒ T1 is congruent to T3.
Therefore, R is transitive.
Thus, the given relation R is an equivalence relation.

© 2020, BYJU'S. All rights reserved


02

Functions

If A and B are two non-empty sets such that each element of A is associated with a unique
element of B under the rule f: A → B (where A is the domain and B is the codomain of the
function), then f: A → B; y = f(x) is known as the function.
Here, y is a dependent variable and x is an independent variable.

: Note

• A function is also known as mapping, association, or correspondance.


• Range ⊆ Codomain

Example:

If A = {1, 2, 3, 4} and B = {5, 6, 7, 8, 9}, then represent f(x) = x + 4 using mapping.

A f(x) = x + 4 B A f(x) = x + 4 B

1 5 1 5 Range
6 6
2 2
7 7
3 3
8 8
4 9 4 9 Co-domain
Domain

Here, Range ⊆ Codomain


Also, f(1) = 5
Here, 5 is the image of 1 and 1 is the preimage of 5.
Set of preimages = {1, 2, 3, 4}
Set of images = {5, 6, 7, 8}

Function Identification

A function can be identified by the following three methods:

Using mapping

By rules

Using vertical line test

© 2020, BYJU'S. All rights reserved


03

By rules

• For every x, there should be a y.


• For every x, there should be only one y.

Mapping method

f f f f

A B A B A B A B
Diagram 1 Diagram 2 Diagram 3 Diagram 4

1. All the inputs have only one output. It is a function.


2. All the inputs have only one output. It is a function.
3. The first element has two outputs. It is not a function.
4. The last element does not have any output. It is not a function.

If X = {a, b, c, d, e} and Y = {p, q, r, s, t}, then which of the following subset(s) of X × Y is/
 are function(s) from X to Y.
(a) {(a, r) (b, r) (b, s) (d, t) (e, q) (c, q)} (b) {(a, p) (b, t) (c, r) (d, s) (e, q)}
(c) {(a, r) (b, p) (c, t) (d, q)} (d) {(a, r) (b, r) (c, r) (d, r) ( e, r)}

Solution

(a) (b)

a r a p
b t
b s
c r
d s
e q

© 2020, BYJU'S. All rights reserved


04

(c) (d)

a r a r
b p b
c t c
d q d
e e

From the diagram, it is clear that,


a. b has two outputs. It is not a function.
b. All the inputs have only one output. It is a function.
c. Element e does not have any output. It is not a function.
d. All the inputs have only one output r. It is a function.
Thus, options (b) and (d) are the correct answers.


Vertical line test

A graph is known to be a function if the vertical line drawn parallel to the Y-axis does not intersect
the graph at more than one point in the domain.
Examples

The vertical line intersects the graph of y = x2 The vertical line intersects the graph of
at one point only. x2 + y2 = 9 at two points.
Hence, y = x2 is a function. Hence, x2 + y2 = 9 is not a function.
Y
Y
y = x2
x2 + y2 = 9
X
X

© 2020, BYJU'S. All rights reserved


05

The vertical line intersects the graph of y2 = x The vertical line intersects the graph of y = x3
at two points. at one point only.
Hence, y2 = x is not a function. Hence, y = x3 is a function.
Y Y x3 = y
y2 = x

X X

Graphical method

A graph can be made by the following three methods:

Using wavy curve method This method is applicable only in those


functions whose factorization is possible.

Using transformation

Using calculus

Wavy Curve Method

Draw the graph of y = x2 - 5x + 6 Draw the graph of y = x3 - 6x2 + 11x - 6


⇒ y = (x - 2)(x - 3) ⇒ y = (x - 1)(x - 2)(x - 3)

Y Y

- + - +
X
1 2 3

+ - +
X
2 3

© 2020, BYJU'S. All rights reserved


06

Draw the graph of y = x2 - x Draw the graph of y = x(x - 1)2


⇒ y = x(x - 1)
Y
Y

- + +
X
0 1

+ - +
X
0 1

Draw the graph of y = (x - 2)2(x - 4)4 Draw the graph of y = x2, x4, x8

Y Y x8 x4
x2

+ + + + +
X X
2 4 -1 1

Draw the graph of y = x3, x7 Draw the graph of y = x3 - x


⇒ y = x(x2 - 1)
Y
x7 x3 Y

- +
X
- + - +
X
-1 1

Y
Draw the graph of y = 9 - x 2

y = -(x + 3)(x - 3)
- + -
X
-3 3

© 2020, BYJU'S. All rights reserved


07

Note

y = xeven y = x(2n+1) ;n ∈ ℕ
Y Y

Domain, Co-domain, and Range of a Function

• If function f is defined from a set A to a set B, then for f : A → B.


A is known as the domain of function f and B is known as the co-domain of function f.
The set of f-images of elements of A is known as the range of function f.
Domain: All the possible values of x for which f(x) exists
Range: All the possible values of f(x) for all the values of x

Note

• Range is also known as a set of images or height of graph.

Function Domain Range

Polynomial function ℝ ℝ

Identity function ℝ ℝ

Constant function ℝ {c} (Value of constant)

Reciprocal function ℝ0 ℝ0

Signum function ℝ {-1, 0, 1}

© 2020, BYJU'S. All rights reserved


08

ax + b; a, b ∈ ℝ ℝ ℝ

ax3 + b; a, b ∈ ℝ ℝ ℝ

x2, |x| ℝ ℝ+ ∪ 0 i.e., [0, ∞)

x3 ℝ ℝ

x + |x| ℝ ℝ+ ∪ 0 i.e., [0, ∞)

x - |x| ℝ ℝ- ∪ 0 i.e., (-∞, 0]

[x] ℝ ℤ

x - [x] ℝ [0, 1)

x
ℝ0 {-1, 1}
x

x [0, ∞) [0, ∞)

ax (a > 0) ℝ ℝ+ i.e., (0, ∞)

log x ℝ+ i.e., (0, ∞) ℝ

sin x ℝ [-1, 1]

cos x ℝ [-1, 1]

π
tan x ℝ - ( 2n + 1)  ; n ∈ ℤ ℝ
 2

cot x ℝ - {nπ}; n ∈ ℤ ℝ

© 2020, BYJU'S. All rights reserved


09

π
sec x ℝ - ( 2n + 1)  ; n ∈ ℤ (-∞, -1] ∪ [1, ∞)
 2

cosec x ℝ - {nπ}; n ∈ ℤ (-∞, -1] ∪ [1, ∞)

 π π
sin-1 x [-1, 1] − 2 , 2 
 

cos-1 x [-1, 1] [0, π]

 π π
tan-1 x ℝ − 2 , 2 
 

cot-1 x ℝ (0, π)

π
sec-1 x (-∞, -1] ∪ [1, ∞) [0, π] -  
2

 π π
cosec-1 x (-∞, -1] ∪ [1, ∞)  − 2 , 2  - {0}
 

Concept Check

Find the domain and range of the following: f(x) = x4 + x2 + 4

Summary Sheet

Key Takeaways

• A
 function is a relation defined from set A to set B when it has the following properties:
(i) Each element of A is associated to some element in B.
(ii) The association is unique.
• A graph is known to be a function if the vertical line drawn parallel to the Y-axis does not
intersect the graph at more than one point in the domain.
• Domain: It is the value of set A for which a function is defined.
• Range: It consists of all the values that the function gives.
• Co-domain: It consists of the set of all the elements in set B. (Range ⊆ Co-domain)


© 2020, BYJU'S. All rights reserved


10

Mind Map

By rules
Definition

Functions Identification of function Mapping method

Domain, co-domain and


range Vertical line test

Self-Assessment
q

Find the domain and range of log (x - 2)

Answers

Concept Check

Given, f(x) = x4 + x2 + 4
Since f(x) is a polynomial function, its domain is ℝ.
Let y = x4 + x2 + 4
1 1
⇒ y = x4 + x2 + 4 + - ( To make it perfect square )
4 4
2
 2 1 15
⇒ y = x +  +
 2 4
1 15
⇒y≥ +
4 4
⇒y≥4
∴ Range: [4, ∞)

Self Assessment

For a logarithmic function, the argument is always positive.


⇒x-2>0
⇒x>2
∴ Domain: (2, ∞)
Range: f(x) ∈ ℝ

© 2020, BYJU'S. All rights reserved


NOTE
M A T H E M A T I C S

RELATIONS AND FUNCTIONS


CLASSIFICATION OF FUNCTIONS

What you already know

• Relation • Function • Domain, codomain, and range

What you will learn

• Polynomial function • Constant function


• Identity function • Exponential function

Classification of Functions

ILATE
Algebraic Non-Algebraic / Transcendental

Inverse
Polynomial Rational Irrational Piecewise Logarithmic
function function function function
Trigonometric
Exponential
Modulus Greatest Fractional Signum Defined
function integer part function function
function function
(G.I.F) (F.P.F)

Polynomial function

Function of the form f(x) = a0xn + a1xn - 1 + . . . + an, where n is a positive integer, a0, a1, . . . , an are
constants, and n ∈ 𝕎 is known as a polynomial function.

Note

• If xn is the highest degree term in the polynomial, then that function is known as a
polynomial function of degree n.
• The coefficient of xn is known as a leading coefficient. Also, a0 ≠ 0

© 2020, BYJU'S. All rights reserved


02

f(x) = a (Constant form)


f(x) = ax + b (Linear form)
f(x) = ax2 + bx + c (Quadratic form)
Polynomial
f(x) = ax3 + bx2 + cx + d (Cubic form)
functions .
.
.
.
.
.
.

Odd degree and even degree polynomials

1. In an odd degree polynomial function, the highest degree is odd.


Domain: ℝ
Range: ℝ

Y Y
Example: y = x3 y = x3 - 6x2 + 11x - 6
y=x Y

X X
X
1 2 3

Domain: ℝ Domain: ℝ Domain: ℝ


Range: ℝ Range: ℝ Range: ℝ

2. In even degree polynomial function, the highest degree is even.


Domain: ℝ
Range: Depends on the function

The values of x for which the polynomial becomes zero are called zeroes, and below
are few cases in which we justify that the total number of zeroes is the degree of
polynomial incase of distinct real roots (maximum possible).
Y Y
y = ax + b y = ax2 + bx + c; a > 0

X X

1st degree polynomial with 1 root 2nd degree polynomial with 2 roots

© 2020, BYJU'S. All rights reserved


03

Y Y
y = ax3 + bx2 + cx + d y = ax4 + bx3 + cx2 + dx + e; a < 0

X X

3rd degree polynomial with 3 roots 4th degree polynomial with 4 roots

Note

For a fourth degree polynomial function For a cubic polynomial function


y = ax4 + bx3 + cx2 + dx + e, y = ax3 + bx2 + cx + d,
when a > 0, the graph is first opening when a > 0, the graph is opening
upwards from left and when a < 0, it is downwards from left and when a < 0,
opening downwards from the left. it is opening upwards from the left.

a>0 a>0

a<0 a<0

It is applicable to all the even degree It is applicable to all the odd degree
polynomial functions. polynomial functions.

f(x) = a0xn + a1xn - 1 + .......+ an = a (x - α1) (x - α2) .... (x - αn), where α1, α2, ...., αn are the zeros of the
given polynomial.

© 2020, BYJU'S. All rights reserved


04

Note

Repeated roots
f(x) = a(x - 4)2; a > 0 f(x) = a(x - 2)2(x - 4); a < 0 f(x) = a(x - 6)4; a > 0
Both roots coincide at x = 4 Two roots coincide at x = 2 Four roots coincide at x = 6

4 6
2 4

Quadratic function

If n = 2, we get, P(x) = a2x2 + a1x + a0


On considering the standard quadratic
polynomial function, f(x) = ax2 + bx + c, we get, Y
 2  bb 2 bcc  c  a>0
ff (( xx ))==f (aax)=xx 2 a+ + x xx++
a
+ x +
  a aaa  a 
 2 b  bb 2 b cc 2  cbb 2 b  2 
2 2
b
= a x + b x + b
 x a+ x x++  x + +  −+ −   
2 + − −
= a= 2
 a
a a 2a
2a  2aaa  a2a

2a   
  2a 
2a
  X
  b  2 4ac 2
b 22 − b 2 
 −
2
= a  =xa+ bx + +b 4ac − 4ac
b
+ 2  2 
= a  x +2a  +  4a D
 2a  2a  4a 2 4a   −
4a  b D
  b 2 2
D  D 
2
 b  b D  − 2a , − 4a 
= aa =xxa+
= +2ax + − − 4a − 2
2
  
 2a  2a 4a   4a 2 
 
2
 D  = Da  x + bb  b 
D   2 2

⇒  y⇒ y +
+ 4ay += a  x= +a 2ax + 
 4a  4a  2a  2a 
Y
a<0  b D
 − 2a , − 4a 
D  
For a > 0, we get, −
4a
 D 
Range:  − , ∞ 
 4a 
For a < 0, we get, X
b
 D −
Range:  −∞ , −  2a
 4a 

© 2020, BYJU'S. All rights reserved


05

Note

If n is even, P(x) is called even degree If n is odd, P(x) is called odd degree
polynomial whose range is always subset of ℝ polynomial whose range is ℝ

y = x2 y = x3 Y
Y

X
X

Find the domain and range of the following functions:


(i) f(x) = sin2x + cos2x (ii) f(x) = x2 + 3x + 1

Solution

(i) Here, f(x) is defined for all the values of x. (ii) Here, f(x) is defined for all the values of x.
∴ The domain of f(x) is x ∈ ℝ ∴ The domain of f(x) is x ∈ ℝ
sin2x + cos2x = 1 f(x) = x2 + 3x + 1
9 9  To make
9  Toit a  it a 
a
itmake
∴ The range of the function is f(x) ∈ {1} ⇒ f ( x ) =
⇒ xf2 + 3x +2 9 + 1 − 99  To make
2
⇒ f ( x ) = x (+ )3x + 4 + 1 − 4  perfect square 
x = x + 3x + + 1 − 
4 44  perfect square square 
4  perfect
2
 3
3  − 5
2
5 3≥ − 5
2
55 5
= x +
=  x + =  x
− + ≥ − − ≥ −
2   
 2   4 4 2 4 44 4
⇒ ff (( xx )) is  x55 , ∞) 5
⇒ The
The range
⇒ Theof
range of the
the function
range of the function
function is f−
(
− 4), is∞) − 4 , ∞ )
 4

Identity function Y
y=x

• y = x is an identity function.
Domain: ℝ
Range: ℝ X
• It is represented by I or IA.

© 2020, BYJU'S. All rights reserved


06

Constant function
Y

In a polynomial function
(0, c )
f(x) = anxn + an - 1xn - 1 + . . . + a0, if n = 0, we get, y=c
f (x) = c → a constant
So, in this case, X
Range: {c }

π π 5
) sin2 x + sin2  x +  + cos x·cos  x +  and g  = 1, then find gof ( x ) .
If f ( x =
 3  3 4

Solution
2
  π   π
f (x) = sin x +  sin  x +   + cos x·cos  x + 
2

  3   3
2
 1 3  1 3
=sin x +  sin x · + cos x ·
2
 + cos x ·  cos x · − sin x · 
 2 2   2 2 
 
sin2 x sin x 3 cos x 3cos2 x cos2 x 3
=sin2 x + + 2· · + + − · sin x· cos x
4 2 2 4 2 2
5 5
= sin2 x + cos2 x
4 4
5
∴f ( x )
4
is a constant = (
function.  sin2 x + cos2 x 1 )
5  5
Now,= g   g= ( f ( x ) ) 1 = f ( x )
4 
4 
∴ gof ( x ) = 1

Note

 π  π 5
(i ) sin2 x + sin2  x +  + cos x· cos  x +  =
 3  3 4
π π 3
( ii ) cos2 x + cos2  x +  − cos x · cos  x +  =
 3  3 4

© 2020, BYJU'S. All rights reserved


07

Exponential function

y = ax , a > 0, and a ≠ 1 i.e., The function in the form of constant variable is known as an exponential
function.

Y a>1 Y 0<a<1

X X

Domain: x ∈ ℝ, Range: x ∈ ℝ+

Find the domain and range of the following functions:


(i) f(x) = e2x (ii) f(x) = ex + 1

Solution
Y
f(x) = e2x

(i) f(x) is defined for all the values of x.


∴ Domain: x ∈ ℝ
f(x) > 0, ∀ x ∈ ℝ
∴ Range: f(x) ∈ (0, ∞)

Y
f(x) = ex + 1

(ii) f(x) is defined for all the values of x.


∴ Domain: x ∈ ℝ
ex > 0
⇒ ex + 1 > 1
∴ Range: f(x) ∈ (1, ∞) y=1
X

© 2020, BYJU'S. All rights reserved


08

Note

In general when any function raised to a variable exponent is given, then it means that the
function will take only positive values of the domain.
Example

( i ) ( sin x )
x

This function is valid only when sin x > 0


∴ Domain = x ∈ ( 2nπ, ( 2n + 1) π )

X
-2π -π 0 π 2π 3π 4π

Y y = 12x

y = 4x

( ii ) 12x and 4x
for x < 0, 4x graph will be above 12x . y = 4x

for x > 0, 12x graph will be above 4x .


y = 12x
y=0
4x > 12x 12x > 4x
When x < 0 When x > 0

Find the domain of f(x) = 2x - 5x

Solution

For y f ( x ) to exist, f ( x ) ≥ 0
∴2x − 5x ≥ 0
⇒ 2x ≥ 5x
This is possible only when x ≤ 0
∴ Domain : x ∈ ( −∞ , 0]

© 2020, BYJU'S. All rights reserved


09

Find the domain of the following:


(i) log10 (1 + x3) (ii) log10 (log10 (1 + x3))

Solution

(i) log10 (1 + x )
3

For this function to exist, 1 + x3 > 0


(
⇒ ( x + 1) x2 − x + 1 > 0 )
Now, for x2 − x + 1,
(1) − 4 (1)(1) =
2
D= −3
We know that a quadratic expression having D < 0 is always positive.
⇒ ( x + 1) > 0
⇒ x > −1
∴ Domain : x ∈ ( −1, ∞ )

(
(ii) log10 log10 1 + x3 ( ))
For this function to exist,
Step 1 :
(
log10 1 + x3 > 0 )
⇒ 1 + x > 100 =
1 3

⇒ x3 > 0 100 =
1 ( )
⇒x>0
Step 2 :
1 + x3 > 0
⇒ x > −1
∴ Domain : x ∈ ( 0, ∞ )

Concept Check

1. Find the domain and range of the function: f(x) = 3x + 5


cos2 x + sin 4 x
2. If f ( x ) = , then find f ( 2015) .
cos4 x + sin2 x
3. Find the domain of ff ( xx )=
= 12 − x 9
xx12 − x9 ++ xx 4 −
4
− xx +
+11
1
1
of ff ( xx )
4. Find the domain=
= +
+ xx +
+22
log10
log 10
( 1
1 −
− x
x )

© 2020, BYJU'S. All rights reserved


10

Summary Sheet

Key Takeaways

Function Domain Range

Polynomial function ℝ ℝ

Identity function ℝ ℝ

Constant function ℝ {c} (Value of constant)

ax (Exponential function) ℝ ℝ+ = (0, ∞)

Mind Map

Odd degree Even degree

Polynomial function

Identity function
Classification of
Functions
functions
Constant function

Exponential function

Self-Assessment

Find the range of function f(x) = ln(x2 - 2x + 3)

© 2020, BYJU'S. All rights reserved


11

Answers

Concept Check

1.
Here, f(x) is defined for all the values of x.
∴ Domain: x ∈ ℝ
Range: f(x) ∈ ℝ

2.
(( )) (( ))
2
=
Numerator cos22 x + 1 − cos22 x 2  sin22 x + cos22=
x 1
=
Numerator cos x + 1 − cos x  sin x + cos = x 1
= cos2 x + 1 − 2cos2 x + cos4 x
2 2 4
= cos x + 1 − 2cos x + cos x
= 1 − cos22 x + cos44 x
= 1 − cos x + cos x
= sin2 x + cos4 x  sin2 x + cos2 x = 1
= sin2 x + cos4 x  sin2 x + cos2 x = 1 (( ))
= Denominator
= Denominator
sin22 x + cos44 x
∴ f ( x ) sin 4 x=
= + cos2 x 1 is a constant fucntion.
∴ f ( x ) cos4 x=
= + sin x 1 is a constant fucntion.
cos x + sin2 x
∴ f ( 2015) = 1
∴ f ( 2015) = 1

3.
For f ( x=
) x12 − x 9 + x 4 − x + 1 to exist, x12 − x 9 + x 4 − x + 1 ≥ 0.
Case 1: x ∈ [0, 1)
( )
x12 [ + ve] − x 9 + x 4 [ + ve] ( − x + 1 ) [ + ve] ⇒ + ve
Case 2: x ∈ [1, ∞ )
(x 12
) ( )
− x 9 [ + ve] x 4 − x [ + ve] ( +1) [ + ve] ⇒ + ve
Case 3: x ∈ [ −∞ , 0)
( ) ( )
x12 [ + ve] − x 9 [ + ve] + x 4 [ + ve] ( − x ) [ + ve] ( +1) [ + ve] ⇒ + ve
x∈
∴ Domain =

4.
( x ) log1 − x 10 +
For f= x + 2 to exist,
Case 1. 1 − x ≠ 1 ⇒ x ≠ 0
Case 2. 1 − x > 0 ⇒ x < 1
Case 3. x + 2 ≥ 0 ⇒ x ≥ −2
Considering cases 1, 2 and 3,
Domain : x ∈ ( −2, 0) ∪ ( 0, 1)

© 2020, BYJU'S. All rights reserved


12

Self-Assessment

Given, f(x) = ln(x2 - 2x + 3)


Base = e > 0 (not equal to 1); this means that the graph of this function will be similar to the graph
of logarithmic function where a > 1
We know that for log x, x must be greater than zero.
⇒ x2 - 2x + 3 > 0
⇒ (x - 1)2 + 2 > 0
⇒ x2 - 2x + 3 ≥ 2 (As (x - 1)2 ≥ 0 ∀ x ∈ ℝ, the domain is x ∈ ℝ)
On taking ln on both the sides, we get,
ln(x2 - 2x + 3) ≥ ln 2
(In an increasing function, the inequality sign does not change)
⇒ f(x) ≥ ln2
Range: f(x) ∈ [ln2, ∞).

© 2020, BYJU'S. All rights reserved


NOTE
M A T H E M A T I C S

RELATIONS AND FUNCTIONS


SOLVING PROBLEMS ON FINDING
DOMAIN OF FUNCTIONS

What you already know What you will learn

• Definition of a function • Finding the domain of various functions


• Domain and range of a function
• Types of functions

Domain

To find domain of basic types of Method to find the domain if addition or subtraction of
functions: two or more functions are given,
1 h(x) = f (x) ± g (x)
If y = then f (x) ≠ 0
f (x)
↓ ↓ ↓
If y = f ( x ) then f (x) ≥ 0 D = D1  D2
If y =
1
then f (x) > 0 where D, D1 and D2 are domains of h ( x ) , f ( x ) and g ( x ) .
f (x)

Find the domain of the following functions:

Solution

1 By taking intersection of Case 1 and Case 2, we get


1. f ( x ) = x2 - x - 20 + domain as follows:
x2 - 5x -14
Case 1 : x2 - x - 20 ≥ 0 x ∈ ( -∞ , - 4]  (7, ∞ )
⇒ ( x - 5)( x + 4 ) ≥ 0
⇒ x ≤ -4  x ≥ 5
⇒ x ∈ ( -∞ , -4]  [5, ∞ )
Case 2 : x2 - 5x - 14 > 0
⇒ ( x - 7 )( x + 2) > 0
⇒ x < -2  x > 7
⇒ x ∈ ( -∞ , -2)  (7, ∞ )

© 2021, BYJU'S. All rights reserved


02

1
2. f ( x ) =
x -x

Analytical method Graphical method Hit and trial


|x| - x > 0 |x| > x |x| > x
Y Let x = 3, 3 > 3
When x ≥ 0, When x < 0,
Not possible
⇒x>x ⇒-x>x
Let x = -3, 3 > -3
Not possible ⇒ 2x < 0
True
⇒x<0 y = -x y=x Let x = 3.1, 3.1 > 3.1
⇒ x ∈ (-∞, 0) Not possible
Let x = -3.1, 3.1 > -3.1
y=x X We can see that the
inequality holds for
negative values of x
⇒ x ∈ (-∞, 0) ⇒ x ∈ (-∞, 0)

1
3. f ( x ) = Y
x- x

x - |x| > 0
⇒ |x| < x y = |x|
From the graph, we can see that the green line does not
exceed the yellow line at any point.
x=Φ X
y=x

4. f ( x ) = sin x + 16 - x2
sin x ≥ 0 is possible if y = sinx, is above Y
the X-axis.
1 y = sin x
16 - x2 ≥ 0
⇒ x2 - 16 ≤ 0
⇒ (x - 4)(x + 4) ≤ 0 -2π -4 -π O π 4 2π X
⇒ -4 ≤ x ≤ 4 -1
By taking intersection of above two sets
of x we get domain as
x ∈ [-4, -π] U [0, π]

Note

For the composite function f(g(x)) we have the rule,


Domain of f(g(x)) = Domain of g(x), if Domain of function f is ℝ.

© 2021, BYJU'S. All rights reserved


03

Find the domain of the following functions:

Solution

1. f ( x ) = sin ( x-1 + 6-x ) 2. f ( x ) = 4


- ( x-1 + 6-x )

f ( x ) = sin ( g ( x ) ) ; g ( x ) = x - 1 + 6 - x f (x) = 4
-g ( x )
;
⇒ Domain of f ( x ) =  g(x) = x - 1 + 6 - x
We know, Domain of f ( g ( x ) ) = Domain of g ( x ) Domain of x - 1 + 6 - x ,
Domain of x - 1 + 6 - x , x - 1 ≥ 0⇒ x ≥ 1
x -1 ≥ 0⇒ x ≥ 1 6-x ≥ 0 ⇒ x ≤ 6
6-x ≥ 0 ⇒ x ≤ 6 By taking intersection of above two sets of x
By taking intersection of above two sets of x we get domain as x ∈ [1, 6]
we get domain as x ∈ [1, 6]

3. f ( x ) = cos ( sinx )
For function to exist, cos ( sinx ) ≥ 0
Y
Step 1 :
f ( x ) = cos θ ; θ = sinx
⇒ cos θ ≥ 0 is that part of cos θ which is above X - axis.

Step 2 :
Also, θ = sin x ∈ [-1, 1] ;
i.e, θ is bounded to [-1, 1] . π -1 0 1 π
- X
2 2

Step 3 :
Part of cos θ in[-1, 1] is already above X - axis.
Hence, x ∈.

Note

1 1 1
1. Domain of ( f ( x ) ) , ( f ( x ) ) , ... , ( f ( x ) )
4 6 Even = Domain of f ( x )
1 1 1
2. Domain of ( f ( x ) ) 3 , ( f ( x ) ) 5 , ... , ( f ( x ) ) Odd = Domain of f ( x )

© 2021, BYJU'S. All rights reserved


04

1
 1 - x  12
Find the domain of the following function: f ( x ) =  
5 + x 

Solution
1
 1 - x  12
f (x) =  
5 + x 
1
Domain of ( f ( x ) ) Even = Domain of f ( x ) -5 1
1-x 1-x
⇒ Domain of ⇒ ≥ 0
5+x 5+x
⇒ x ∈ ( -5, 1]

Logarithmic Functions

Find the domain of the following functions:


   2log10 x + 1 
(
1. log 3  log 1 x 2 + 10x + 5  ) 2. f ( x ) = log100x 
 -x 

 3 

Solution

 
(
1. log3  log 1 x2 + 10x + 5  ) Step 3:
 3  For log 1 ( h ( x ) )
Step 1: 3

For log3 ( g ( x ) ) , Base =


1 1
> 0, Also , ≠ 1
3 3
Base = 3 > 0, Also, 3 ≠ 1

Step 4:
Step 2 : Now, for x 2 + 10x + 5 = 0,
(
Now, log 1 x 2 + 10x + 5 > 0 ) -10 ± 100 - 20
3 ⇒x =
2
⇒ x 2 + 10x + 5 < 1 ⇒ x2 + 10x + 4 < 0
-10 ± 2 20
For x 2 + 10x + 4 = 0, ⇒x =
2
-10 ± 100 - 16 ⇒ x = -5 ± 20
⇒x =
2 So, for x2 + 10x + 5 > 0
-10 ± 2 21
⇒x =
2
⇒ x = -5 ± 21 (
x ∈  - -5 - 20, -5 + 20 )
For x 2 + 10x + 4 < 0,
(
x ∈ -5 - 21, -5 + 21 ) Step 5:
( ) (
x ∈ -5 - 21, -5 - 20 ∪ -5 + 20, -5 + 21 )
© 2021, BYJU'S. All rights reserved
05

 2log10 x + 1 
2. f ( x ) = log100x  
 -x 
For function to exist,
1
a. Base = 100x > 0; x > 0, Also 100x ≠ 1; x ≠
100
2log10 x + 1 2log10 x + 1
b. >0⇒ <0
-x x
⇒ 2log10 x + 1 < 0 ⇒ 2log10 x < -1
1
1 - 1
⇒ log10 x < - ⇒ x < 10 2 ⇒ x <
2 10
c. In 2log10 x + 1, x > 0
Considering ( a) , (b ) and ( c ) ,
 1   1 1 
x ∈  0,   , 
 100   100 10 

Find the domain of the following functions:


1. log10 1 + x 3 2. f ( x ) = log10 4x - 1 ( )
3. log10 1 - log10 x 2 - 5x + 16 

Solution

1. log10 1 + x3 (
3. log10 1 - log10 x2 - 5x + 16  )
For function to exist, For function to exist,
a. Base = 10 ≠ 0, also 10 > 1 a. Base = 10 ≠ 0, also, 10 > 1
b. 1 + x3 ≠ 0
(
b. 1 - log10 x2 - 5x + 16 > 0 )
⇒ x ≠ -1 ⇒ x ≠ -1
3

⇒ x ∈  - {-1}
⇒ log10 (x 2
- 5x + 16 ) < 1 ⇒ x 2
- 5x + 16 < 10
⇒ x2 - 5x + 6 < 0 ⇒ ( x - 2)( x - 3) < 0 ⇒ 2 < x < 3
2. f ( x ) = log10 4x - 1 c. x 2 - 5x + 16 > 0
For function to exist, ⇒ D = 25 - 4 × 16 = -ve ⇒ x ∈ 
a. Base = 10 ≠ 0, also 10 > 1 ⇒ x ∈ ( 2, 3)
1
b. 4x - 1 ≠ 0 ⇒ x ≠
4
1 
⇒x ∈  -  
4

© 2021, BYJU'S. All rights reserved


06

Polynomial-Based Questions (Type 1)

If f(x) is a polynomial with degree 5 with leading coefficient 2 and f(1) = 1, f(2) = 2, f(3) =
3, f(4) = 4, and f(5) = 5, then find f(6) .

Solution

Step 1: Step 2:
From the step 1 calculations, we can conclude that,
Let g ( x ) = f ( x ) - x
1. g ( x ) is a polynomial having roots 1, 2, 3, 4, and 5.
So, g (1) = f (1) - 1 = 1 - 1 = 0
2. g ( x ) is also a 5 degree polynomial.
g ( 2) = f ( 2) - 2 = 2 - 2= 0
g ( 3) = f ( 3) - 3 = 3 - 3 = 0
g ( 4) = f ( 4) - 4 = 4 - 4 = 0
g ( 5) = f ( 5) - 5 = 5 - 5 = 0

Step 3:
Now, g ( x ) = ( x - 1)( x - 2)( x - 3)( x - 4 )( x - 5)
f ( x ) - x = 2( x - 1)( x - 2)( x - 3)( x - 4 )( x - 5) ( the leading coefficient is 2. )
⇒ f ( x ) = 2( x - 1)( x - 2)( x - 3)( x - 4 )( x - 5) + x
⇒ f ( 6 ) = 2( 6 - 1)( 6 - 2)( 6 - 3)( 6 - 4 )( 6 - 5) + 6 = 246

Concept Check

Find the domain of the following functions.


x+4 1
1. f ( x ) = 2. f ( x ) = 3. f ( x ) = cos2x + 4 - x2
ln ( x - 8 ) x2 - 4x
1
1 - 5x  1 - x  11
4. f ( x ) = -x 5. f ( x ) =   6. f ( x ) = log3 cos ( sinx )
7 -7 5 + x 
7. f ( x ) = log10 ( log e x )
8. If f(x) is a polynomial with degree 4 with leading coefficient 3 and f(1) = 2, f(2) = 5, f(3) = 10,
and f(5) = 26, then find f(4) .

© 2021, BYJU'S. All rights reserved


07

Summary Sheet

Key takeaways

• To find the domain of basic types of functions:


1
If y = then f (x) ≠ 0
f (x)
If y = f ( x ) then f (x) ≥ 0
1
If y = then f (x) > 0
f (x)
• When we need to find the domain of two or more functions,
h(x) = f (x) ± g (x)
↓ ↓ ↓
D = D1  D2
where D, D1 and D2 are domains of h ( x ) , f ( x ) and g ( x ) .
1 1 1
• 1. Domain of ( f ( x ) ) 4 , ( f ( x ) ) 6 , ... , ( f ( x ) ) Even = Domain of f ( x )
1 1 1
2. Domain of ( f ( x ) ) , ( f ( x ) ) , ... , ( f ( x ) )
3 5 Odd = Domain of f ( x )

Mind Map

Logarithmic functions

Functions Domain

Polynomial functions

Self-Assessment

3x + 4
 x -1
Find the domain of the function: f ( x ) =  
 x -2

© 2021, BYJU'S. All rights reserved


08

Answers

Concept Check

1. 2.
1 x 2 - 4x > 0
f (x) = x + 4 ×
ln ( 8 - x ) ⇒ x ( x - 4) > 0
= x + 4 × log 8-x ( e ) ⇒x < 0  x > 4
We know the base of the logarithmic ⇒ x ∈ ( −∞ ,0)  ( 4, ∞ )
function cannot be negative or 1. ⇒ x ∈  - [0, 4]
x+4≥0 8-x ≠1 8- x>0
⇒ x ≥ -4 ⇒x ≠7 ⇒ x<8
By taking intersection of above three
sets of x we get domain as x ∈ [-4, 8 ) - {7}

3.
cos 2x ≥ 0
Is possible if y = cos2x, is above the X-axis. Y
4 - x2 ≥ 0
1 y = cos 2x
⇒ x2 - 4 ≤ 0
⇒ (x - 2)(x + 2) ≤ 0
⇒ -2 ≤ x ≤ 2 π
- 3π -π O 3π X
By taking intersection of above two sets of 4 4 4 4
-1
x we get domain as
 π π
x ∈ - , 
 4 4

4.
1 - 5x
≥ 0
7-x - 7
Multiplying numerator and denominator by 7x
(5 x
)
- 1 7x
≥ 0
+ - +
7x+1 - 1 -1 0
Now finding the critical points,
5x - 1 = 0
⇒ 5x = 50
⇒x=0
7x + 1 - 1 ≠ 0
⇒ 7x + 1 ≠ 70
⇒ x ≠ -1
⇒ x ≥ 0 and x < -1
On taking intersection of above, we get domain as
x ∈ ( -∞, - 1)  [0, ∞ )

© 2021, BYJU'S. All rights reserved


09

5. 6.
f ( x ) = log3 cos ( sinx ) ; Now, for the function to exist,
1
1-x  11
f (x) =  
5 + x  a. Base = 3 > 0, also 3 ≠ 1
b. log3 cos ( sinx ) ≥ 0
1
⇒ Domain of ( f ( x ) ) Odd = Domain of f ( x )
1-x ⇒ cos ( sinx ) ≥ 1 i,e cos θ ≥ 1
⇒ Domain of ⇒ 5+x≠ 0
5+x But cos θ = 1 is the only possible solution.
⇒ x ≠ -5 ⇒ cos ( sinx ) = 1
⇒ x ∈  - {-5} ⇒ sinx = 0
⇒ x = nπ, n ∈

7.
f ( x ) = log10 log e x
For the function to exist,
a. Base = 10 ≠ 0, also , 10 > 1
b. log e x ≠ 0 ⇒ x ≠ 1. Also, x > 0
c. Base = e ≠ 0, also e > 1
⇒ x ∈( 0, ∞ ) - {1}

8.
Step 1:
Let g ( x ) = f ( x ) - x2 + 1 ( )
So, g (1) = f (1) - (1 + 1) = 2 - 2 = 0
g ( 2) = f ( 2) - ( 4 + 1 ) = 5 - 5 = 0
g (3) = f (3) - ( 9 + 1) = 10 - 10 = 0
g (5) = f (5) - ( 25 + 1) = 26 - 26 = 0

Step 2:
From the calculations, we can conclude that,
1. g ( x ) is a polynomial having roots 1, 2, 3, 5.
2. g ( x ) is also a 4 degree polynomial.

Step 3:
Now, g ( x ) = ( x - 1)( x - 2)( x - 3)( x - 5)
( )
f ( x ) - x2 + 1 = 3( x - 1)( x - 2)( x - 3)( x - 5) ( The leading coefficient is 3. )
⇒ f ( x ) = 3( x - 1)( x - 2)( x - 3)( x - 5) + x2 + 1 ( )
⇒ f ( 4 ) = 3( 4 - 1)( 4 - 2)( 4 - 3)( 4 - 5) + 17 = 17 - 18 = -1

© 2021, BYJU'S. All rights reserved


10

Self-Assessment

3x + 4
 x -1
As  is in the form of h(x) ( ) .
g x

 x -2 
Here,
g ( x ) = 3x + 4 is defined for  ….. ( 1)
x -1
h(x) = >0
x -2


(x - 1)( x - 2)
>0
( x - 2)
2

⇒ ( x - 1)( x - 2) > 0
By using the wavy curve method, we get the following :
x ∈ ( - ∞ , 1)  ( 2, ∞ ) ........ ( 2)
On taking the intersection of ( 1) and ( 2) , we get,
x ∈ ( - ∞ , 1)  ( 2, ∞ )

© 2021, BYJU'S. All rights reserved


NOTE
M A T H E M A T I C S

RELATIONS AND FUNCTIONS


FUNCTIONAL EQUATIONS

What you already know What you will learn

• Definition of a function • Polynomial based questions


• Domain and range of a function • Value of a function
• Types of functions • Functional equations

Polynomial Based Questions (Type 2)

Remainder theorem
• If a polynomial function f(x) is divided by x – a, then it leaves a remainder of f(a).
• If the remainder is 0, then f(a) is 0.
Example:
If a polynomial function f(x) when divided by x –1 leaves a remainder of 3, then find f(1).
By using the remainder theorem, we get,
f(1) = Remainder = 3

If a polynomial function f(x) when divided by (x – 1) and (x + 1) leaves remainder of 3 and


5 respectively, and when divided by x2  –   1 leaves g(x) as the remainder, then find g(2).

Solution

Step 1: Step 2:
By using the remainder theorem, we get, By solving equations (i) and (ii), we get,
f(1) = 3 and f(–1) = 5 a = –1
Now, f(x) = Divisor × Q(x) + R(x) b=4
Where, Q(x) = Quotient, R(x) = Remainder ⇒ g(x) = –x + 4
In this case, f(x) = (x2 – 1) × Q(x) + g(x) ∴ g(2) = –2 + 4 = 2
Also when f(x) is divided by x2 – 1 the
remainder will either be linear or constant.
⇒ f(x) = (x2 – 1) × Q(x) + ax + b
For x=1,
f(1) = (12 – 1) × Q(x) + a(1) + b
⇒ 3 = a + b ... (i)
For x= –1,
f(–1) = ((–1)2 – 1) × Q(x) + a(–1) + b
⇒ 5 = –a + b ... (ii)

© 2021, BYJU'S. All rights reserved


02

Polynomial Based Questions (Type 3)

For a polynomial function f(x),


1 1
If f ( x ) ·f   = f ( x ) + f   , then f ( x ) = 1 ± x n
x x
Example:
1 1
If f(x) is a polynomial satisfying f ( x ) · f   = f ( x ) + f   and f(2)= 9, then find f(3).
x x
Let f(x) = 1 + xn (f(2)= 9 > 0)
For x = 2,
f(2) = 1 + 2n ⇒ 9 = 1 + 2n ⇒ n= 3
⇒ f(x) = 1 + x3
⇒ f(3) = 1 + 33 = 28
1 1
If f ( x ) ·f   = f ( x ) + f   , then f ( x ) = 1 ± x n
x x
Proof:
Step 1 : St ep 2 :
By taking everything to the L.H.S  1  1
Now, let ( f ( x ) − 1) = g ( x ) and  f   − 1  = g  
1 1  x  x
f (x) · f   − f (x) − f   = 0
x x 1
∴ g (x) · g   = 1
Adding 1 on both sides x
1 1
f (x) · f   − f (x) − f   + 1 = 0 + 1
x x Step 3 :
 1   1  Now, the function ±x n satisfies this condition.
⇒ f (x) ·  f   − 1  − 1 f   − 1  = 1
 x   x  So, g ( x ) = ± x n
 1  ⇒ f ( x ) − 1 = ± xn
⇒ ( f ( x ) − 1)  f   − 1  = 1
 x  ⇒ f ( x ) = 1 ± xn

1 1
If f(x) is a polynomial satisfying f ( x ) · f   + 8 = 3f ( x ) + 3f   and f(3)= 30, then find
f(2). x x
1 1
Given , f ( x ) · f   + 8 = 3f ( x ) + 3f  
x x
Solution Step 1 :
By arranging the given equation, we get,
1 1
Given , f ( x ) · f   + 8 = 3f ( x ) + 3f   1 1
x x f ( x ) · f   − 3f ( x ) − 3f   + 9 = 1
x x
Step 1 :
 1   1 
By arranging the given equation, we get, ⇒ f ( x )  f   − 3  − 3 f   − 3  = 1
 x   x 
1 1
f ( x ) · f   − 3f ( x ) − 3f   + 9 = 1  1 
x x ⇒ ( f ( x ) − 3)  f   − 3  = 1
 1   1   x 
⇒ f ( x )  f   − 3  − 3 f   − 3  = 1
 x 
© 2021, BYJU'S. All rights reserved
 x 
03

Step 2 : Step 3 :
 1  1 Now, the function ± x n satisfies
Now, let ( f ( x ) − 3) = g ( x ) and  f   − 3  = g  
 x  x this condition.
1 ⇒ f ( x ) = 3 ± xn
∴ g(x) · g  = 1
x Also, given that f (3) = 30 > 0
Let the polynomial be
f (3) = 3 + 3n ⇒ 30 = 3 + 3n ⇒ n = 3
1 A
g ( x ) = Ax n ⇒ g ( x ) · g   = Ax n · n = A 2 ∴ f ( x ) = 3 + x3
x x
⇒ 1 = A ⇒ A = ±1
2 ⇒ f ( 2) = 3 + 23 = 11

∴ g ( x ) = ± xn

Value of a Function

Example:
x
If f(x) = ln x, then find f  .
y
 
x x
f   = ln   = ln x − ln y
y y
x
∴ f   = f (x) − f (y)
y

a
f 
x b
If f ( x ) = , then find   .
x+1 b
f 
a

Solution

a b
a a  b  b
f  = b = and f   = a =
b a + 1 a + b a b
+1 a+b
b a
a
f 
b a
∴  =
b b
f 
a

© 2021, BYJU'S. All rights reserved


04

 1 1
If f  x +  = x 2 + 2 , then find f ( x - 1 ) .
 x x Rearranging given expression we get,
 1 1
f  x +  = x2 + 2 + 2 − 2
Solution  x x
2
 1  1
Rearranging given expression we get, ⇒ fx +  = x +  − 2
 x  x
 1 1
f  x +  = x2 + 2 + 2 − 2 ⇒ f ( x ) = x2 − 2
 x x
⇒ f ( x − 1) = ( x − 1) − 2
2
2
 1  1
⇒ fx +  = x +  − 2
 x  x
⇒ f ( x ) = x2 − 2
⇒ f ( x − 1) = ( x − 1) − 2
2
Let x − y = p and x + y = q
If f ( x - y, x + y ) = x·y , then find arithmetic mean of f ( x, y )pand
+ qf ( y, x ) . q − p
∴x = and y =
2 2
p + q q − p q2 − p2
Solution ∴ f ( p, q ) = · =
2 2 4
y −x
2 2
Let x − y = p and x + y = q Similarly, f ( x, y ) = . . . (i)
4
p+q q− p
∴x = and y = Now A.M of f ( x, y ) and f ( y, x )
2 2
p + q q − p q2 − p2 y 2 − x2 x2 − y 2
∴ f ( p, q ) = · = f ( x, y ) + f ( y, x ) +
2 2 4 = = 4 4 =0
2 2
y 2 − x2
Similarly, f ( x, y ) = . . . ( i)
4
Now A.M of f ( x,yn)−and 3, xf≥( 100
y, x )
If f ( x ) =  , then find the digit at the unit's place of f ( 84 ) .
f ( f ( ny+
2 5 ) )2, x <21002
−x x −y
f ( x, y ) + f ( y, x ) +
= = 4 4 =0
2 2
Solution

f ( 84 ) = f ( f ( 89) ) ( n <100) ( )
= f f ( f (102) ) ( n <100)

( )
= f f ( f ( 94 ) ) ( n <100) = f ( f ( 99) )

((
= f f f ( f ( 99) ) ) ) ( n <100) (
= f f ( f (104 ) ) )
= f ( f ( f ( f ( f (104 ) ) ) ) ) = f ( f (101) )
= f ( 98 )
= f ( f ( f ( f (101) ) ) )
= f ( f (103) )
(
= f f ( f ( 98 ) ) ) = f (100)
((
= f f f ( f (103) ) )) = 97
So,the unit's place of f ( 84 ) is 7.
(
= f f ( f (100) ) )
= f ( f ( 97 ) )

© 2021, BYJU'S. All rights reserved


05

For f(x), if f(f(x)).(1 + f(x)) = f(x), then find f(3). f (x)


Given : f ( f ( x ) ) =
1 + f (x)
f (x)
Solution Given : f ( f ( x ) ) = Let f ( x ) = y ⇒ f ( y ) =
y
1 + f (x) 1+y
f (x) y 3 3
Given : f ( f ( x ) ) = Let f ( x ) = y ⇒ f ( y ) = So, f (3) = =
1 + f (x) 1+y 1+3 4
3 3
y So, f (3) = =
Let f ( x ) = y ⇒ f ( y ) = 1+3 4
1+y
3 3 f(x), if f(3)= 1 and f(3x) = x + f(3x - 3), then find f(300).
So, f (For
3) =any function
=
1+3 4

Solution

f(3x) = x + f(3x - 3) For x = 4,


For x = 2, f(12) = 4 + f(12 - 3)= 4 + f(9) = 4 + 3 + 2 + 1 = 10
f(6) = 2 + f(6 - 3)= 2 + f(3) = 2 + 1 = 3 Now, for x = 4,
For x = 3, f(12) = f(3 × 4)= 4 + 3 + 2 + 1 = 10
f(9) = 3 + f(9 - 3)= 3 + f(6) = 3 + 2 + 1 = 6 Similarly, for x = 100,
f(300) = f(3 × 100) = 100 + 99 + ... + 2 + 1 = 5,050

 x + 59 
If 3f ( x ) + 2f   = 10x + 30, then find f (7 ) .
 x−1 

Solution

 66 
For x = 7, 3f (7 ) + 2f   = 10 (7 ) + 30 ⇒ 3f (7 ) + 2f (11) = 100 ... ( i )
 6 
 70 
For x = 11, 3f (11) + 2f   = 10 (11) + 30 ⇒ 3f (11) + 2f (7 ) = 140 ... ( ii)
 10 
By solving ( i ) and ( ii) , we get,
f (7 ) = 4

If f(1) = 2005, f(1) + f(2) + f(3) + ... + f(n) = n2f(n), then find f(2004).

Solution

f (1) + f ( 2) + f (3) + ... + f ( n ) = n2f ( n )


f (1)
For n = 2, f (1) + f ( 2) = 22 f ( 2) ⇒ f (1) = 3f ( 2) ⇒ f ( 2) =
1+2
f ( 2) f (1 )
For n = 3, f (1) + f ( 2) + f (3) = 32 f (3) ⇒ 4f ( 2) = 8f (3) ⇒ f (3) =
© 2021, BYJU'S. All rights reserved =
2 1+2+3
06
f (1) + f ( 2) + f (3) + ... + f ( n ) = n2f ( n )
f (1)
For n = 2, f (1) + f ( 2) = 22 f ( 2) ⇒ f (1) = 3f ( 2) ⇒ f ( 2) =
1+2
f ( 2) f (1 )
For n = 3, f (1) + f ( 2) + f (3) = 32 f (3) ⇒ 4f ( 2) = 8f (3) ⇒ f (3) = =
2 1+2+3
f (1 )
Similarly, for n = 2004, f ( 2004 ) =
1+...+2004
2004 ( 2004 + 1)
1+...+2004 = = 1002 × 2005
2
f (1 ) 2005 1
f ( 2004 ) = = =
1+...+2004 1002 × 2005 1002

f (x)
If for positive x, y and f ( 30 ) = 20, f ( x·y ) = , then find f ( 40 ) .
y

Solution

f (10)
Let f (30) = f (10·3) =
3
f (10)
⇒ 20 = ⇒ f (10) = 60
3
f (10) 60
Now, f ( 40) = f (10·4 ) = = = 15
4 4

Functional Equation Based Questions

• Functional equations are the equations where the unknowns are functions rather than
traditional variables. However, the methods used to solve functional equations can be quite
different from the methods used to isolate a traditional variable.
• Each functional equation provides some information about a function or about multiple
functions.
Example:
f(x) – f(y) = x – y is a functional equation. Here, f is a function. We are given that the difference
between any two output values is equal to the difference between the input values.
f(x) = x satisfies the given functional equation and so does f(x) = x + c for all constants c.

If f(x) + 2f(1 - x) = 3x, then find f(x).

Solution

Given, f(x) + 2f(1 – x) = 3x ... (i) ⇒ f(1 – x) + 2f(x) = 3 – 3 x ... (ii)


By replacing x by 1 – x, we get, By solving equations (i) and (ii), we get,
f(1 – x) + 2f(1 – (1 – x)) = 3(1 – x) f(x) = 2 – 3x

© 2021, BYJU'S. All rights reserved


07

If f(sin x) + 3f(cos x) = tan2x, then find f(x).

Solution

Given, f(sin x) + 3f(cos x) = tan2x ... (i) On replacing sin x by t, we get,


π
By replacing x by – x, we get,
2 8f ( t ) =
3 1− t2 (

)
t2
f(cos x) + 3f(sin x) = cot2 x ... (ii) t2 1− t2
On multiplying equation (ii) by 3, and solving Similarly, on replacing t by x, we get
equations (i) and (ii), we get,
8f(sin x) = 3cot2x – tan2x f (x) =
(
3 1 − x2 ) −
x2
3cos2x sin2x
8x2 (
8 1 − x2 )
⇒ 8f ( sin x ) = −
sin2x cos2x

100
If 2f ( x·y ) = ( f ( x ) ) + ( f ( y ) ) ; f (1 ) = 2, then find ∑ f ( r ) .
y x

r =1

Solution

Let y = 1
⇒ 2f ( x·1) = ( f ( x ) ) + ( f (1) ) ⇒ 2f ( x ) = f ( x ) + ( 2) ⇒ f ( x ) = 2x
1 x x

∴ f ( r ) = 2r
100 100
(
2 2100 − 1 ) =2 2
Now , ∑ f (r) ∑2 r 1 2 99
= 2 + 2 + ... + 2 + 2 100
=
2−1
( 100
−1 )
r=1 r=1

If f(x + y) = f(x) + y(y + 2x + 1) and f(5) = 32, then find f(x).

Solution

Given, f(x + y) = f(x) + y(y + 2x + 1) By replacing y + 5 =x , i.e., y = x – 5, we get,


By putting x = 5, we get, f(x) = 32 + (x – 5)2 + 11(x – 5)
f(5 + y) = f(5) + y(y + 10 + 1) ⇒ f(x) = x2 + x + 2
⇒ f(5 + y) = 32 + y2 + 11y

Concept Check

1. If a polynomial function f(x) when divided by x, (x – 1), and (x + 1) leaves a remainder of 2, 4,


and 5, respectively, then find the remainder when f(x) is divided by x3 – x.

© 2021, BYJU'S. All rights reserved


08

2. If a polynomial f(x) = x135 + x125 – x115 + x5 + 1 is divided by x3 – x leaves a remainder of g(x), then
find g(3).
1 1
3. If f(x) is a polynomial satisfying f ( x ) ·f   = f ( x ) + f   and f(3)= –8, then find f(4).
x x
1 1
4. If f(x) is a polynomial satisfying f ( x ) ·f   − 2f ( x ) − 2f   = 5 and f(2)= 14, then find f(3).
x x
b( x − a ) a ( x − b)
5. If f ( x ) = + , then find f ( a ) , f ( b ) and f ( a + b ) .
b− a a− b
x
f ( x·y ) + f  
x y
6. If f(x) = cos (log x), then find f ( x·y ) , f   and
y f (x) · f (y )
 1  x
 ,x≥4   3 
7. If f ( x ) =  2  , then find f  3 + log 2    .
f x + 1 , x < 4   2 
 ( )
 2002 
8. If f ( x ) + 2f   = 3x, then find f ( 2) .
 x 
 1− x 
9. If f ( x − 1) + 4f   = 3x, then find f ( x ) .
 x 
10. If f(x + y) = f(xy + 2), f(1) = 5, then find f(x).

Summary Sheet

Key Takeaways

1 1
• For a polynomial function f(x), If f ( x ) ·f   = f ( x ) + f   , then f ( x ) = 1 ± x n
x x
• Functional equations are the equations where the unknowns are functions rather than traditional
variables. However, the methods used to solve functional equations can be quite different from
the methods to isolate a traditional variable.

Mind Map

Functions

Value of Polynomial based Functional


function questions equation

© 2021, BYJU'S. All rights reserved


09

Self-Assessment

If f(f(x))(1 – f(x)) = (f(x))2, then find f(–5).

Answers

Concept Check

1. By
 using the remainder theorem, we get, f(1) = 0 × Q(x) + a(1) + b(1) + c
f(0) = 2, f(1) = 4, and f(-1) = 5 ⇒4=a+b+2
Now, f(x) = Divisor × Q(x) + R(x) ⇒ 2 = a + b . . . (i)
Where, Q(x) = Quotient , R(x) = Remainder For x = –1,
In this case, f(x) = (x3 – x) × Q(x) + R(x) f(–1) = 0 × Q(x) + a(1) + b(–1) + c
Also, when f(x) is divided by x3 – x, the ⇒5=a–b+2
remainder will either be quadratic, linear, or ⇒ 3 = a – b . . . (ii)
constant. By solving equations (i) and (ii), we get,
⇒ f(x) = (x3– x) × Q(x) + ax2 + bx + c 5 −1
a = and b =
For x = 0, 2 2
f(0) = 0 × Q(x) + c
5x2 x
⇒2= c g(x) = − +2
For x = 1, 2 2

2. Since f(x) is divided by x3 – x , i.e, For x = 1,


x(x – 1)(x + 1) the three factors of f(x) are f(1) = 0 × Q(x) + a(1) + b(1) + c
0, 1, and –1. By substituting these values in ⇒3=a+b+1
f(x), we get, f(0) = 1, f(1) = 3, and f(-1) = -1 ⇒ 2 = a + b . . . (i)
Now, f(x) = Divisor × Q(x) + g(x) For x= –1,
Where, Q(x) = Quotient , g(x) = Remainder f(–1) = 0 × Q(x) + a(1) + b(–1) + c
In this case, f(x) = (x3 – x) × Q(x) + g(x) ⇒ –1 = a – b + 1
Also,     remainder of f(x) after dividing by x3 – x ⇒ –2 = a – b . . . (ii)
will be either quadratic or linear or constant. By solving equations (i) and (ii), we get,
⇒ f(x) = (x3 – x) × Q(x) + ax2 + bx + c a = 0 and b = 2
For x = 0, g(x) = 2x + 1
f(0) = 0 × Q(x) + c ⇒g(3) = 2(3) + 1 =7
⇒1= c

3. Let f(x) = 1 – xn ⇒f(x) = 1 – x2


For x = 3, ⇒f(4) = 1 – 42 = –15
f(3) = 1 – 3n ⇒ –8 = 1 – 3n ⇒ n= 2

1 1
4. Given : f ( x ) ·f   − 2f ( x ) − 2f   = 5
x x
Step 1 :
By arranging the given equation, we get,
1 1
f ( x ) · f   − 2f ( x ) − 2f   + 4 = 9
 x reserved
© 2021, BYJU'S. All rights
 x
10

1 1
Given : f ( x ) ·f   − 2f ( x ) − 2f   = 5
x x
Step 1 : Step 2 :
By arranging the given equation, we get, Now, let ( f ( x ) − 2) = g ( x ) and
1 1  1  1
f ( x ) · f   − 2f ( x ) − 2f   + 4 = 9
x x  f   −2  = g  
 x  x
 1   1  1
⇒ f ( x )  f   − 2  − 2 f   − 2  = 9 ∴ g ( x ) ·g   = 9
 x   x  x
 1 
⇒ ( f ( x ) − 2)  f   − 2  = 9
 x 
Step 3 :
Now, the function ±3x n satisfies this condition.
So , g ( x ) = ±3x n
⇒ f ( x ) − 2 = ±3x n
⇒ f ( x ) = 2 ± 3x n . Also, given that f ( 2) = 14 > 0
f ( 2) = 2 + 3 × 2n ⇒ 14 = 2 + 3 × 2n ⇒ 12 = 3 × 2n ⇒ 4 = 2n ⇒ n = 2
∴ f ( x ) = 2 + 3x2
⇒ f (3) = 2 + 3·32 = 29

5. By rearranging f ( x ) , we get,
b( x − a ) a ( x − b) bx − ab − ax + ab
f (x) = − ⇒ f (x) =
b− a b− a b− a

⇒ f (x) =
(b− a) x ⇒ f x = x
( )
b− a
∴ f ( a ) = a, f ( b ) = b, and f ( a + b ) = a + b

6. Given, f ( x ) = cos ( log x )


f ( y ) = cos ( log y )
f ( x·y ) = cos ( log ( x·y ) ) = cos ( log x + log y ) = cos ( log x ) cos ( log y ) − sin ( log x ) sin ( log y )
x  x
f   = cos  log  = cos ( log x − log y ) = cos ( log x ) cos ( log y ) +sin ( log x ) sin ( log y )
y  y
x
f ( x·y ) + f  
⇒  y  = 2cos ( log x ) ·cos ( log y ) = 2
f ( x )·f ( y ) cos ( log
Step 1 : x ) ·cos ( log y )
  3 
f  3 + log 2   
7. Step 1 :   2 
= f (3 + log 2 3 −= log 2 2) f (3 + log=
2 3 − 1 ) f ( 2 + log 2 3 )
  3 
f  3 + log 2   
=  2  ( f= )
log 2 22 + log 2 3 f = ( log2 4 + log2 3 ) f ( log2 12 )
= f (3 + log 2 3 −= 2) f (23: + log=
log 2 Step 2 3 − 1 ) f ( 2 + log 2 3 )
© 2021, BYJU'S. All rights reserved
2 Now, log 2 8 < log 2 12 < log 2 16 i.e 3 < log 2 12 < 4
  3 
11 f  3 + log 2   
  2 
= f (3 + log 2 3 −= log 2 2) f (3 + log=
2 3 − 1 ) f ( 2 + log 2 3 )

f=( )
( log2 4 + log2 3 ) f ( log2 12 )
log 2 22 + log 2 3 f =
Step 2 :
Now, log 2 8 < log 2 12 < log 2 16 i.e 3 < log 2 12 < 4
So, f ( log 2 12 ) = f ( log 2 12 + 1)
= f ( log 2 12 + log 2 2 )
= f ( log 2 24 )

Step 3 :
Now, log 2 16 < log 2 24 < log 2 32 i.e 4 < log 2 24 < 5
log2 24
1 1
( )
log2 24
∴ f ( log 2 24 ) =  
-1
= 2-1 = 2-log2 24 = = 2log2 24 =
2 24

 2002 
8. For x = 2, f ( 2) + 2f   = 3( 2) ⇒ f ( 2) + 2f (1001) = 6 ... ( i)
 2 
 2002 
For x = 1001, f (1001) + 2f   = 3(1001) ⇒ f (1001) + 2f ( 2) = 3003 ... ( ii)
 1001 
By solving ( i) and ( ii) , we get,
f ( 2) = 2000

9. Step 1 : Step 2 :
1  Let x − 1 = t i.e., t + 1 = x
Given, f ( x − 1) + 4f  − 1  = 3x ... ( i )
x  12
15 f ( t ) = − 3( t + 1)
1 t+1
By replacing x by , we get,
x By replacing t by x , we get
1  3
f  − 1  + 4f ( x − 1) = ... ( ii ) f (x) =
4

( x + 1)
x  x 5( x + 1) 5
Multiplying ( ii) by 4 and solving ( i) and ( ii) ,
12
15 f ( x − 1) = − 3x
x

10. Given, f(x + y) = f(xy + 2) f(x) = f(1) = 5 = k


By substituting y = 0, we get, For x = 2,
f(x + 0) = f(0x + 2) ⇒ f(x) = f(2) = k f(x) = f(2) = k = 5
Now, for x = 1, ⇒f(x) = 5

( f ( x ))
2

Self-Assessment Given, f ( f ( x ) ) =
1− f (x)

( f ( x )) ( −5) = 25
2 2

Given, f ( f ( x ) ) = So, f ( −5) =


1− f (x) 1 − ( −5) 6

( −5) = 25
2

So, f ( −5) =
1 − ( −5) 6
© 2021, BYJU'S. All rights reserved
NOTE
M A T H E M A T I C S

RELATIONS AND FUNCTIONS


GREATEST INTEGER FUNCTION AND
FRACTIONAL PART FUNCTION

What you already know

• Domain and range of a function • Value of a function


• Types of functions • Functional equations

What you will learn

• Greatest integer function • Fractional part function

4x 2001
 r 
If f ( x ) = x
then, find ∑ f  .
4 +2 r =1  2002 

Solution

Step 1:
4x 41− x
f ( x ) + f (1 − x ) = x + 1− x
4 +2 4 +2
4
4x x 4x 4 4x + 2
= x + 4 = x + = =1
4 +2 4
+2 4 + 2 2 2 + 4 x
(
4 x
+ 2 )
4x

Step 2:
2001
 r   1   2   1001   2000   2001 
∑ f  =f  +f  + ... + f   + ... + f  +f 
r =1  2002   2002   2002   2002   2002   2002 
 1   2   1001   2   1 
=f  +f   + ...+ f   + ...+ f  1 −  + f 1 −
 2002   2002   2002   2002   2002 
 1001 
= 1 + 1 + 1 + ... + f  
 2002 
1
= 1000 + f  
2

© 2021, BYJU'S. All rights reserved


02

Step 3:
1

1 42 2
1000 + f   = 1000 + 1
= 1000 + = 1000.5
2 2+2
4 +2
2

Note

ax 2001
 r  2001
In case of f ( x ) = , ∑ f  is half of the upper limit, i.e.,
a + a r=1  2002 
x
2

Greatest Integer Function (G.I.F)

The expression y = [x] is the greatest integer Y


function, which is less than or equal to x.
This is also referred to as a step function.
3
Domain: ℝ
Range: y ∈ ℤ
2

–3 –2 –1 1 2 3 X
Examples: –1
[2.3]= 2 (The greatest integer less than or
equal to 2.3 is 2) –2
[7.99]=7 (The greatest integer less than or
equal to 7.99 is 7) –3
[–3.7]= –4 (The greatest integer less than
or equal to –3.7 is –4)

Properties
y=x y=x–1
Y
• x – 1 < [x] ≤ x
Example: 2
Let x = 2.3
2.3 –1 < [2.3] ≤ 2.3 1

• [x+m] = [x] + m, for m ∈ ℤ


Example: –2 –1 1 2 3 X
[2.3+1] = [2.3] + 1= 2+1 = 3 –1
0, x ∈
• [ x ] + [-x ] = −1, x ∉  –2

© 2021, BYJU'S. All rights reserved


03

Example:
[7] + [–7] = 7 – 7 = 0
[7.1] + [–7.1] = 7 – 8 = –1
• [x] = m, ⇒ x ∈ [m, m + 1)
Example:
[x] = –1
⇒ x ∈ [–1, 0)
1 2 n− 1
• [ x ] +  x + n  +  x + n  + ... +  x + n 
= [ nx ]
    
 x   x + 1
•   +   = [x]
2  2 
• If [x] ≤ n, then x ∈ (–∞, n + 1)
Example:
1.  If [x] ≤ 2, then find x.
x is an integer, which is less than or equal to 2.
[x] ≤ 2 ⇒ x ∈ (– ∞, 3) –3 –2 –1 0 1 2 3

2. If –2 ≤ [x] ≤ 6, find x.
x is an integer lying between –2 and 6.
–2 ≤ [x] ≤ 6 ⇒ x ∈ [–2, 7) –2 –1 0 1 2 3 4 5 6 7

3. If [x] > 2, find x.


x is an integer, which is greater than 2.
[x] > 2 ⇒ x ≥ 3 3 4 5 6 7

4. If [x] < 3, find x.


x is an integer, which is less than 3. –2 –1 0 1 2 3
[x] < 3 ⇒ x < 3

1  1 1  1 2   1 99 
Find value of the following :   +  +  + +  + ... +  + 
 4   4 100   4 100   4 100 

Solution

 1  2  n -1 
By comparing with [ x ] +  x +  +  x +  + ... +  x + = [ nx ] , we get
 n  n  n 
1
x = and n = 100
4
1  1 1  1 2   1 99   1
 4  +  4 + 100  +  4 + 100  + ... +  4 + 100  = 100 × 4  = [25] = 25
         

© 2021, BYJU'S. All rights reserved


04

5 1  5 2   5 199 
Find value of the following :  +  + +  + ... +  + 
 3 200   3 200   3 200 

Solution

5 
By adding and subtracting   , we get
3 
5 5 1  5 2   5 199   5 
 3  +  3 + 200  +  3 + 200  + ... +  3 + 200  −  3 
         
5  5   1  2  n − 1 
=  × 200 −    [ x ] +  x +  +  x +  + ... +  x +  = [ nx ] 
3  3   n  n  n  
 1000   5 
=  −   = [333.33] − [1.66] = 333 − 1 = 332
 3  3 

 x + 8   x +7   x +6   x +5   x + 4   x +3   x +2   x +1 
If  + + + + + + +  = 24 , find x.
 2   2   2   2   2   2   2   2 

Solution

Step 1 :
 x   x + 1
We know that   +   = [x]
2  2 
 ( x +7 ) +1   x +7   ( x +5) +1   x +5   ( x +3) +1   x +3 
⇒  +  +  +  +   +  
 2   2   2   2   2   2 
 ( x +1 ) +1   x +1 
+  + 
 2   2 
= [ x +7] + [ x +5] + [ x +3] + [ x +1]
Step 2 :
[ x +7] + [ x +5] + [ x +3] + [ x +1] = 24
⇒ [ x ] +7+ [ x ] +5+ [ x ] +3+ [ x ] +1 = 24
⇒ 4[ x ] = 8 ⇒ [ x ] = 2 ⇒ x = [2, 3)

© 2021, BYJU'S. All rights reserved


05

4 [ x + 2] 1
= ; then find [ x ].
[ x +[ x + [ x ]]] − 12 3

Solution

4[ x + 2] 4[ x ] + 8 4[ x ] + 8 4[ x ] + 8
= = =
[ x + [ x + [ x ]]] − 12  x +[ x ] + [ x ]  − 12 [ x ] +[ x ] + [ x ] − 12 3[ x ] − 12
4[ x ] + 8 1
⇒ =
3[ x ] − 12 3
⇒ 12[ x ] + 24 = 3[ x ] − 12 ⇒ 9[ x ] = −36 ⇒ [ x ] = − 4 ⇒ x ∈ [ − 4, −3)

x  x   x  4x
If 0 < x < 300 and   +   +   = . The sum of possible values of x is m,
 2   5   10  5
 m 
then find  .
 1000 

Solution

Step 1 :
We know that [I] = I
 x   x   x  5x + 2x + x 4x
 2  +  5  +  10  = = , (L.C.M ( 2, 5, 10) = 10) is only possible,
      10 5
x x x
when all   ,   and   are integers.
2 5  10 
Now, they will be integers for all multiples of 10.
∴ x = 10, 20, 30, ..., 290 ( 0 < x < 300)
Step 2 :
10×29
∑ x =10 + 20 + ... + 290 = 10(1 + 2+... + 29) = 2
(1+ 29) = 4350
 m   4350 
Now  =  = [ 4.35] = 4
 1000   1000 

© 2021, BYJU'S. All rights reserved


06

 π
Find the range of y = [sin x + [ cos x + [ tan x + [ sec x  ]]  ; x ∈  0, 
 4

Solution

y = [sin x ] + [cos x + [ tan x + [sec x ]]]


⇒ y = [sin x ] + [cos x ] + [tan x + [sec x ] ]
⇒ y = [sin x ] + [cos x ] + [ tan x ] + [sec x ]
 π  1
Now for x ∈  0,  , sin x ∈  0,  i.e., ( 0, 0.707 ) ⇒ [sin x ] = 0
 4  2
 π  1 
for x ∈  0,  , cos x ∈  , 1  i.e., ( 0.707, 1) ⇒ [cos x ] = 0
 4  2 
 π
for x ∈  0,  , tanx ∈ ( 0, 1) i.e., ( 0, 1) ⇒ [ tanx ] = 0
 4
 π
( )
for x ∈  0,  , sec x ∈ 1, 2 i.e., (1, 1.41) ⇒ [sec x ] = 1
 4
∴ [sin x ] + [cos x ] + [ tan x ] + [sec x ] = 0 + 0 + 0 + 1 = 1

Example:
Draw a graph of Y Y
the following:
y = 1 + [x]
1. Make a graph of
y = [x] –1 01 X 01 X
–1
2. Shift it by 1 unit.

y = [x] y = 1 + [x]

Example:
Draw a graph of Y
the following: 4
x ∈ [0, 1) y=x+0 3
y = x + [x]
2
x ∈ [1, 2) y=x+1
1
x ∈ [2, 3) y=x+2 0 1 2 3 X
–1
–1
x ∈ [–1, 0) y=x–1 –2

y = x + [x]

© 2021, BYJU'S. All rights reserved


07

Example: Y
Draw a graph of
the following: 4
y = x[x] x ∈ [0, 1) y = x0 = 1 3
2
x ∈ [1, 2) y = x1 = x 1

x ∈ [2, 3) y = x2 –2 –1 0 1 2 3 X
–1
–2
x ∈ [–1, 0) y = x–1
–3

y = x[x]

Find the range of the following: y = [√x ]; 0 ≤ x ≤ 4

Solution

Y
x ∈ [0, 1)
y = x1 = 0
√x ∈ [0, 1)
2
x ∈ [1, 4)
y = [√x ] = 1 1
√x ∈ [0, 2)
x=4 01 2 3 4 X
y = [√x ]= 2
√x = 2
Domain: [0, 4]
Range:{0, 1, 2}
y = [√x ]

Find the domain of f(x) = √1 – [x]2 where [.] denotes G.I.F.


(a) (1, 2) (b) [―1, 2) (c) [1, 2] (d) (―1, 0)

Solution

For f(x) = √ 1 ― [x]2 to exist,


1 – [x]2 ≥ 0 ⇒ [x]2 ― 1 ≤ 0
⇒([x] ― 1)([x] + 1) ≤ 0
⇒ ―1 ≤ [x] ≤ 1 ⇒ x ∈ [―1, 2)
Therefore, option (b) is the correct answer.

© 2021, BYJU'S. All rights reserved


08

Fractional Part Function

Any real number x, can be written as x = [x] + {x}, where [.] represents the greatest integer function,
and {.} represents the fractional part function.
{x} = x ― [x]

Graph of fractional part function


We know that y = {x} = x ― [x] Y
⇒ 0 ≤ {x} < 1
1

1
Domain: x ∈ ℝ

x
x–
x+

=
=
y
Range: y ∈ [0, 1)

y
y
―2 ―1 0 1 2 3 X
Example:
{3.92} = 0.92
{5} = 0
{―6} = 0
{―16.21} = ― 16.21 + 17 = 0.79 ∵ ({x} = x ― [x])

Properties

• 0 ≤ {x} < 1 Example: 2. Find the domain of the


• {x} = 0 if x ∈ ℤ 1. Find the domain of the following:
• {x} > 0 if x ∉ ℤ or x ∈ ℝ – {ℤ} following: y = ln {x}
• {x + n} = {x}, n ∈ ℤ y = √ x ― [x] ⇒ {x} > 0 ⇒ x ≠ 𝕀
• x= [x] + {x} ⇒ {x} = x – [x] ⇒ x―[x] ≥ 0 ⇒ {x} ≥ 0 ⇒ x ∈ ℝ ⇒ x ∈ ℝ – {ℤ}
• {x} + {–x} = 0, if x ∈ ℤ
{x} + {–x} =1, if x ∉ ℤ
⇒ x ∈ ℝ – {ℤ}
Example:
Find the domain of the following:
y = √{–x}

Case 1: Let us assume that x ∈ ℤ Case 2: Let us assume that x ∉ ℤ


{–x} ≥ 0 ⇒ –{x} ≥ 0 ⇒ {x} ≤ 0 {–x} ≥ 0 ⇒ 1 –{x} ≥ 0 ⇒ {x} ≤ 1
It is possible only when {x} = 0 {x} < 1 is always true

By the intersection of both the cases, x ∈ ℝ.

100

∑{x + r}
If y = [ x ] + r=1
then, find ∫ y dx.
100

Solution

Step 1 :

y = [x] +
{x + 1} + {x + 2} + ... + {x + 100}
100

⇒ y = [x] +
{x} + {x} + ... + {x} = [x] +
100{x}
= [ x ] + {x} = x
© 2021, BYJU'S. All rights reserved
100 100
09
Step 1 :

y = [x] +
{x + 1} + {x + 2} + ... + {x + 100}
100

⇒ y = [x] +
{x} + {x} + ... + {x} = [x] +
100{x}
= [ x ] + {x} = x
Step 2 :
100 100 x2
∫ y·dx = ∫ x·dx = 2
+c

Solve for x; 2x + 3{x} = 4 [ x ] − 2

Solution

Step 1 :
Step 1 :
2[ x ] − 2
2([ x ] + {x} ) + 3{x} = 4[ x ] − 2 ⇒ {x} = 2[ x ] − 2
2([ x ] + {x} ) + 3{x} = 4[ x ] − 2 ⇒ {x} = 5
Step 2 : 5
Step 2 :
2[ x ] − 2 7
0 ≤ 2[ x ] − 2 < 1 ⇒ 0 ≤ 2[ x ] − 2 < 5 ⇒ 1 ≤ [ x ] < 7
0≤ 5 < 1 ⇒ 0 ≤ 2[ x ] − 2 < 5 ⇒ 1 ≤ [ x ] < 2
5 2

Step 3 :
7
For 1 ≤ [ x ] < , the possible cases are as follows,
2
2×1 − 2
[ x ] = 1, {x} = 5 = 0 ⇒ x = [ x ] + {x} = 1 + 0 = 1
2×2 − 2 2 2 12
[ x ] = 2, {x} = 5 = 5 ⇒ x = [ x ] + {x} = 2 + 5 = 5
2×3 − 2 4 4 19
[ x ] = 3, {x} = 5 = 5 ⇒ x = [ x ] + {x} = 3 + 5 = 5
 12 19 
x ∈ 1, , 
 5 5

Find the range of the following: y = x – [x – 2]

Solution Y

y = x – [x] + 2 ⇒ y = {x} + 2
{x} ∈ [0, 1) ⇒ y = {x} + 2 ∈ [2, 3) y=3
y=2
X

© 2021, BYJU'S. All rights reserved


10

−1, x < 0

If g ( x ) = 1 + x − [ x ] and f ( x ) =  0, x = 0 , then find f ( g ( x ) ) .
 1, x > 0

Solution

g ( x ) = 1 + {x} ⇒ g ( x ) ∈ 1 + [0, 1)
g ( x ) ∈ [1, 2)
∴ f ( g ( x ) ) = 1 as g ( x ) ∈ [1, 2) > 0

Concept Check

I
 2f ( x ) 
1. If x = log 4  , then find f ( 2010) + f ( −2009) .
 1 − f ( x ) 
 

2. If [ x ] + x + 1   2   99 
  + x +  + ... +  x + = 7 , then find x.
 100   100   100 
3. If  x + 6  +  x + 7  + 2[ x ] = 18, then find [ x ] .
 2   2 
   
4. If [x] + x = a has a solution, where a ∈ ℕ , a ≤ 20 and x > 0, then find the possible values of a.
2

5. Find the range of the following:


y = (x + 5) – [x – 3]

Summary Sheet

Key Takeaways

• y = [x] is the greatest integer function, which is less than or equal to x. This is also referred to
as a step function.
Domain: ℝ
Range: y ∈ ℤ
• Any real number x, can be written as x = [x] + {x}, where [.] represents the greatest integer
function, and {.} represents the fractional part function.
{x} = x – [x]
Domain: x ∈ ℝ
Range: y ∈ [0, 1)

© 2021, BYJU'S. All rights reserved


11

Mind Map

Greatest integer
function (G.I.F)

Functions Types of function

Fractional part
function (F.P.F)

Self-Assessment

Find the domain of the following function:


f(x) = √ [x] – 1 + √ 4 – [x]

Answers

Concept Check

2f ( x ) 4x
1.  Given, 4x = ⇒ f (x) = x
1 − f (x) 4 +2
We know that for such functions , f ( x ) + f (1 − x ) = 1
By substituting x = 2010 , we get ,
f ( 2010) + f (1 − 2010) = f ( 2010) + f ( − 2009) = 1

 1  2  n −1
2. By comparing with [ x ] +  x +  +  x +  + ... +  x + = [ nx ] , we get,
 n  n  n 
7 8
[nx ] = [100x ] = 7 ⇒ 100x = [7, 8 ) ⇒ x =  100 , 100 
 

x + 6 x + 7
3. Given,  +  + 2[ x ] = 18
 2   2 
x  x + 1
We know that   +   = [x]
2  2 
 x + 6  (x + 6)+ 1
⇒  +  + 2[ x ] = 18
 2   2 
⇒ [ x + 6] + 2 [ x ] = 18 ⇒ [ x ] + 6 + 2[ x ] =18 ⇒ 3[ x ] = 12 ⇒ [ x ] = 4
∴ x ∈ [ 4, 5)

© 2021, BYJU'S. All rights reserved


12

4. Step 1 : Step 2 :
[x]
2
+ x = a, Now , a = x2 + x = x ( x + 1)

Since [ x ] and a are integers,


2 For x = 1, a = 1 (1 + 1) = 2

x has to be an integer. For x = 2, a = 2( 2 + 1) = 6

∴ [ x ] = x and [ x ] = x2
2 For x = 3, a = 3(3 + 1) = 12
For x = 4, a = 4 ( 4 + 1) = 20
∴ Possible values of a = {2, 6, 12, 20} ( 1 ≤ a ≤ 20 )

5. y = x + 5 – [x] + 3
⇒ y = {x} + 8
{x} ∈ [0, 1) ⇒ {x} + 8 ∈ [8, 9)

Self-Assessment

f(x) = √ [x] – 1 + √ 4 – [x] is defined only in the following conditions:


[x] – 1 ≥ 0 ⇒ x ∈ [1, ∞)
4 – [x] ≥ 0 ⇒ x ∈ (–∞, 5)
⇒ f(x) ∈ [1, 5)

© 2021, BYJU'S. All rights reserved


NOTE
M A T H E M A T I C S

RELATIONS AND FUNCTIONS


TRANSFORMATIONS OF GRAPHS

What you already know

• Domain and range of a function • Greatest integer function


• Types of functions • Fractional part function
• Value of a function

What you will learn

• Transformation of graphs

 π 
Find the domain of f(x), where f(x) = [sinx] . cos    and [.] denotes the G.I.F of x.
 [ x + 1] 
 

Solution

Step 1:
The domain of [sin x] is x ∈ ℝ
 π 
For cos  , [x + 1] ≠ 0
 [ x + 1] 
 
⇒ [x] + 1 ≠ 0 (∵ [x + n] = [x] + n , n ∈ ℤ)
⇒ [x] ≠ -1
⇒ x ∉ [-1, 0)
The domain of f(x) is x ∈ ℝ - [-1, 0)

x−2
Find the domain of f(x), where f ( x ) = 22x + 64 3
− 2−1 ( 72 + 22x )

Solution

Step 1:
x−2
f ( x ) = 2 + 642x 3
(
− 2−1 72 + 22x )
x−2 4x
= 4 +4x
© 2021, BYJU'S. All rights reserved − 36 −
2
02

x−2
f ( x ) = 22x + 64 3
(
− 2−1 72 + 22x )
4x
= 4x + 4x − 2 − 36 −
2
4x 4x
Now, 4x + − 36 − ≥0
16 2

Step 2:
Let 4x = y
y y
⇒y+ − 36 − ≥ 0
16 2
⇒ y ≥ 64, 4 ≥ 64
x

⇒ x ≥3
The domain of f(x) is x ∈ [3, ∞)

Find the domain and range of y = 7− x C x − 3

Solution

Step 1:
We know that for nCr to be defined n > 0, r ≥ 0, n ≥ r, n and r ∈ ℤ
7 - x > 0 ⇒ x < 7 ... (i)
x - 3 ≥ 0 ⇒ x ≥ 3 ... (ii)
7 - x ≥ x - 3 ⇒ x ≤ 5 ... (iii)

Step 2:
After taking the intersection of (i), (ii), (iii), we get x ∈ [3, 5]
However, x should be an integer. So, x ∈ {3, 4, 5}
Domain of y : x ∈ {3, 4, 5}

Step 3:
When x = 3, 7 − xCx − 3 = 4C0 = 1
When x = 4, 7 − xCx − 3 = 3C1 =3
When x = 5, 7 − xCx − 3 = 2C2 = 1
Range of y : {1, 3}

Find the domain of 2x + 2y = 2.

Solution

Step 1:
2y= 2 − 2x
Taking log 2 on both sides, we get,
© 2021, BYJU'S. All rights reserved
y x
03

2y= 2 − 2x
Taking log 2 on both sides, we get,
log=
2 2
y
( )
log 2 2 − 2x ( )
y log 2 2 − 2x
⇒= ( )
Step 2:
For the logarithm function to be defined, 2 - 2x > 0
2x < 2 ⇒ x <1
Domain: x ∈ (-∞, 1)

Find the domain of


= y log ( px 3 + ( p + q ) x 2 + ( q + r ) x + r ) , if q2 − =
4pr 0 and p > 0

Solution

Step 1:
= (
y log px3 + ( p + q ) x 2 + ( q + r ) x + r )
=y log ( px ( x + 1 ) + qx ( x + 1 ) + r ( x + 1 ) )
2

=
y log ( x + 1 ) ( px + qx + r ) 
2

For logarithm function to be defined,


( x + 1) ( px2 + qx + r ) > 0

Step 2:
It is given that q2 − 4pr = 0 and p > 0
⇒ px2 + qx + r is a perfect square, which is greater than or equal to zero.
-q
Also, q2 - 4pr = 0 ⇒ Roots of px2 + qx + r are equal and sum of roots is .
2
p
 q 
⇒ px 2 + qx + r= p  x + 
 2p 
2
 q 
⇒ p ( x + 1)  x +  >0
 2p 
Using wavy curve method, we get,
q
x ∈ ( −1, ∞ ) −  
 2p 

© 2021, BYJU'S. All rights reserved


04

If f(x) is defined for (0, 1), then find the domain of f(ex) + f(ln |x|).

Solution Y
4
Step 1:
2
0 < ex < 1 (0, 1)
⇒ x ∈ (-∞, 0) ... (i)
X
-4 -2 0 2 4

-2

Step 2:
0 < ln|x| < 1
⇒ 1 < |x| < e
⇒ x ∈ (-e, -1) ⋃ (1, e) ... (ii)
Taking the intersection of (i) and (ii), we get,
x ∈ (-e, -1)

If the domain of f(x) is (-∞, 0], then find the domain of f(6{x}2 - 5{x} + 1).

Solution

-∞ < 6{x}2 − 5{x} + 1 ≤ 0


Let {x} = t
⇒ 6t 2 − 5t + 1 ≤ 0
⇒ ( 2t − 1)(3t − 1) ≤ 0
1 1
⇒ ≤t≤
3 2
1 1
⇒ ≤ {x} ≤
3 2
 1 1
⇒ x ∈ n + , n +  , n ∈ 
 3 2

( x2 + x + 1)

Find the domain of f ( x ) , where f ( x )= 2

Solution

© 2021, BYJU'S. All rights reserved


05

1
The given expression is, f ( x ) =
( )
3
x2 + x + 1
We can see that x2 + x + 1 is always greater than zero as the discriminant is negative and
coefficient of x2 is greater than 0. So, x ∈ ℝ.

x−2 1−x
Find the domain of
= f (x) + .
x+2 1+x

Solution

Step 1:
x−2
0
x+2
x ∈ (-∞, -2) ⋃ [2, ∞) ... (i)

Step 2:
1−x
0
1+x
x −1
⇒ ≤0
x +1
x ∈ (-1, 1] ... (ii)
After taking intersection of (i) and (ii), we get,
x∈Φ

Transformation of Graphs

Basic graphs

Y Y
4 4
y=x y = -x

2 2

X X
-4 -2 0 2 4 -4 -2 0 2 4

-2 -2

© 2021, BYJU'S. All rights reserved


06

Y Y
4
4
y = x3
y = x2 2
2
X
X -2 0 2 4
-4 -2 0 2 4
-2
-2

1
y=
x
Y 1
Y
y = x3 4
4 y= x

2 y = |x|
2
X
X -4 -2 0 2 4
-2 0 2 4 6
-2
-2

1 Y Y
y= 4
x
1 4 1
y=x 3 y=
2
x
y= x 2
1
y=x 3

-2 y= x
X X
-4 -2 0 2 4 2 4 6
0

-2
-2

1. Vertical stretch

Let us consider a function y = f(x)


When the points on the Y-axis of the graph of the given function f(x) are multiplied by k units, it
gives the graph of y = k f(x)

© 2021, BYJU'S. All rights reserved


07

Y
y = x2
y = 2x2
x2
y=
2
Example: x
y=
Let us plot the graph of y = x2, 2x2 and x on
2
3
the same coordinate plane. 2
3π π 3π
− −
2 2 2
1
=
y (Xx − 2)3
5π 9π 3π

4 4 4

Plot the graph of the function y = 3ex

Solution Y y = 3ex

Step 1:
Plot the graph of the function y = ex
3 y = ex
Step 2:
2
Multiply the points on the Y-axis of the
graph by 3 units to get the graph of y = 3ex 1
X

Plot the graph of the function y = 3 sin x.

Solution Y y = 3 sin x
3

y = sin x
1

_
4π _
3π _
2π _
π 0 π 2π 3π 4π 5π
X
-1

-3

Step 1:
Plot the graph of the function y = sin x
Step 2:
Multiply the points on the Y-axis of the graph by 3 units to get the graph of y = 3 sin x

© 2021, BYJU'S. All rights reserved


08

|x|
Plot the graph of the function y = 3 |x| and .
3

Solution
Y
Step 1: y = 3|x|
y = |x|
Plot the graph of the function y = |x|
x2
y=
Step 2: 2
(a) Multiply the points on the Y-axis of x
y=
the graph by 3 units to get the graph 3
of y = 3|x|. 3π X π 3π
0 − −
(b) Divide the points on the Y-axis of the 2 2 2
graph by 3 units to get the graph of 1

|x| =
y ( x − 2) 3
y= .
3 5π 9π 3π

4 4 4

2. Vertical shift
Y y = 2x + 5
Let us consider a function y = f(x) y = 2x
When the given graph of f(x) is shifted
upwards by a units, it gives the graph of the (0, 5) y = 2x - 3
function y = f(x) + a; a > 0
When the given graph of f(x) is shifted
downwards by a units, it gives the graph of X
the function y = f(x) - a; a > 0 (0, 0)

Example:
Let us plot the graph of y = 2x, 2x - 3, and (0, -3)
2x + 5 on the same coordinate plane.

Plot the graph of the function y = |x| + 3 and y = |x| - 2.

Solution Y y = |x| + 3

Step 1: y = |x|
Plot the graph of the function y = |x|
Step 2: (0, 3) y = |x| - 2

Shift the graph of y = |x| by 2 units


downwards to plot the graph of y = |x| - 2 X

Shift the graph of y = |x| by 3 units upwards


to plot the graph of y = |x| + 3 (0, -2)

© 2021, BYJU'S. All rights reserved


09

Plot the graph of the function y = 1 - |x|.

Solution Y

Step 1: (0, 1)
Draw the graph of y = -|x|
X
Step 2:
Shift the graph of y = -|x| by 1 unit y = 1 - |x|
upwards to plot the graph of y = -|x| + 1
y = -|x|

Plot the graph of the function y = {x} + 3, where { } denotes the fractional part function.

Solution Y

y = {x} + 3
Step 1: 4
Draw the graph of y = {x}
y=3
Step 2: y = {x}
Shift the graph of y = {x} by 3 units X
upwards to plot the graph of y = {x} + 3

Plot the graph of the function y = x + 5

Solution Y
y = √x + 5

Step 1: y = √x
Draw the graph of y = √x
(0, 5)
Step 2:
X
Shift the graph of y = √x  by 5 units
upwards to plot the graph of y = √x + 5

© 2021, BYJU'S. All rights reserved


10

3. y = f(x) → y = -f(x)
The graph of y = -f(x) is obtained by taking the mirror image of y = f(x) about the X-axis.
Y

y = ex

Example:
1
The graph of y = -ex is obtained by taking X
the mirror image of y = ex about the X-axis. 0
-1

y = -ex

Plot the graph of the function y = -cos x

Solution Y

x2 x 2
x2 x2
y= y= y =y = cos x y=
Step 1: 2 2 1 2 2
x x x x
Draw the graph of y = cos x y= y= y= y=
3 3 3 3
X
Step 2:
3π π3π 3ππ

π
− 3π π 3π π 3π 3ππ π



π
2 2 2 −2 − − −2 2 2 2
2 2 2 2 2 2 2 2
The graph of y = -cos x is obtained by taking 1
-1 1
=
y ( x − 2) 3 1 1 =
y ( x − 2) 3
the mirror image of y = cos x about the X-axis. =
y ( x − 2) 3 y y(=
= x− 2) x
3
-cos
5π 9π 3π 5π 9π 3π
π −9π 5π 9π 3π −
5
4 4 4 − 3π − 4 4 4
4 4 4 4 4 4

Plot the graph of the function y = -{x}, where {.} denotes the fractional part function.

Solution Y

Step 1: 1
y = {x}
Draw the graph of y = {x}

Step 2: X

The graph of y = -{x} is obtained by taking


the mirror image of y = {x} on the X-axis. y = -{x}
-1

© 2021, BYJU'S. All rights reserved


11

4. y = f(x) → y = f(-x)
The graph of y = f(-x) is obtained by taking the mirror image of y = f(x) about the Y-axis.

1
y = {-x} y = {x}
Example:
The graph of y = {-x} is obtained by taking
the mirror image of y = {x} about the Y-axis, X
where {.} denotes the fractional part function.

Plot the graph of the function y = e-x.

Solution
y = e−x Y y = ex

Step 1:
Draw the graph of y = ex
1
Step 2:
X
The graph of y = e-x is obtained by taking 0
the mirror image of y = ex on the Y-axis.

Plot the graph of the function y = ln(-x), x < 0

Solution Y
y = ln(-x) y = ln(x)

Step 1:
Draw the graph of y = ln x, x > 0

Step 2:
The graph of y = ln(-x) is obtained by taking
X
the mirror image of y = ln x about the Y-axis. -1 1

© 2021, BYJU'S. All rights reserved


12

Plot the graph of the function y = √-x, x < 0

Solution Y

Step 1: y = √-x y = √x
Draw the graph of y = √x

Step 2:
The graph of y = √-x is obtained by taking X
the mirror image of y = √x on the Y-axis.

5. y = f(x) → y = f(x ± a)
Let us consider a function y = f(x)
a) When the given graph of f(x) is shifted by a units toward the left, it gives the graph of the
function y = f(x + a), (a > 0)
b) When the given graph of f(x) is shifted by a units toward the right, it gives the graph of the
function y = f(x - a), (a > 0)

Plot the graph of the function y = (x - 3)2

Solution Y
y = x2 y = (x - 3)2
Step 1:
Draw the graph of y = x2

Step 2:
Shift the graph of y = x2 in the direction
of the positive X-axis by 3 units to get the X
0 3
graph of y = (x - 3)2

Plot the graph of the function y = ln(x + e)

Solution

Step 1:
Plot the graph of the function y = ln x

© 2021, BYJU'S. All rights reserved


13

Y
x = -e y = ln(x + e)
Step 2:
y = ln x
Shift the graph of y = ln x in the direction
of the negative X-axis by e units to get the
graph of y = ln(x + e)
X
0 (1, 0)

(1 - e, 0)

Plot the graph of the function y = x + 4

Solution Y
y= x+4

Step 1:
Plot the graph of the function y = √x y = √x

Step 2:
Shift the graph of y = √x in the direction
X
of the positive X-axis by 4 units to get the -4 0
graph of y = x + 4

Plot the graph of the function y = |x - 3| + 2

Solution

Step 1: Y
Plot the graph of the function y = |x| y = |x - 3| + 2

Step 2:
Shift the graph of y = |x| in the direction y = |x|
of the positive X-axis by 3 units to get the
graph of y = |x - 3|
(3, 2)
Step 3:
Shift the graph of y = |x - 3| in the upward X
0
direction by 2 units to get the graph of
y = |x - 3| + 2

© 2021, BYJU'S. All rights reserved


14

Plot the graph of the function y = x2 + x + 1

Solution

Step 1:
y = x2 + x + 1 can be written as
2
 1 3 Y
y = x +  +
 2 4 y = x2 + x + 1

Step 2:
Plot the graph of the function y = x2 y = x2

Step 3:
Shift the graph of y = x2 in2the direction of
 1 3  1 3
y = xby+  units
the negative X-axis + to get the − 2, 4
 2 2 4  
 1 3 X
graph of y = x +  + 0 3
 2 4
Step 4: 2
 1 3
Shift the graph of y = x +  in + the upward
2  2 4
 1 3
y=  x + 2  by
direction + units to get the graph
  4
2
 1 3
of y = x +  +
 2 4

1
=
Plot the graph of the function y +3
x+4

Solution
Y

Step 1: 1
1 y=
Plot the graph of the function y = x
x 1
Step 2: 1 y=
y= x+4
1 x+4
Shift the graph of y = in the direction
1 x
y= X
of the negative X-axis
x by14 units to get 0
y= -4
1 x+4 1
the graph of y = 1 y=
x+4 y= x
x 1
1 1 y=
=
Let’s plot y + 3 from y = . x+4
x+4 x+4

© 2021, BYJU'S. All rights reserved


15

1
=y +3
x+4
1 1
= y
To yplot
= + 3 , shift the graph
x x+4 3
1
of y = in the upward direction by
x+4 X
0
3 units. -4
1
y=
x
1
y=
x+4

6. y = f(x) → y = f(|x|)
Let us consider a function y = f(x)
We can obtain the graph of f(|x|) by the following ways:
1. Retaining the graph corresponding to only the non-negative values of x
2. Taking the mirror image of the retained graph about the Y-axis

Plot the graph of the function y = e|x|

Solution
Y
y = ex, x ≥ 0

Step 1:
Plot the graph of y = ex, x ≥ 0
X

y = e|x| Y
Step 2:
Take the mirror image of the obtained graph
about the Y-axis to get the required graph.

© 2021, BYJU'S. All rights reserved


16

Plot the graph of the function y = sin |x|.

Solution Y

y = sin x, x ≥ 0
Step 1:
Plot the graph of y = sin x, x ≥ 0 X

Y
Step 2:
y = sin |x|
Take the mirror image of the obtained
graph about the Y-axis to get the required
graph. X

Plot the graph of the function y = ln |x|.

Solution Y
y = ln x, x > 0

Step 1:
Plot the graph of y = ln x, x > 0
X
1

Y
y = ln |x|
Step 2:
Take the mirror image of the obtained
graph about the Y-axis to get the required
graph.
X
-1 1

© 2021, BYJU'S. All rights reserved


17

Plot the graph of the function y = |x|

Solution
Y

y = √x

Step 1:
Plot the graph of y = √x, x ≥ 0 X

y = |x|
Step 2:
Take the mirror image of the obtained
graph about the Y-axis to get the required
graph. X

Plot the graph of the function y = |x|2 - 5|x| + 6.

Solution Y

x>0
y = x - 5x + 6
2

Step 1:
Plot the graph of y = x2 - 5x + 6, x ≥ 0
2 3

Y
y = |x|2 - 5|x| + 6
Step 2: x>0

Take the mirror image of the obtained


graph about the Y-axis.
2 3

© 2021, BYJU'S. All rights reserved


18

7. y = f(x) → y = |f(x)|
 Let us consider a function y = f(x)
We can obtain the graph of |f(x)| by the following ways:
  1) Retaining the graph that is above the X-axis
  2) Flipping the graph corresponding to the negative values of y about the X-axis

Plot the graph of the function y = |log x|

Solution
Y

Step 1:
X
Plot the graph of y = log x 0 1

Step 2:
Retain the graph that is above the X-axis . y = log x

Step 3:
y = |log x|
Flip the graph corresponding to the
negative values of y about the X-axis X
to get the required graph. 1

1
y
Plot the graph of the function= ( x − 2) 3 .
1

Solution Y
=
y ( x − 2) 3
1
1
=
y ( x − 2) 3 y = x3
Step 1:
1
X
Plot the graph of y = x 3 0

© 2021, BYJU'S. All rights reserved


19 x2
y=
2
x
y=
3
1 Y 3π π 3π π
=
y ( x − 2) 3
2 2

2

2
Step 2: 1
1
Shift the graph of y = x by 2 units on the
3 =
y ( x − 2) 3
positive X-axis to obtain the graph of 0 2 5π 9π X 3π
1

4 4 4
=
y ( x − 2) 3

Y 1

Step 3: =
y ( x − 2) 3
1
Plot the graph of by =
y ( x − 2) 3 retaining 1

1 y = x3
X
the graph of =y ( x − 2) that
1 is above the
3
y = x 3
X-axis and flipping the graph corresponding 0 2
to the negative values of y about the X-axis.

8. y = f(x) → |y| = f(x)


 Let us consider a function y = f(x)
For |y| = f(x),
  1. The region below the X-axis cannot be considered, as |y| gives non-negative values.
  2.  As |y| = f(x) ⇒ y = f(x) and -y = f(x), we also consider the mirror image of the graph obtained
    in step (1) about the X-axis.

Plot the graph of the function |y| = x.

Solution Y
y=x

Step 1: X
Plot the graph of y = x

Y
|y| = x

Step 2:
Retain the region that is present on and
X
above the X-axis, and draw the mirror image
of the retained graph about the X-axis.

© 2021, BYJU'S. All rights reserved


20

Plot the graph of the function |y| = ln x.

Solution Y

Step 1:
X
Plot the graph of y = ln x 0 1

y = ln x

Step 2: |y| = ln x
Retain the region that is present on and
above the X-axis, and draw the mirror image X
of the retained graph about the X-axis. 0 1

Plot the graph of the function |y| = 1 - |x|.

Solution Y

Step 1:
X
Plot the graph of y = 1 - |x| 0

y = 1 - |x|

Step 2:
Retain the region that is present on and
above the X-axis, and draw the mirror image X
of the retained graph about the X-axis. 0

|y| = 1 - |x|

© 2021, BYJU'S. All rights reserved


21

9.  y = f(x) → y = f{x}, where {.} represents fractional part function.


 Let us consider a function y = f(x)
For y = f{x},
  1. Retain the graph of y = f(x) in [0, 1).
  2.  Repeat the retained graph in an interval of 1 unit.

Plot the graph of the function y = e{x}.

Solution Y

e
Step 1: 1
Plot the graph of y = ex.
X
0 1

Y
Step 2:
e
We know the following:
{x} ∈ [0, 1) ⇒ e{x} ∈ [1, e)
1
As {x} is a periodic function with period
1, e{x} is also a periodic function with the X
-2 -1 0 1 2 3
same period.

Concept Check

1. If f(x) is defined in [0, 1], then find the domain of f(tan x).
1 1
2. Find the domain of f(x), where f ( x ) =
[x]
( )
+ log (2{x} − 5) x2 − 3x + 10 +
1− x
x
3. Plot the graph of the function y = 4 x and
2
π
4. Plot the graph of the function y = sin(x - )
4
5. Plot the graph of the function y = |log |x||
6. Plot the graph of the function y = ln{x}

© 2021, BYJU'S. All rights reserved


22

Summary Sheet

Key Takeaways

Let us consider a function y = f(x)


• When the points on the Y-axis of the graph of the given function f(x) are multiplied by k units,
it gives the graph of y = k f(x)
• When the given graph of f(x) is shifted upwards by a units, it gives the graph of the function
y = f(x) + a; a > 0
• When the given graph of f(x) is shifted downwards by a units, it gives the graph of the
function y = f(x) - a; a > 0
• The graph of y = -f(x) is obtained by taking the mirror image of y = f(x) about the X-axis.
• The graph of y = f(-x) is obtained by taking the mirror image of y = f(x) about the Y-axis.
• When the given graph of f(x) is shifted by k units toward the left, it gives the graph of the
function y = f(x + k), (k > 0).
• When the given graph of f(x) is shifted by k units toward the right, it gives the graph of the
function y = f(x - k), (k > 0)
• f(x) → f(|x|) can be obtained by retaining the graph corresponding to the non-negative values
of x and taking the mirror image of it about the Y-axis.
• f(x) → |f(x)| can be obtained by retaining the graph above the X-axis and flip the graph
corresponding to the negative values of y about the X-axis.
• To find the number of solutions for f(x) = g(x), try to plot the graphs of both the functions and
look for the number of points of intersection.

Mind Map

f(x) → -f(x) f(x) → -f(-x)

f(x) → f(-x) f(x) → f(|x|)

f(x) → |f(|x|)|
Transformation
Functions Graphs
of graphs
f(x) → (f{x})

f(x) → f(x) + a f(x) → f(x + a)

f(x) → f(x) - a f(x) → f(x - a)

© 2021, BYJU'S. All rights reserved


23

Self-Assessment

Find the number of solutions for the following pair of equations:


−3 2
=
f(x) x + 3x + 1 and g(x)
= 2x
4

Answers

Concept Check

1.
f(tan x) will be defined when 0 ≤ tan x ≤ 1
π
⇒ tan −1 ( 0) ≤ x ≤ tan −1  
4
 π
⇒ x ∈  nπ , nπ +  , n ∈ 
 4

2.
1 1
f (x) =
[x]
( )
+ log (2{x} − 5) x2 − 3x + 10 +
1− x

( )
log (2{x} − 5) x2 − 3x + 10 to be defined,
2{x} − 5 > 0
⇒ {x} > 2.5
However, {x} ∈ [0, 1), which means that there is no value of x for which the given logarithm is
defined.
So, x ∈ ∅

3.
Step 1:
Y y = 4√x
Plot the graph of the function y = √x

Step 2:
a) Multiply the points on the Y-axis of the
graph by 4 units to get the graph of
y = √x
y = 4√x
x
y=
b) Divide the points on the Y-axis of the 2
graph by 2 units to get the graph of X
O
y = √x
2

© 2021, BYJU'S. All rights reserved


24
x2 x2 x2
y= y= y=
2 2 2
x Y x x
4. y= y= y=
3 3 3
Step 1: 3π π 3π π y = sin x 3π y π sin33 π π π π 3π
π π
− − = −  x − − − −
Plot the graph of the function y = sin x 2 2 2 2 2 2 22 2 4 2 2 2
1 1 1
Step 2: ( x − 2=
y) 3 ( x − 2) 3
=
y ( x − 2) 3 =
y
Shift the graph of y = sin x in the direction X
5π 9-π
π 3π O π 5π 9π 2π
5π3π9π 3π
 π − − −
= y X-axis
of the positive − units to get
sin  xby 4 4 4 4 4 44 4 4
 4
 π  π
the graph= of y sin  x −  =y sin  x − 
 4  4

5. Y

Step 1: y = log x
Plot the graph of the function y = log x X
O 1

Step 2:
y = log |x|
Plot the graph of y = log |x| by retaining the
graph of y = log x corresponding to only the X
non-negative values of x and taking the -1 0 1
mirror image of the retained graph about
the Y-axis.

Step 3:
y = |log|x||
Plot the graph of y = |log|x|| by retaining the
graph of y = log |x| which is above the X-axis
X
and flipping the graph corresponding to the -1 0 1
negative values of y about the X-axis.

6. Y

Step 1: X
0 1
Plot the graph y = ln x

© 2021, BYJU'S. All rights reserved


25

Y
y = ln {x}

Step 2:
We know the following: X
0 1 2 3
{x} ∈ [0, 1) ⇒ ln x ∈ (-∞, 0)

Self-Assessment Y

Plot the graph for the following:


−3 2
f ( x=
) x + 3x + 1 and g ( x=
) 2x
4 f(x)
g(x)
We can observe that the number of points
of intersection for both the graphs is two. X
-5 -4 -3 -2 -1 1 2 3 4 5
−3 2
Hence, f ( x=
) x + 3x + 1 and g ( x=) 2x
4
have two solutions.

© 2021, BYJU'S. All rights reserved


NOTE
M A T H E M A T I C S

RELATIONS AND FUNCTIONS


PROBLEMS ON FINDING RANGE OF FUNCTIONS

What you already know What you will learn

• Domain and range of a function • Range-based questions


• Transformation of graphs • Composite function

Find the range of the function f(x) = 1 - |x - 2|

Solution Y

We can draw a graph of f(x) = 1 - |x - 2| as


1
shown in the figure.
From the figure, we can conclude that the range X
of the function will be (-∞, 1]. 2
f(x) = 1 - |x - 2|

Find the number of solutions of 3cos x = |sin x| in the interval [-2π, 2π].

Y
Solution 3 f(x) = 3cosx

Step 1: f(x) = |sinx| 1


Plot the graph of f(x) = |sin x|
X
-2π 3π -π π π π 3π 2π
Step 2: - -
2 2 2 2
Plot the graph of f(x) = 3cos x

x 3cos x 1
π Step 3:
3
0 or 2π 3 Both the graphs intersect at 8 points in the given
3π interval as shown in the figure.
π 1
1 2 ∴ Number of solutions = 8
2

© 2021, BYJU'S. All rights reserved


02

Range-Based Questions

Find the range of the following functions:


(1) f(x) = ||x| - 3| (2) f(x) = |ln |-x|| (3) f(x) = [|x|]
1
(4) f(x) = |{x}| (5) f(x) = x + sinx (6) f(x) = x +
x
(7) f(x) = sgn(ex)

Solution

(1) (2)
By plotting the graph of ||x| - 3|, we get, |ln |-x|| = |ln |x||
Range of ||x| - 3| is [0, ∞). By plotting the graph of |ln |x||, we get,
Range of |ln |-x|| is [0, ∞).
Y
Y
(0, 3) y = |ln |x||
y = ||x| - 3|

X
(-3, 0) (3, 0) -1 1 X

(3) (4)
By plotting the graph of [|x|], we get, By plotting the graph of |{x}|, we get,
Range of [|x|] is Whole numbers. Range of |{x}| is [0, 1).
Y Y y = |{x}|
2 1
1 y = [|x|]
0 X
X -2 -1 1 2 3
-3 -2 -1 0 1 2 3

(5) Y
3π y = x + sin x
By plotting the graph of y = sin x on y = x,
we get, 2π
Range of x + sin x is ℝ.
π
-2π -π
X
π 2π 3π

-2π

© 2021, BYJU'S. All rights reserved


03

(6) (7)
By plotting the graph of y = sgn(ex) = 1 as ex > 0, always.
1 By plotting the graph of sgn(ex), we get,
x + , we get, Range of sgn(ex) is 1.
x
1
Range of x + is (-∞, -2] ∪ [2, ∞).
x
Observation Y
1 1
x + ≥ 2 for x > 0, and x + ≤ 2 for x < 0.
x x y=1
Y y = sgn(ex)

1 X
y=x+
x
2
X
-2

 x, − 2 ≤ x ≤ −1
 2
 x + 2x, − 1 < x ≤ 0
If f ( x ) =  , then find the number of integers in the range of f(x).
2
 2x − x , 0 < x ≤ 1
2 − x, 1 < x ≤ 2

Solution
Y
f(x) = x; -2 ≤ x ≤ -1 y = 2x - x2
−2, x = −2
f (x) = 
−1, x = −1 1
y=2-x
f(x) = x + 2x ; -1 < x ≤ 0
2
X
-2 -1 0 1 2
( x ) 0,=x 0
f=
y = x2 + 2
y=x
f(x) = 2x - x ; 0 < x ≤ 1
2

( x ) 1,=
f= x 1 (-2, -2)

f(x) = 2 - x; 1 < x ≤ 2
Range of f(x) is [-2, 1]
( x ) 0,=
f= x 2
∴ Number of integers in the range of
f(x) = 4, i.e., {-2, -1, 0, 1}

© 2021, BYJU'S. All rights reserved


04

Recap

• Range of sin x or sin (f(x)) is [-1, 1] in its respective domain.


• Range of cos x or cos (f(x)) is [-1, 1] in its respective domain.

Type a + b cos x or a + b sin x

Find the range of the following functions:


1
(1) f(x) = 2cos x - 3 (2) f(x) =
9 − 7 sin x
(3) f(x) = sin2 x + cos4 x (4) f(x) = cos2 x + 2cos x - 3

Solution

(1) (2)
We know that -1 ≤ cos x ≤ 1 We know that -1 ≤ sinx ≤ 1
⇒ -2 ≤ 2cos x ≤ 2 ⇒ -7 ≤ 7sin x ≤ 7
⇒ -2 - 3 ≤ 2cos x - 3 ≤ 2 - 3 ⇒ 7 ≥ -7sin x ≥ -7
⇒ -5 ≤ 2cos x - 3 ≤ -1 ⇒ 7 + 9 ≥ 9 -7sin x ≥ -7 + 9
Range of 2cos x - 3 = [-5, -1] ⇒ 16 ≥ 9 -7sin x ≥ 2
1 1 1
≤ ≤
Alternative solution: 16 9 − 7 sin x 2
Substitute the range of cos x in 2cos x - 3. 1  1 1
Range of 2cos x - 3 = [2(-1) - 3, 2(1) - 3] Range of = , 
9 − 7 sin x  16 2 
Range of 2cos x - 3 is [-5, -1].
Alternative solution:
1
Substitute the range of sinx in .
9 − 7 sin x
1  1 1 
Range of = , 
9 − 7 sin x  9 − 7 ( −1) 9 − 7 (1) 
1  1 1
⇒ Range of = , 
9 − 7 sin x  16 2 

Note

This alternative method is applicable only when 0 is not a part of the range/denominator.

© 2021, BYJU'S. All rights reserved


05

(3) (4)
f(x) = sin2 x + (1 - sin2 x)2 By using the completing square method,
⇒ f(x) = sin4 x - sin2 x + 1 f(x) = (cos x + 1)2 - 4
By using the completing square method, We know that -1 ≤ cos x ≤ 1
 2
2
1 1
2 ⇒ 0 ≤ cos x + 1 ≤ 2
f(x) =  sin x −  −   + 1 ⇒ 0 ≤ (cos x + 1)2 ≤ 4
 2 2
2 ⇒ -4 ≤ (cos x + 1)2 - 4 ≤ 0
 1 3 Range of cos2 x + 2cos x - 3 is [-4, 0].
⇒ f(x) =  sin2 x −  +
 2 4
Now, for the minimum value, we know that,
2
 2 1
 sin x − ≥ 0,
 2 
2
 2 1 3 3
 sin x − 2  + 4 ≥ 4 ,
 
Also,
2
 2 1 3
 sin x − 2  + 4 = 1 ,for sin x = 0
2

 
2
 2 1 3
 sin x − 2  + 4 = 1 ,for sin x = 1
2

 
2 2
 2  1 2 31  3
Range of  sin x−sin x+− is+[ , 1]
  2  42  4

Recap

• For asin θ + bcos θ, Range =  − a + b , a + b 


2 2 2 2
 
Example

Find the range of the functions


(1) f(x) = sin x - cos x
We know that for sin x - cos x,
Range =  − 12 + 12 , 12 + 12 
 
Range is  − 2, 2 

(2) f(x) = sin x - cos x + 3


We know that for sin x - cos x + 3,
Range =  − 1 + 1 + 3, 1 + 1 + 3
2 2 2 2
 
Range is 3 − 2, 3 + 2 

© 2021, BYJU'S. All rights reserved


06

Find the range of the function f(x) = 3sin x + 4cos x + 5

Solution

We know that for 3sin x + 4cos x,


Range =  − 3 + 4 , 3 + 4 
2 2 2 2
 
Range = [-5, 5]
∴ Range of 3sin x + 4cos x + 5 is [0, 10]

Type y = ax2 + bx + c

 −D 
• For y = ax2 + bx + c, Range =  , ∞  when a > 0
 4a 
 −D 
• For y = ax2 + bx + c, Range =  −∞ ,  when a < 0
 4a 

Find the range of the function f(x) = x2 + x + 1

Solution

Here, for x2 + x + 1, a = 1, b = 1, c = 1
D = 12 - 4(1)(1) = -3 and a > 0
  −D −D  3 
Range is − ∞ , , ∞
=   , ∞ 
  4a 4a   4 

Range of a composite function

For the range of a composite function.


Let y = g(f(x)),
1. If g(x) is an increasing function, then substitute the range of f(x) into g(x) as it is.
2. If g(x) is a decreasing function, then substitute the range of f(x) into g(x) in the reverse order.
3. If g(x) is neither an increasing nor a decreasing function,
(i) Draw the graph of g(x).
(ii) Highlight the range of f(x) in that graph.
(iii) Consider the height of the highlighted portion.

© 2021, BYJU'S. All rights reserved


07

Find the range of the following functions:


(1) y = 3x 2 + 4x + 5 (2) y = e-(3x
2
+ 4x + 5)

1
(3) y = loge(3x2 + 4x + 5) (4) y = 2
3x + 4x + 5
2
(5) y = sin x + x + 1 (6) y = log5 { 2 ( sin x − cos x ) + 3 }

Solution
Y
(1)
 −D   11 
Range of f(x) = 3x2 + 4x + 5 =  , ∞=   , ∞
 4a  3  y= f (x)
Now, y = f ( x ) is an increasing function as shown
in the graph.
 11 
∴ Range of 3x 2 + 4x + 5 is  , ∞ . X
3 
 

(2) Y
 −D   11 
Range of f(x) = 3x2 + 4x + 5 =  , ∞=   , ∞
 4a  3  y = e-x
Now, y = e is a decreasing function as shown in
-f(x)

the graph.
 −∞ − 113   − 
11
∴ Range of e -(3x2 + 4x + 5)
is  e , e  i.e  0, e 3  X
   

(3) Y
 −D   11 
Range of f(x) = 3x2 + 4x + 5 =  , ∞=   , ∞ y = logex
 4a  3 
Now, y = logef(x) is an increasing function as
shown in the graph.
X
 11 
∴ Range of loge(3x2 + 4x + 5) is log e , log e ∞  i.e
 3 
 11 
log
 e 3 , ∞ 
 

(4) Y
 −D   11  1
Range of f(x) = 3x2 + 4x + 5 =  , ∞=   3 , ∞ y=
 4a    f (x)
1
Now, y = is a decreasing function as shown in
f (x)
the graph.
X
 
1 1 1   3
∴ Range of 2 is  ,  i.e  0, 
3x + 4x + 5  ∞ 11   11 
 3 

© 2021, BYJU'S. All rights reserved


08

(5)
  −D −D  3 
Range of f(x) = x2 + x + 1 = − ∞ , , ∞
=   , ∞ 
  4a 4a   4  Y
 3  y = sin (f(x))
Range of x 2 + x + 1 =  , ∞ 1

 2 
y = sin(f(x)) is neither an increasing nor a decreasing
function as shown in the graph. X
-1
Considering the height of the highlighted graph,
Range of sin x 2 + x + 1 is [-1, 1].

(6)
We know that for sin x - cos x, Y
Range =  − 12 + 12 , 12 + 12  =  − 2, 2 
 
- 2 ≤ sin x - cos x ≤ 2 y = log5f(x)
-2 ≤ 2 (sin x - cos x) ≤ 2
1 ≤ 2 (sin x - cos x) + 3 ≤ 5
Range of 2 (sin x - cos x) + 3 = [1, 5] X
Now, y = log5f(x) is an increasing function as shown in
the graph.
∴ Range of log5{ 2 (sin x - cos x) + 3} is [log51, log55]
i.e [0, 1]

Concept Check

Find the range of the following functions:


 π x8
1. y = 3sin  x −  2. y = ,x≠0 3. y = (sin x)0
 3 x
 π
4. y = (1 − cos x ) (1 − cos x ) .........∞ 5. y =5 cos x + 3 cos  x +  + 3 6. y = 3x2 + 4x + 5
 3

Summary Sheet

Key Takeaways

• For asin θ + bcos θ, Range =  − a + b , a + b 


2 2 2 2
 

© 2021, BYJU'S. All rights reserved


09

 −D 
• For y = ax2 + bx + c, Range =  , ∞  when a > 0
 4a 
 −D 
• For y = ax2 + bx + c, Range =  −∞ ,  when a < 0
 4a 
• For the range of a composite function.
Let y = g(f(x)),
1. If g(x) is an increasing function, then substitute the range of f(x) into g(x) as it is.
2. If g(x) is a decreasing function, then substitute the range of f(x) into g(x) in the reverse order.
3. If g(x) is neither an increasing nor a decreasing function,
(i) Draw the graph of g(x).
(ii) Highlight the range of f(x) in that graph.
(iii) Consider the height of the highlighted portion.

Mind Map

Quadratic functions Signum functions Modulus functions F.P.F functions

Functions Range of functions

Composite functions G.I.F functions Trigonometric functions

Self-Assessment

Find the range of the following functions:


1. f(x) = log{x}, where {x} is an F.P.F.
2. f(x) = logesinx

Answers

Concept Check

1.
 π
y = 3sin  x − 
 3

© 2021, BYJU'S. All rights reserved


10

 π
By plotting the graph of y = 3sin  x −  , we get,
 3
Y
3

π X
2π 5π π 4π 11π 7π
3
3 6 3 6 3

-3
 π
Range of y = 3sin  x −  is [-3, 3].
 3

2. 3.
y = (sin x)0
x8
y= = x7, x ≠ 0 The function (sin x)0 exists only when
x
sin x ≠ 0, i.e., x ≠ nπ
By plotting the graph of y = x7, we get,
Now, by plotting the graph of (sin x)0, we get,
Period of (sin x)0 = π
Y
Domain of (sin x)0 = ℝ - {nπ}
Range of (sin x)0 is 1.

Y
-1
1 X 1

X
-π 0 π 2π

x8
Range of is (-∞,∞) - {0}.
x

(4)

(1 − cos x )
y= (1 − cos x ) .........∞
1 1 1
⇒ y = (1 − cos x ) 2 × (1 − cos x ) 4 × (1 − cos x ) 8 .....
1 1 1
⇒ y = (1 − cos x ) 2
+ + + .....
22 23

1
2
1
1−

⇒ y = (1 − cos x ) = (1 − cos x )
2

Range of 1 - cos x is [1-(1),1-(-1)] i.e [0, 2]

© 2021, BYJU'S. All rights reserved


11

(5)
3 cos x 3 3 sin x
y = 5 cos x + − +3
2 2
13 cos x 3 3 sin x
= − +3
2 2

13 cos x 3 3 sin x
Range of
2

2
[ −7, 7]
=

 π
Range of 5 cos x + 3 cos  x +  + 3 is [-7+3,7+3] i.e [-4, 10]
 3

(6)
Here, for 3x2 + 4x + 5, a = 3, b = 4, c = 5
D = 42 - 4(3)(5) = -44 and a > 0
 −D   44   11 
Range =  , ∞=   , ∞ ⇒  , ∞
 4a   12   3 

Self-Assessment

(1)
We know that 0 ≤ {x} < 1
Now, for {x} = 0, log{x} is not defined.
So, {x} cannot accept integers.
∴ For 0 < {x} < 1,
Range of log{x} is (-∞, 0)

(2)
We know that logesin x is defined only if sin x ∈ (0, 1]
∴ Range of logesin x is (-∞, 0]

© 2021, BYJU'S. All rights reserved


NOTE
M A T H E M A T I C S

RELATIONS AND FUNCTIONS


ODD AND EVEN FUNCTIONS

What you already know What you will learn

• Function • Even and odd functions


• Domain & range of a function • Properties of even and odd functions

Even Function

If f(–x) = f(x) Ɐ x in the domain of f, then f is said to be an even function. Some examples of
even functions are f(x) = cos x, g(x) = |x|, and h(x)= x2 + 3. The graph of every even function is
symmetric about the Y-axis.
Example:
y = f(x) = cos x y = f(x) = x2
Y Y

X X
–3π π π 3π (0,0)
� , 0��– , 0� � , 0� � , 0�
2 2 2 2

In both the cases graph is symmetric about the Y-axis.

Odd Function

If f(–x) = –f(x) Ɐ x in the domain of f, then f is said to be an odd function. Some examples of odd
functions are f(x) = x5, g(x) = sin x, and h(x) = tan x. The graph of an odd function is symmetric
about the origin.

© 2021, BYJU'S. All rights reserved


02

Y Y
y=x y=x 3

X
X

In both cases, the graph is symmetric about the origin.

Note

• For an odd function,


f(–x)= –f(x) Ɐ x in the domain of f.
Now, substituting x = 0, we get,
f(0)= –f(0)
⇒f(0)=0
If an odd function is defined at x = 0, then f(0) = 0
• Some functions may be Neither Even Nor Odd (NENO)
For example, f(x) = 3x + 2 cos x
f(–x) = –3x + 2 cos x
Clearly, for f(x), neither f(–x) = f(x) nor f(–x) = –f(x)
Therefore, f(x) is neither even nor odd.

Check whether the following functions are even or odd.


1+x
(a) f(x) = √1 + x + x2 + √1 – x + x2 (b) f(x) = ln � �, �x� < 1
1– x
ax – a–x ax – 1
(c) f(x) = (d) f(x) = x
ax + a–x a +1

Solution

a. Given, f(x) = √1 + x + x2 + √1 – x + x2 1+x


b. Given, f(x) = ln � � , �x� < 1
⇒f(–x) = √1 – x + (–x)2 + √1 + x + (–x)2 1–x
⇒f(–x) =√1 – x + x2 + √1 + x + x2 1 – x
⇒f(–x) = ln � �
⇒f(–x) =√1 + x + x2 + √1 – x + x2 1+x
⇒f(–x) = f(x)
1+x
Hence, function f(x) is an even function. = –ln � �
1–x
⇒ f(–x) = –f(x) ⇒ Odd function

© 2021, BYJU'S. All rights reserved


03

ax – a–x ax – 1
c. Given, f(x) = x d. Given, f(x) =
a +x a–x ax + 1
a –a
–x
a –1
–x
⇒ f(–x) = –x ⇒ f(–x) = –x
a + ax a +1
ax – a–x ax – 1
⇒ f(–x) = – � x � ⇒ f(–x) = – x
a + a–x a +1
⇒ f(–x) = –f(x) ⇒ Odd function ⇒ f(–x) = –f(x) ⇒ Odd function

ax – 1 n
f(x) = . x is odd then find the value of n.
ax + 1
(a) 1 (b) 2 (c) 4 (d) None of these
3 3 3

Solution

ax – 1 n
Given, f(x)= .x
ax + 1
a–x – 1
⇒ f(–x) = –x (–x)n
a +1
ax – 1
⇒ f(–x) = – � x �(–1)n xn
a +1
ax – 1
⇒ f(–x) =(–1)n+1 � x � xn
a +1
Case 1: when n = odd power then n + 1 = even power
ax – 1
⇒ f(–x) = � x � xn = f(x)= Even function
a +1
Case 2: when n = even power then n + 1 = odd power
ax – 1
⇒ f(–x) = – � x � xn = –f(x)= Odd function
a +1
Hence n can be an even power only
2 4
Therefore, option (b) n = 3 and (c) n = 3 are correct.

x x x ≤ −1

Identify whether the function f ( x ) = [ x +1] + [1 − x ] −1 < x < 1 is an even or an odd
−x x x ≥1

function, where [] represents G.I.F

Solution

x x x ≤ −1

Given function, f ( x ) = [ x +1] + [1 − x ] −1 < x <1
−x x x≥1

© 2021, BYJU'S. All rights reserved


04

( −x ) −x −x ≤ −1

⇒ f ( −x ) = [ −x +1] + [1+ x ] −1 < −x <1
− ( −x ) −x −x ≥ 1

−x x x ≥1

⇒ f ( −x ) = [ x +1] + [1 − x ] −1 < − x <1
x x x ≤ −1

x x x ≤ −1

⇒ f ( −x ) = [ x +1] + [1 − x ] −1 < x <1
−x x x ≥1

⇒ f(x) = f(–x) ⇒ even function.
Step 1 :
x ( sin x + tan x )
f (x) =
x (sin
x +xπ+ 1 x)
 tan
Identify whether the function f ( x ) =  π  − 2 , x ≠ n π & n ∈  is even or odd
 x + π 1

x( sin
π x+ tan
2 x)
⇒ f (x) =
where [.] represents the Greatest Integer Function.
x  1
 π +1 − 2

Solution x ( sin x + tan x )


⇒ f (x) =
x 1
  +1 −
Step 1 : π 2
x ( sin x + tan x ) x ( sin x + tan x )
f (x) = ⇒ f (x) =
x+π 1 x 1
 π  − 2  π  + 2
Step 2:
x ( sin x + tan x ) −x ( −sin x − tan x )
⇒ f (x) = Wef (know
⇒ −x ) =that
x  1  −x  1
 π +1 − 2  x   −x 
+ = − 1asπ x +≠ 2
π  π  π
   
x ( sin x + tan x ) x ( sin x + tan x )
⇒ f (x) = ⇒ f ( −x ) =−x  = − 1 −  x 
Hence,
x 1  π   −x   π 1
 π  +1 − 2     + 
π 2
x ( sin x + tan x )
x ( sin x + tan x ) ⇒ f ( −x ) =
⇒ f (x) = x 1
−1 −   +
Step 2: x 1
 + π 2
We know that  π  2 x ( sin x + tan x )
⇒ f ( −x ) =
⇒x f −x−x=
( )  −x ( −sinxx − tan x ) x 1
 π   π  = − 1 as
+ ≠  −  −
    −xπ 1 π 2
+
 π  2
 −x  x x ( sin x + tan x )
Hence,   = − 1 −  
( ⇒ f ( −x ) = −
π x sin x + π  x)
tan
⇒ f ( −x )=  x 1
 −xx+ tan
x ( sin 1 x)  π  + 2
⇒ f ( −x ) =  π  + 2
x 1 ⇒ f ( −x ) = − f ( x )
−1 −   +
π 2 Hence, f ( x ) is an odd function.
x ( sin x + tan x )
⇒ f ( −x ) =
© 2021, BYJU'S. All rights reserved x  1
−  −
05

Note

If f(x) is difficult to predict then try f(x)+f(-x)=0 or f(x) – f(–x) = 0

1
Find whether the function f ( x ) = [ x ] + , x ∉  is odd/even.
2

Solution

1
Given, f ( x ) = [ x ] +
2
1
⇒ f ( −x ) = [ −x ] +
2
As, [ x ] + [ −x ] = −1 for x ≠ 
⇒ f ( x ) + f ( −x ) = [ x ] + [ −x ] +1
⇒ f ( x ) + f ( −x ) = −1+1
⇒ f ( x ) + f ( −x ) = 0
⇒ f ( − x ) = −f ( x )
Hence f ( x ) is odd.

x x
Identify whether the function f(x) = + + 1 is even or not.
ex – 1 2

Solution

Step 1: Step 2:
x x x x
Given, f(x) = x + +1 ⇒ f(–x)= +x– +1
e –1 2 ex – 1 2
–x –x x x
⇒ f(–x) = –x +� �+ 1 ⇒ f(–x)= x + + 1 = f(x)
e –1 2 e –1 2
–x x Hence, function f(x) is an even function.
⇒ f(–x) = – +1
1 – ex 2
ex
–xex x
⇒ f(–x) = – +1
1 – ex 2
–xex + x – x x
⇒ f(–x) = – +1
1 – ex 2
x(e – 1) + x x
x
⇒ f(–x) = – +1
ex – 1 2

© 2021, BYJU'S. All rights reserved


06

Identify whether the function f ( x ) = ln x + 1 + x 2 ( ) is even or odd.


Solution

(
f ( x ) = ln x + 1+ x 2 )

( −x + 1+ x2 )( x + )
1+ x2 
(
⇒ f ( −x ) = ln −x + 1+ x 2
) = ln  

 (x + 1+ x 2 ) 


 
= ln 
(1 + x ) − x
2 2


 (x + 1 + x2 ) 


 
1
= ln  

(x + )
1 + x2



= −ln ( x + 1+ x ) 2

⇒ f ( −x ) = − f ( x )
Hence, f ( x ) is an odd function.

Properties of Even and Odd Functions

• If f(x) = x2 = Even, and g(x) = |x| = Even


Then,
f.g = x2 ±|x| = Even
f.g = x2 × |x|= Even
f x2
= = Even, x ≠ 0
g |x|

f ; g≠0
f g f±g f×g g

Even Even Even Even Even

• If f(x) = x = Odd, and g(x) = sin x = Odd


Then,

f ; g≠0
f g f±g f×g g

Odd Odd Odd Even Even

© 2021, BYJU'S. All rights reserved


07

• If f(x) = x2 = Even, and g(x) = x = Odd


Then,

f ; g≠0
f g f±g f×g g

Even Odd NENO Odd Odd

Note

f ; g≠0
f g f±g f×g g

Even Even Even Even Even

Odd Odd Odd Even Even

Even Odd NENO Odd Odd

• Conditions of odd/even for composite functions

f g f(g(x))

Even Even Even

Even Odd Even

Odd Even Even

Odd Odd Odd

© 2021, BYJU'S. All rights reserved


08

 3   1 − x  
 sin x + x + tan  ln   
Find whether the function f ( x ) =    1 + x    is odd or even.
 x cos x 
 
 

Solution

Step 1 : Step 2 :
As we know,  3   1− x  
sin x → odd function  sin x + x + tan  ln   
Now, f ( x ) =    1+ x   
x3 → odd function  x cos x 
 
tan x → odd function  
x → even function  O+O+O ( O ) 
⇒ f (x) =  
cos x ⇒ even function  E.E 
 1−x  where O = odd function, E = even function
Let , g ( x ) = ln  
 1+ x   O+O ( O ) 
⇒ f (x) =  
 1+ x  E
⇒ g ( − x ) = ln    
 1−x   O+O 
-1 ⇒ f (x) =  
 1−x   E 
⇒ g ( − x ) = ln  
 1+ x  O
⇒ f (x) =  
 1−x  E
⇒ g ( − x ) = −ln  
 1+ x  ⇒ f ( x ) is an Odd function
⇒ g ( − x ) = −g ( x ) ⇒ odd function
 1−x 
 ln   → odd function
 1+ x 

• The only function that is defined on the entire number line and is even as well as
odd is f(x) = 0
Y
For function f(x) = 0,
f(–x) = f(x) = 0 ⇒ Even function X
f(–x) = –f(x) = 0 ⇒ Odd function 0
y=0

• The function f(x) = k, where k ≠ 0 is an even function.


• All the functions (whose domain is symmetric about origin) can be expressed as the sum of an
even and an odd function.
f(x) + f(–x) f(x) – f(–x)
• f (x) = +
2 2
Even Odd

© 2021, BYJU'S. All rights reserved


09

Proof:
Let’s consider
f(x) + f(–x) f(x) – f(–x)
h(x) = and g(x) =
2 2
f(x) + f(–x) f(-x) – f(x)
Now, h(–x) = and g(-x) =
2 2
⇒ h(– x) = h(x) and g(–x) = –g(x)
Hence, function h(x) is even and g(x) is odd.

sin x
Find whether the function f(x) =� � is even or odd, where [] represents G.I.F
2

Solution

As, �sin x� ∊ [0,1]


⇒ sin x ∊ [0, 1 ]
2 2
⇒�
sin x
2
�=0
f (x) =
( )
x.f x 2
2+ tan x.f ( x )
2 2

⇒ f(x) = 0
Hence, f(x)is both even and odd.
⇒ f ( −x ) =
(
)
−x.f ( −x )
2

2+ ( tan ( −x ) ) .f ( ( −x ) )
2 2

x . f (x ) 2

⇒ f ( −x ) = −
( (x) ) f (x) =
It is given that f ( x ) is an even function and satisfy thex.frelation
2

2+ tan x . f ( x )
2 2

then find the value of f (10 ) ?


2+ tan ( x ) .f x
2
( ) 2

⇒ f ( − x ) = − f ( x ) ⇒ odd function
Given, f ( x ) is an even function
Solution
⇒ f ( x ) is even as well as odd

f (x) =
( )
x.f x 2 Therefore, f ( x ) = 0
2+ tan x.f ( x )
2 2
Hence, f (10) = 0

⇒ f ( −x ) =
) (
−x.f ( −x )
2

2+ ( tan ( −x ) ) .f ( ( −x ) )
2 2

x.f ( ( x ) )
2

⇒ f ( −x ) = −
2+ tan2 ( x ) .f x2 ( )
⇒ f ( − x ) = − f ( x ) ⇒ odd function
Given, f ( x ) is an even function
⇒ f ( x ) is even as well as odd
Therefore, f ( x ) = 0
Hence, f (10) = 0
© 2021, BYJU'S. All rights reserved
10

Express function f(x) = x + ex as the sum of an even and an odd function.

Solution

Given, f(x) = x + ex
As we know that,
f(x) + f(–x) f(x) – f(–x)
f (x) = +
2 2
Even Odd
(x + ex) + (–x + e–x) (x + ex) – (–x + e–x)
f (x) = +
2 2
(ex + e–x) (2x + ex – e–x)
⇒f (x) = +
2 2

Concept Check

1. Find the range of the following functions:


x2 + e
(v) y = 3 sin � π – x2
2
(i) y = ln � �
x2 + 1 16
(ii) y = 3 – 2x x2
(vi) y =
x2 + 1
(iii) y = sin ln x
(iv) y= � π – x2
2

16
ex – e–|x|
2. Let f be a real valued function defined for f(x) = x , then find its range.
1 e + e|x|
a) ℝ b) [0,1] c) [0,1) d) [0, )
2
3. Find the range of y = √x – 4 + √6 – x .
4. Find the range of y = 4x + 2x + 1.

Summary Sheet

Key Takeaways

• If f(–x)= f(x)Ɐ x in the domain of f, then f is said to be an even function.


• If f(–x)= –f(x) Ɐ x in the domain of f, then f is said to be an odd function.
• For an odd function, f(–x) = –f(x) Ɐ x in the domain of f, f(0) = 0
• All the functions (whose domain is symmetric about origin) can be expressed as the sum of an
even and an odd function.

© 2021, BYJU'S. All rights reserved


11

Mind Map

Even function

Relations and Properties of even


Functions and odd functions

Odd function

Self-Assessment

Identify whether the following function is odd/even or neither:


f(x) = x.g(x).g(–x) + tan (sinx)

Answers

Concept Check

1.
(i) Step 1 : Step 2 :
 x +e  2
 e −1 
Given, y = ln  2 ⇒ ln 1 < ln  1+ 2 ≤ ln e

 x +1   x +1 

(
 x 2 +1 + ( e − 1 )  ) 
⇒ 0 < ln  1+ 2
e −1 
≤1
⇒ y = ln    x +1 
 x 2 +1 
  ⇒ 0< y ≤1
 e −1  Hence, Range is ( 0,1].
⇒ y = ln  1+ 2
 x +1 
Now, 0 ≤ x 2 ≤ ∞
⇒ 1 ≤ x 2 +1 ≤ ∞
1
⇒0< 2 ≤1
x +1
e −1
⇒0< 2 ≤ e −1
x +1
e −1
⇒ 1 < 1+ 2 ≤e
x +1

© 2021, BYJU'S. All rights reserved


12

(ii) Given, y =3 − 2x (iii) Given, y = sin lnx


We know that, We know that, − ∞ < ln x < ∞
0 < 2x < ∞ Hence, −1 ≤ sin lnx ≤ 1
⇒ −∞ < −2x < 0 Therefore, range is [-1,1].
⇒ −∞ < 3 − 2x < 3
⇒ −∞ < y < 3
Hence range is ( −∞ ,3) .

(iv) Step 1:
Step 2: �0, π�
π 2
4
Given, y = − x2
16
Squaring both sides,
π2 (0,0)
2
y = − x2
16
π2
⇒ x2 + y 2 =
16 It is clear from the graph that the range is
This represents the portion of �0, π�.
a circle above X - axis. 4

(v) (vi) x2
π2  π Given, y = 2
Since, − x 2 ∈ 0,  x +1
16  4
⇒ yx + y = x2
2
 π2 2   π
⇒ sin 
 16
- x  ∈ sin 0, sin 
  ⇒ ( y − 1) x2 = −y
  4
y
 π2   1  ⇒ x2 =
⇒ sin  − x2  ∈ 0, 
(1 − y )
 16   2
  Since, x2 ≥ 0
 π2   3  y
⇒ 3 sin  − x 2  ∈ 0,  Hence, ≥0
 16

 
 2 (1 − y )
 3  y
⇒ y ∈ 0, ⇒ ≤0
 2
 ( y − 1)
⇒0≤ y <1
Therefore, Range is [0,1) .

© 2021, BYJU'S. All rights reserved


13

2. Step 2 :
Step 1 : 1
⇒ = e2x ≥ 1 as x ≥ 0
( )
-x
ex − e 1 − 2y
Given, y = x
ex + e 1
⇒ ≥1
e − e

x -x
x ≥0
(1 − 2y )
⇒ y =  2ex 1
0 ⇒ −1 ≥ 0
 x <0 (1 − 2y )
ex − e-x 1 − 1+2y
⇒ For x ≥ 0 , y = ⇒ ≥0
2ex (1 − 2y )
e2x − 1 2y
⇒y= ⇒ ≥0
2e2x
(1 − 2y )
⇒ 2y.e2x = e2x − 1
2y
⇒ e2x ( 2y − 1) = −1 ⇒ ≤0
(2y − 1)
1
⇒0≤ y <
2
 1
Hence, Range is 0,  .
 2
So, option (d) is the correct answer.

3.
Step 1 : Step 2 :
Given, y = x − 4 + 6 − x Now,
⇒ x − 4 ≥ 0 and 6 − x ≥ 0 f ( 4) = 4 − 4 + 6 − 4 = 2
⇒ 4 ≤ x ≤ 6 ( domain ) f ( 5) = 5 − 4 + 6 − 5 = 2
dy 1 1
= − f (6) = 6 − 4 + 6 − 6 = 2
dx 2 x − 4 2 6 − x
dy Hence, the minimum value is 2 and
Now for obtaining critical points, =0 maximum value is 2.
dx

1

1
=0 Therefore, Range is  2,2 .
2 x−4 2 6−x
1 1
⇒ =
2 x−4 2 6−x
⇒ x − 4=6 − x Given, y = 4x +2x +1
⇒ x =5 Let 2x = t where t > 0
⇒ y = t 2 + t +1
2
4.  1 3
Given, y = 4x +2x +1 ⇒ y =t +  +
 2 4
Let 2x = t where t > 0 1 3
2

⇒ y min =  0 +  + as t > 0
⇒ y = t 2 + t +1  2 4
Hence, Range is (1, ∞ ) .
2
 1 3
⇒ y =t +  +
 2 4
© 2021, BYJU'S. All rights reserved
2
 1 3
14

Self-Assessment

Given, f ( x ) = x. g ( x ) . g ( −x ) + tan ( sinx )


⇒ f ( −x ) = ( −x ) . g ( −x ) . g ( x ) + tan ( sin ( −x ) )
⇒ f ( −x ) = −x. g ( x ) . g ( −x ) − tan ( sin x )
⇒ f ( − x ) = −f ( − x )
Hence, f ( x ) is an odd function .

© 2021, BYJU'S. All rights reserved


Relations and Functions
Periodic Functions

© 2021, BYJU'S. All rights reserved


Road Map

Examples of Periodic Functions

Properties of Periodic Functions

Periodic Functions

Equal or identical Functions


© 2021, BYJU'S. All rights reserved
Illustration : If function satisfies 𝑓 𝑥 + 𝑦 = 𝑓 𝑥 + 𝑓 𝑦 ∀ 𝑥, 𝑦 ∈ ℝ , then check
if 𝑓(𝑥) is even/odd.
Solution : 𝑓 𝑥 + 𝑦 = 𝑓 𝑥 + 𝑓 𝑦
Put 𝑦 = 𝑥 = 0
⇒𝑓 0+0 =𝑓 0 +𝑓 0
⇒𝑓 0 =𝑓 0 +𝑓 0
⇒𝑓 0 =0
Now, put 𝑦 = −𝑥
𝑓 𝑥 + −𝑥 = 𝑓 𝑥 + 𝑓 −𝑥
So, 𝑓(𝑥) is an Odd function
⇒ 𝑓 0 = 𝑓 𝑥 + 𝑓 −𝑥
0 = 𝑓 𝑥 + 𝑓 −𝑥 ⇒ 𝑓 −𝑥 = −𝑓 𝑥
© 2021, BYJU'S. All rights reserved
Condition Function
𝑓 𝑥+𝑦 =𝑓 𝑥 +𝑓 𝑦 𝑓 𝑥 = 𝑘𝑥

𝑓 𝑥𝑦 = 𝑓 𝑥 + 𝑓 𝑦 𝑓 𝑥 = 𝑘 ln 𝑥

𝑓 𝑥 + 𝑦 = 𝑓 𝑥 .𝑓 𝑦 𝑓 𝑥 = 𝑘𝑥

Explanation :
Let’s try to understand how 𝑓 𝑥 = 𝑘𝑥 satisfies 𝑓 𝑥 + 𝑦 = 𝑓 𝑥 + 𝑓 𝑦
We have, 𝑓 𝑥 = 𝑘𝑥 ⇒ 𝑓 𝑥 + 𝑦 = 𝑘 𝑥 + 𝑦 = 𝑘𝑥 + 𝑘𝑦
Also, 𝑓 𝑦 = 𝑘𝑦 ⇒ 𝑓 𝑥 + 𝑓 𝑦 = 𝑘𝑥 + 𝑘𝑦
∴ 𝑓 𝑥 + 𝑦 = 𝑓 𝑥 + 𝑓(𝑦)
So, whenever a function satisfies 𝑓 𝑥 + 𝑦 = 𝑓 𝑥 + 𝑓 𝑦 , then the
function can be considered of the form 𝑓 𝑥 = 𝑘𝑥
© 2021, BYJU'S. All rights reserved
Illustration : If 𝑓 𝑥 + 𝑦 + 𝑓 𝑥 − 𝑦 = 2𝑓 𝑥 . 𝑓 𝑦 is true for ∀𝑥, 𝑦 ∈ ℝ , then is
𝑓(𝑥) an even or an odd function ?

Solution : 𝑓 𝑥 + 𝑦 + 𝑓 𝑥 − 𝑦 = 2𝑓 𝑥 . 𝑓 𝑦 ⋯ 𝑖 is given.
Now, we replace 𝑥 by 𝑦 and 𝑦 by 𝑥.
𝑓 𝑦 + 𝑥 + 𝑓 𝑦 − 𝑥 = 2𝑓 𝑦 . 𝑓 𝑥 ⋯ 𝑖𝑖
We can see that R.H.S of 𝑖 and (𝑖𝑖) are same
So, L.H.S of 𝑖 = L.H.S of (𝑖𝑖)
⇒𝑓 𝑥+𝑦 +𝑓 𝑥−𝑦 =𝑓 𝑦+𝑥 +𝑓 𝑦−𝑥
Now, put 𝑦 = 0
⇒ 𝑓 𝑥 = 𝑓 −𝑥
Hence, 𝑓 𝑥 is an even function
© 2021, BYJU'S. All rights reserved
Illustration : If 𝑓 𝑥 = 𝑎 + sin 𝑥 is odd function, then find value of 𝑎.

0 1 2 3
Solution :
Case I : 𝑎 is any non-zero number
Let say 𝑎 = 2 ,
𝑓 𝑥 = 2 + sin 𝑥
↓ ↓
even + odd = NENO
Since, 𝑓 𝑥 is sum of even and odd functions so 𝑓 𝑥 is neither even nor odd
But, 𝑓 𝑥 is odd so 𝑎 can not be non zero number
Case II : 𝑎 = 0
𝑓 𝑥 = sin 𝑥 which is odd function ⟹ So, option A is correct answer 𝑎 = 0
© 2021, BYJU'S. All rights reserved
𝑥2
Illustration : If 𝑓: −10,10 → ℝ ; 𝑓 𝑥 = 𝑥 + sin 𝑥 + is odd function, then
𝑎
find range of 𝑎.
Solution :
𝑥2
𝑓 𝑥 = 𝑥 + sin 𝑥 +
𝑎
Here, 𝑥 and sin 𝑥 are odd functions and 𝑓 𝑥 is odd function is given.
𝑥2
So, for 𝑓 𝑥 to be an odd function = 0.
𝑎
𝑥2
⇒0≤ <1
𝑎
Since, 𝑥 2 > 0 ⇒ 𝑎>0
𝑥2
Also, < 1 ⇒ 𝑎 > 𝑥2
𝑎

⇒ 𝑎 > 100 (∵ 𝑥 2 ∈ 0,100 , maximum value of 𝑥 2 is 100)


⇒ 𝑎 ∈ 100, ∞
© 2021, BYJU'S. All rights reserved
𝑥 2 +1
Illustration : If 𝑓: −2,2 → ℝ ; 𝑓 𝑥 = 𝑥 3 + tan 𝑥 + is odd function,
𝑝
then find range of p.
Solution :
2
3
𝑥 +1
𝑓 𝑥 = 𝑥 + tan 𝑥 +
𝑝
Here, 𝑥 3 and tan 𝑥 are odd functions and given 𝑓 𝑥 is an odd function and
𝑥 2 +1
this is possible only when = 0.
𝑝
𝑥 2 +1
⇒0≤ <1
𝑝
𝑥 2 +1 𝑥 2 +1
Since, 0 ≤ & 𝑥2 +1>0⇒𝑝>0 Also, < 1 ⇒ 𝑝 > 𝑥2 + 1
𝑝 𝑝
(∵ 𝑥 2 ∈ 0,4 ⇒ 𝑥 2 + 1 ∈ 1,5 , maximum value of 𝑥 2 + 1 is 5)
⇒ 𝑝 > 5 ⇒ 𝑝 ∈ 5, ∞
© 2021, BYJU'S. All rights reserved
Equal or Identical function:

Functions 𝑓 𝑥 and 𝑔 𝑥 are said to be equal or identical if,


Domain of 𝑓 = Domain of 𝑔
Range of 𝑓 = Range of 𝑔

𝑓 𝑥1 = 𝑔 𝑥2 ∀ 𝑥1 , 𝑥2 ∈ domain (graph of 𝑓 and 𝑔 overlap)

© 2021, BYJU'S. All rights reserved


Illustration : Find whether the following functions are Identical or not ?
𝑥2
𝑓 𝑥 = 𝑥, 𝑔 𝑥 =
𝑥

Solution :
𝑥2
𝑓 𝑥 = 𝑥, 𝑔 𝑥 =
𝑥
𝐷𝑓 ∶ Domain of 𝑓(𝑥) is ℝ

𝐷𝑔 ∶ Domain of 𝑔(𝑥) is ℝ − {0}

So, 𝐷𝑓 ≠ 𝐷𝑔

∴ 𝑓 𝑥 and 𝑔 𝑥 are not identical.


© 2021, BYJU'S. All rights reserved
Illustration : Find whether the following functions are Identical or not ?
𝑓 𝑥 = 1, 𝑔 𝑥 = sin2 𝑥 + cos 2 𝑥
2 2
Solution : 𝑓 𝑥 = 1, 𝑔 𝑥 = sin 𝑥 + cos 𝑥
𝐷𝑓 ∶ Domain of 𝑓(𝑥) is ℝ
𝐷𝑔 ∶ Domain of 𝑔(𝑥) is ℝ
So, 𝐷𝑓 = 𝐷𝑔
𝑔 𝑥 = sin2 𝑥 + cos2 𝑥 = 1 = 𝑓(𝑥) for 𝑥 ∈ ℝ
Here, 𝑓 𝑥 = 𝑔 𝑥 so graph of 𝑓 𝑥 and 𝑔 𝑥 are same.

Hence, 𝑅𝑓 (Range of 𝑓(𝑥)) = 𝑅𝑔 (Range of 𝑔(𝑥))

∴ 𝑓 𝑥 and 𝑔 𝑥 are identical.


© 2021, BYJU'S. All rights reserved
Illustration : Find whether the following functions are Identical or not ?
1 𝑥
𝑓 𝑥 = ,𝑔 𝑥 = 2
𝑥 𝑥
Solution :
𝐷𝑓 ∶ Domain of 𝑓(𝑥) is ℝ − {0}

𝐷𝑔 ∶ Domain of 𝑔(𝑥) is ℝ − {0}


So, 𝐷𝑓 = 𝐷𝑔
𝑥 1
𝑔 𝑥 = = = 𝑓(𝑥), where 𝑥 ≠ 0
𝑥2 𝑥

Here, 𝑓 𝑥 = 𝑔 𝑥 so graph of 𝑓 𝑥 and 𝑔 𝑥 are same.


Hence, 𝑅𝑓 (Range of 𝑓(𝑥)) = 𝑅𝑔 (Range of 𝑔(𝑥))

∴ 𝑓 𝑥 and 𝑔 𝑥 are identical.


© 2021, BYJU'S. All rights reserved
Illustration : Find whether the following functions are Identical or not ?
𝑓 𝑥 = 𝑒 ln 𝑥 , 𝑔 𝑥 = ln 𝑒 𝑥
Solution :
𝐷𝑓 ∶ Domain of 𝑓(𝑥) is ℝ+ (∴ ln 𝑥 is defined for 𝑥 > 0)

𝐷𝑔 ∶ Domain of 𝑔(𝑥) is ℝ (∴ 𝑒 𝑥 > 0 for 𝑥 ∈ ℝ)

So, 𝐷𝑓 ≠ 𝐷𝑔

∴ 𝑓 𝑥 and 𝑔 𝑥 are not identical.

© 2021, BYJU'S. All rights reserved


Illustration : Find whether the following functions are Identical or not ?
𝑓 𝑥 = Sgn(𝑥), 𝑔 𝑥 = Sgn 𝑥 2 − 4𝑥 + 5
Solution : 𝐷𝑓 ∶ Domain of 𝑓(𝑥) is ℝ

𝐷𝑔 ∶ Domain of 𝑔(𝑥) is ℝ So, 𝐷𝑓 = 𝐷𝑔


1 𝑥>0

Sgn 𝑥 = 0 𝑥=0 ⇒ 𝑅𝑓 = Range of 𝑓 𝑥 = {−1, 0, 1}

−1 𝑥 < 0
2
𝑔 𝑥 = Sgn 𝑥 2 − 4𝑥 + 5 = Sgn 𝑥 − 2 2 + 1 (∵ 𝑥 − 2 + 1 > 1)
=1
𝑅𝑔 = range of 𝑔 𝑥 = {1}
So, 𝑅𝑓 ≠ 𝑅𝑔 ∴ 𝑓 𝑥 and 𝑔 𝑥 are not identical.
© 2021, BYJU'S. All rights reserved
Illustration : Find whether the following functions are Identical or not ?
𝑓 𝑥 = cot 2 𝑥 ∙ cos2 𝑥 , 𝑔 𝑥 = cot 2 𝑥 − cos 2 𝑥
cos 2𝑥
Solution : 𝑓 𝑥 = cot 2 𝑥 ∙ cos 2 𝑥 = ∙ cos 2𝑥
sin2 𝑥
𝐷𝑓 ∶ Domain of 𝑓(𝑥) is ℝ − {𝑛𝜋} (∵ sin2 𝑥 = 0 for 𝑥 = 𝑛𝜋)
cos 2 𝑥
𝑔(𝑥) = − cot 2 𝑥
= 2
cos 2 𝑥
− cos 2 𝑥
sin 𝑥
𝐷𝑔 ∶ Domain of 𝑔(𝑥) is ℝ − {𝑛𝜋} (∵ sin2 𝑥 = 0 for 𝑥 = 𝑛𝜋)
So, 𝐷𝑓 = 𝐷𝑔
cos 2 𝑥 2 2
1 cos 2 𝑥 2𝑥
𝑔 𝑥 = − cos 𝑥 = cos 𝑥 − 1 = ∙ cos
sin2 𝑥 sin2 𝑥 sin2 𝑥
Hence, 𝑅𝑓 (Range of 𝑓(𝑥)) = 𝑅𝑔 (Range of 𝑔(𝑥)). So, 𝑓 and 𝑔 Identical
© 2021, BYJU'S. All rights reserved
Periodic Functions:

➢ A periodic function is a function that repeats its values after every


particular interval.

➢ Mathematically, a function 𝑓(𝑥) is said to be a periodic function if


there exists a positive real number 𝑇 such that ,
𝑓 𝑥 + 𝑇 = 𝑓 𝑥 ∀ 𝑥 ∈ 𝐷𝑓 ; 𝑇 > 0
Here, 𝑇 is known as a period of function 𝑓 and the smallest value of 𝑇
is known as the fundamental period.

Example : If 𝑓 𝑥 + 3 = 𝑓 𝑥

Then, period 𝑇of 𝑓 𝑥 is 3


© 2021, BYJU'S. All rights reserved
Periodic Functions:
Example : Find the fundamental period
𝑌 of the function 𝑓 𝑥 = sin 𝑥
Graphical approach
1

𝑋
−8𝜋 −7𝜋 −6𝜋 −5𝜋 −4𝜋 −3𝜋 −2𝜋 −𝜋 0 𝜋 2𝜋 3𝜋 4𝜋 5𝜋 6𝜋 7𝜋 8𝜋

−1
Mathematical approach
𝑓 𝑥 + 𝑇 = 𝑓 𝑥 ⇒ sin 𝑥 + 𝑇 = sin 𝑥
𝑇 2𝑥+𝑇 𝑇
⇒ sin 𝑥 + 𝑇 − sin 𝑥 = 0 ⇒ 2 sin cos = 0 ⇒ sin =0
2 2 2
𝑇
⇒ = 𝑛𝜋 ⇒ 𝑇 = 2𝑛𝜋
2
Thus, fundamental period of sin 𝑥 = 2𝜋
© 2021, BYJU'S. All rights reserved
Illustration : If 𝑓 𝑥 is periodic and 𝑓 𝑥 + 3 + 𝑓 𝑥 = 0, then find its period.

Solution :
𝑓 𝑥+3 +𝑓 𝑥 =0
𝑓 𝑥 = −𝑓 𝑥 + 3 ⋯ (𝑖)
Now, replace 𝑥 by 𝑥 + 3
𝑓 𝑥 + 3 = −𝑓 𝑥 + 6

⇒ −𝑓 𝑥 + 3 = 𝑓 𝑥 + 6 ⋯ (𝑖𝑖)

From equation 𝑖 and 𝑖𝑖 ,

𝑓 𝑥 =𝑓 𝑥+6 (∵ 𝑓 𝑥 = 𝑓 𝑥 + 𝑇 then 𝑇 is the period of 𝑓(𝑥))

Hence, Period of 𝑓(𝑥) is 𝑇 = 6


© 2021, BYJU'S. All rights reserved
Illustration : If 𝑓 𝑥 is periodic and 𝑓 𝑥 + 𝑓 𝑥 + 4 = 𝑓 𝑥 + 2 + 𝑓(𝑥 + 6),
then find its period 𝑇.

Solution :
𝑓 𝑥 + 𝑓 𝑥 + 4 = 𝑓 𝑥 + 2 + 𝑓(𝑥 + 6) ⋯ (𝑖)
Now, replace 𝑥 by 𝑥 + 2
𝑓 𝑥 + 2 + 𝑓 𝑥 + 6 = 𝑓 𝑥 + 4 + 𝑓 𝑥 + 8 ⋯ (𝑖𝑖)
From equation 𝑖 and (𝑖𝑖)
𝑓 𝑥 +𝑓 𝑥+4 =𝑓 𝑥+4 +𝑓 𝑥+8
𝑓 𝑥 =𝑓 𝑥+8
Hence, Period of 𝑓(𝑥) is 𝑇 = 8
© 2021, BYJU'S. All rights reserved
Illustration : If a function 𝑓(𝑥) is odd and periodic with period 2, then find 𝑓(4)
Solution :
𝑓(𝑥) is an odd function
⇒ 𝑓 −𝑥 = −𝑓 𝑥 ⋯ (𝑖)
𝑓(𝑥) periodic with period 2
⇒ ⋯ = 𝑓 −2 = 𝑓 0 = 𝑓 2 = 𝑓 4 = 𝑓 6 = ⋯
⇒ 𝑓 0 = 𝑓(4)
Now, substitute 𝑥 = 0 in 𝑖 , We get,
𝑓 0 = −𝑓 0
⇒ 2𝑓 0 = 0 ⇒ 𝑓 0 = 0
∴𝑓 4 =0
© 2021, BYJU'S. All rights reserved
Periodic Functions:
Note
Fundamental period
Functions Nature of 𝑛
𝑇

sin𝑛 𝑥 , cos 𝑛 𝑥 , 𝑛 = odd 2𝜋


cosec 𝑛 𝑥, sec 𝑛 𝑥
𝑛 = even 𝜋

tan𝑛 𝑥 , cot 𝑛 𝑥 𝑛 = even/odd 𝜋

| sin𝑛 𝑥 |, | cos 𝑛 𝑥 |,
cosec 𝑛 𝑥 , sec 𝑛 𝑥 , 𝑛 = even/odd 𝜋
|tan𝑛 𝑥 |, | cot 𝑛 𝑥 |
© 2021, BYJU'S. All rights reserved
Illustration : Find the fundamental period of following functions

➢ 𝑓 𝑥 = sin3 𝑥
Degree of sin 𝑥 is odd so period is 𝑇 = 2𝜋
➢ 𝑓 𝑥 = sin2 𝑥
Degree of sin 𝑥 is even so period is 𝑇 = 𝜋
➢ 𝑓 𝑥 = cos 31 𝑥
Degree of cos 𝑥 is odd so period is 𝑇 = 2𝜋
➢ 𝑓 𝑥 = sec 2 𝑥

Degree of sec 𝑥 is even so period is 𝑇 = 𝜋

© 2021, BYJU'S. All rights reserved


Illustration : Find the fundamental period of following functions

➢ 𝑓 𝑥 = tan2 𝑥

Degree of tangent is even so period is 𝑇 = 𝜋

➢ 𝑓 𝑥 = cot 3 𝑥

Degree of cotangent is odd so period is 𝑇 = 𝜋

➢ 𝑓 𝑥 = | sin3 𝑥 |

Period of | sin𝑛 𝑥| (𝑛 = even/odd) is 𝜋 so period of 𝑓(𝑥) is 𝑇 = 𝜋

© 2021, BYJU'S. All rights reserved


Note Functions 𝑇

sin 𝑥 + cos 𝑥 , tan 𝑥 + cot 𝑥 , 𝜋


sec 𝑥 + |cosec 𝑥| 2
𝜋
sin𝐸 𝑥+ cos 𝐸 𝑥 (𝐸 = even > 2)
2

𝑥 − 𝑥 = {𝑥} 1

Constant Function Fundamental period


is undefined
1 , 𝑥 = 𝑄 Rational number Periodic but no
Dirichlet 𝑓(𝑥)
fundamental
Function
−1, 𝑥 = 𝑄′ (irrational number) Period
© 2021, BYJU'S. All rights reserved
Illustration : Find the fundamental period of following functions

➢ sin4 𝑥 + cos 4 𝑥
𝜋
sin𝐸 𝑥 + cos 𝐸 𝑥 (𝐸 = 𝑒𝑣𝑒𝑛) then period is
2

➢ sin2 𝑥 + cos 2 𝑥

sin2 𝑥 + cos 2 𝑥 = 1 (constant)


So, sin2 𝑥 + cos 2 𝑥 is constant function
Hence, fundamental period is not defined

© 2021, BYJU'S. All rights reserved


Illustration : Find the fundamental period of following functions

➢ 𝑓 𝑥 = sin 𝑥
Degree of sin 𝑥 is odd so period is 𝑇 = 2𝜋
➢ 𝑓 𝑥 = sec 𝑥
Degree of sec 𝑥 is odd so period is 𝑇 = 2𝜋
➢ 𝑓 𝑥 = sec 4 𝑥

Degree of sec 𝑥 is even so period is 𝑇 = 𝜋


➢ 𝑓 𝑥 = tan 𝑥

Degree of tan 𝑥 is odd so period is 𝑇 = 𝜋

© 2021, BYJU'S. All rights reserved


Illustration : Find the fundamental period of following functions

➢ 𝑓 𝑥 = sin2 𝑥
Degree of sin 𝑥 is even so period is 𝑇 = 𝜋
➢ 𝑓 𝑥 = sec 3 𝑥

Degree of sec 𝑥 is odd so period is 𝑇 = 2𝜋


➢ 𝑓 𝑥 = | sin 𝑥 |

Period of | sin 𝑥 | 𝑇 = 𝜋
➢ 𝑓 𝑥 = tan3 𝑥
Degree of tan 𝑥 is odd and so period is 𝑇 = 𝜋

© 2021, BYJU'S. All rights reserved


Illustration : Find the fundamental period of following functions
➢ 𝑓 𝑥 = sin31 𝑥
Degree of sin 𝑥 is odd so period is 𝑇 = 2𝜋
➢ 𝑓 𝑥 = cosec 𝑥

Degree of cosec 𝑥 is odd so period is 𝑇 = 2𝜋

➢ 𝑓 𝑥 = | sin3 𝑥 |

Period of | sin𝑛 𝑥| (𝑛 = even/odd) is 𝜋 so period of 𝑓(𝑥) is 𝑇 = 𝜋


➢ 𝑓 𝑥 = | sec 32 𝑥 |

Period of | s𝑒𝑐 𝑛 𝑥| (𝑛 = even/odd) is 𝜋 so period of 𝑓(𝑥) is 𝑇 = 𝜋

© 2021, BYJU'S. All rights reserved


Illustration : Find the fundamental period of following functions

➢ 𝑓 𝑥 = sin82 𝑥
Degree of sin 𝑥 is even so period is 𝑇 = 𝜋

➢ 𝑓 𝑥 = cosec 34 𝑥

Degree of cosec 𝑥 is even so period is 𝑇 = 𝜋

➢ 𝑓 𝑥 = | cot 51 𝑥 |

Period of | cot 𝑛 𝑥| (𝑛 = even/odd) is 𝜋 so period of 𝑓(𝑥) is 𝑇 = 𝜋

© 2021, BYJU'S. All rights reserved


Illustration : Find the fundamental period of following functions

➢ 𝑓 𝑥 = sin 𝑥 + | cos 𝑥 |
𝜋
Period of sin 𝑥 + cos 𝑥 is 𝑇 =
2
𝑌
𝑥
➢ 𝑓 𝑥 = 𝑥 ({. } denotes fractional part function) 𝑌=1

𝑋
Period of x is 𝑇 = 1 −2 −1 0 1 2

➢ 𝑓 𝑥 = tan4 𝑥 + cot 4 𝑥

𝜋 𝜋
Period of |tann 𝑥| + |cot 𝑛 𝑥| (𝑛 = even) is so period of 𝑓(𝑥) is 𝑇 =
2 2

© 2021, BYJU'S. All rights reserved


1 + 2 cos 𝑥
Illustration : Find the fundamental period of function 𝑓 𝑥 =
sin 𝑥 (2 + sec 𝑥)

Solution :
1 + 2 cos 𝑥
𝑓 𝑥 =
sin 𝑥 (2 + sec 𝑥)
1 + 2 cos 𝑥
=
2 cos 𝑥 + 1
sin 𝑥 cos 𝑥
cos 𝑥
=
sin 𝑥
𝑓 𝑥 = cot 𝑥

Hence, Period of 𝑓(𝑥) is 𝑇 = 𝜋


© 2021, BYJU'S. All rights reserved
When constant is involved with 𝑓(𝑥)
➢ If a function 𝑓 𝑥 has a period 𝑇, then
Example : If 𝑓 𝑥 = sin 𝑥
𝑓 𝑥 ±𝑘
𝑓 𝑥±𝑘 𝑓 𝑥 + 2 = sin 𝑥 + 2 2𝑓 𝑥 = 2 sin 𝑥
𝑘𝑓(𝑥) 𝑓(𝑥+2) sin(𝑥+2)
=
𝑓 𝑥
3 3 2𝑓 𝑥 = 2sin 𝑥
𝑘
1 1
1 Period remains 𝑇 =
𝑓 𝑥 𝑠𝑖𝑛𝑥 𝑓 𝑥 = sin 𝑥
𝑓 𝑥
𝑓 𝑥 In all these examples period remain 2𝜋
𝑓(𝑥)
𝑘
© 2021, BYJU'S. All rights reserved
Illustration : Find the period of following functions
2 sin(𝑥+5)
−13
➢ 𝑓 𝑥 =𝑒 3

2𝑔(𝑥+5) 2 sin(𝑥+5)
−13 −13
If 𝑔 𝑥 = sin 𝑥 then 𝑒 3 =𝑒 3

So, period of 𝑓(𝑥) is remain 2𝜋


1
7 𝑥−16 +𝑘
➢ 𝑓 𝑥 = 3
1 1
7𝑔 𝑥− +𝑘 7 𝑥−16 +𝑘
If 𝑔 𝑥 = {𝑥} then 3 16 = 3
So, period of 𝑓(𝑥) is remain 𝑇 = 1

© 2021, BYJU'S. All rights reserved


Illustration : Find the period of following functions
2
➢ 𝑓 𝑥 =
𝜋 sin 𝑥+5
−13
𝑒 3

2 2
=
𝜋𝑔(𝑥+5) 𝜋 sin 𝑥+5
−13 −13
If 𝑔 𝑥 = sin 𝑥 then 𝑒 3 𝑒 3

So, period of 𝑓(𝑥) is remain 2𝜋


2 1
➢ 𝑓 𝑥 = 𝑥− +1
9 3
2 1 2 1
If 𝑔 𝑥 = {𝑥} then 9 𝑔 𝑥 − 3 + 1 = 9 𝑥 − 3 + 1

So, period of 𝑓(𝑥) is remain 𝑇 = 1


© 2021, BYJU'S. All rights reserved
Properties of Periodic Functions:

𝑇
If a function 𝑓 𝑥 has a period 𝑇, then 𝑓 𝑘𝑥 has the period .
𝑘

Example :
34
➢ tan 𝑥
𝜋
Here, coefficient of 𝑥 is 1 so period is 𝑇 = =𝜋
1
34
➢ tan 3𝑥
𝜋
Here, coefficient of 𝑥 is 3 so period is 𝑇 =
3
𝑥
➢ tan34
9
1 𝜋
Here, coefficient of 𝑥 is so period is 𝑇 = 1 = 9𝜋
9
© 2021, BYJU'S. All rights reserved 9
sin 𝑥+sin 2𝑥+sin 4𝑥+sin 5𝑥
Illustration : If 𝑓 𝑥 = then find period of 𝑓(𝑥)
cos 𝑥+cos 2𝑥+cos 4𝑥+cos 5𝑥

sin 𝑥+sin 2𝑥+sin 4𝑥+sin 5𝑥 (sin 𝑥+sin 5𝑥)+(sin 2𝑥+sin 4𝑥)


Solution : 𝑓 𝑥 = =
cos 𝑥+cos 2𝑥+cos 4𝑥+cos 5𝑥 (cos 𝑥+cos 5𝑥)+(cos 2𝑥+cos 4𝑥)

Now,
sin 2𝑥 + sin 4𝑥 = 2 sin 3𝑥 cos 𝑥
cos 2𝑥 + cos 4𝑥 = 2 cos 3𝑥 cos 𝑥
sin 𝑥 + sin 5𝑥 = 2 sin 3𝑥 cos 2𝑥
cos 𝑥 + cos 5𝑥 = 2 cos 3𝑥 cos 2𝑥
2 sin 3𝑥 cos 2𝑥 + 2 sin 3𝑥 cos 𝑥
∴𝑓 𝑥 =
2 cos 3𝑥 cos 2𝑥 + 2 cos 3𝑥 cos 𝑥
2 sin 3𝑥 (cos 𝑥 + cos 2𝑥) 𝜋
∴𝑓 𝑥 = = tan 3𝑥 ∴ Period 𝑇 =
2 cos 3𝑥 (cos 𝑥 + cos 2𝑥) 3
© 2021, BYJU'S. All rights reserved
𝑇
Illustration : If period of 𝑒 3(𝑥− 𝑥 ) is 𝑇1 and period of 𝑒 3𝑥−[3𝑥] is 𝑇2 then find 1.
𝑇2

Solution :
𝑓(𝑥)= 𝑒 3(𝑥− 𝑥 ) 𝑔(𝑥) = 𝑒 3𝑥− 3𝑥 )

= 𝑒 3{𝑥} (∵ 𝑥 = 𝑥 + 𝑥 ) = 𝑒 {3𝑥} (∵ 3𝑥 = 3𝑥 + 3𝑥 )
1
So, period of 𝑓(𝑥) is 𝑇1 = 1 So, period of 𝑓(𝑥) is 𝑇2 =
3

𝑇1 1
∴ = 1 =3
𝑇2
3

© 2021, BYJU'S. All rights reserved


π
Illustration : If period of tan [𝑛] 𝑥 is 3 then find range of 𝑛.
Solution :
Period of tan 𝑥 is 𝑇 = 𝜋

In tan [𝑛] 𝑥 coefficient of 𝑥 is [𝑛]

So, period of tan [𝑛] 𝑥 is given by


𝜋 𝜋
⇒𝑇= = (Given)
𝑛 3

⇒ 𝑛 =3

⇒ 𝑛 =9

⇒ 𝑛 ∈ 9,10
© 2021, BYJU'S. All rights reserved
Summary Sheet

➢ A periodic function is a function that repeats its values after every


particular interval.

➢ Mathematically, a function 𝑓(𝑥) is said to be a periodic function if there


exists a positive real number 𝑇 such that , 𝑓 𝑥 + 𝑇 = 𝑓 𝑥 ∀ 𝑥 ∈ 𝐷𝑓 ;
𝑇>0
Here, 𝑇 is known as a period of function 𝑓 and the smallest value of 𝑇 is
known as the fundamental period.

© 2021, BYJU'S. All rights reserved


Summary Sheet

Fundamental period
Functions Nature of 𝑛
𝑇

sin𝑛 𝑥 , cos 𝑛 𝑥 , 𝑛 = odd 2𝜋


cosec 𝑛 𝑥, sec 𝑛 𝑥
𝑛 = even 𝜋

tan𝑛 𝑥 , cot 𝑛 𝑥 𝑛 = even/odd 𝜋

| sin𝑛 𝑥 |, | cos 𝑛 𝑥 |,
cosec 𝑛 𝑥 , sec 𝑛 𝑥 , 𝑛 = even/odd 𝜋
|tan𝑛 𝑥 |, | cot 𝑛 𝑥 |
© 2021, BYJU'S. All rights reserved
Functions 𝑇

sin 𝑥 + cos 𝑥 , tan 𝑥 + cot 𝑥 , 𝜋


sec 𝑥 + |cosec 𝑥| 2
𝜋
sin𝐸 𝑥+ cos 𝐸 𝑥 (𝐸 = even > 2)
2

𝑥 − 𝑥 = {𝑥} 1

Constant Function Fundamental period


is undefined
1 , 𝑥 = 𝑄 Rational number Periodic but no
Dirichlet 𝑓(𝑥)
fundamental
Function
−1, 𝑥 = 𝑄′ (irrational number) Period
© 2021, BYJU'S. All rights reserved
Relations and Functions
Solving problems on Periodic functions

© 2021, BYJU'S. All rights reserved


Road Map

Types of Functions

Period of Composite Function

Methods to find period of a


function

Period of various kind of Functions

© 2021, BYJU'S. All rights reserved


Recall

Fundamental period
Functions Nature of 𝑛
𝑇

sin𝑛 𝑥 , cos 𝑛 𝑥 , 𝑛 = odd 2𝜋


cosec 𝑛 𝑥, sec 𝑛 𝑥
𝑛 = even 𝜋

tan𝑛 𝑥 , cot 𝑛 𝑥 𝑛 = even/odd 𝜋

| sin𝑛 𝑥 |, | cos 𝑛 𝑥 |,
cosec 𝑛 𝑥 , sec 𝑛 𝑥 , 𝑛 = even/odd 𝜋
|tan𝑛 𝑥 |, | cot 𝑛 𝑥 |

© 2021, BYJU'S. All rights reserved


Functions 𝑇

sin 𝑥 + cos 𝑥 , tan 𝑥 + cot 𝑥 , 𝜋


sec 𝑥 + |cosec 𝑥| 2
𝜋
sin𝐸 𝑥 + cos 𝐸 𝑥 (𝐸 = even > 2)
2

𝑥 − 𝑥 = {𝑥} 1

Constant Function Fundamental period


is undefined
1 , 𝑥 = 𝑄 Rational number Periodic but no
Dirichlet 𝑓(𝑥)
fundamental
Function
−1, 𝑥 = 𝑄′ (irrational number) Period
© 2021, BYJU'S. All rights reserved
When constant is involved to 𝑓(𝑥)
➢ If a function 𝑓 𝑥 has a period 𝑇, then

𝑓 𝑥 ±𝑘 Example : If 𝑓 𝑥 = 𝑠𝑖𝑛𝑥
𝑓 𝑥±𝑘
𝑓 𝑥 + 2 = 𝑠𝑖𝑛𝑥 + 2 2𝑓 𝑥 = 2𝑠𝑖𝑛𝑥
𝑘𝑓(𝑥)
𝑓(𝑥+2) sin(𝑥+2)
𝑘𝑓 𝑥 =
3 3 2𝑓 𝑥 = 2𝑠𝑖𝑛𝑥
1 Period remains 𝑇
1 1
𝑓 𝑥 = 𝑓 𝑥 = 𝑠𝑖𝑛𝑥
𝑓 𝑥 𝑠𝑖𝑛𝑥
𝑓 𝑥
𝑓(𝑥) In all these examples period remain 2𝜋
𝑘
© 2021, BYJU'S. All rights reserved
Illustration : Find the period of following functions.

➢ 𝑓 𝑥 = sec 3 𝑥
Degree of 𝑠𝑒𝑐𝑎𝑛𝑡 is odd so period is 𝑇 = 2𝜋

➢ 𝑓 𝑥 = sec 3 3𝑥
2𝜋
Degree of 𝑠𝑒𝑐𝑎𝑛𝑡 is odd and coefficient of 𝑥 is 3 so period is 𝑇 =
3
𝑥
➢ 𝑓 𝑥 = sec 3
4 1 2𝜋
Degree of 𝑠𝑒𝑐𝑎𝑛𝑡 is odd and coefficient of 𝑥 is so period is 𝑇 = 1 = 8𝜋
4
4
𝑥
➢ 𝑓 𝑥 = tan
𝑛!
1 𝜋
Degree of tangent is odd and coefficient of 𝑥 is so period is 𝑇 = 1 = 𝑛! 𝜋
𝑛!
𝑛!

© 2021, BYJU'S. All rights reserved


Illustration : Find the period of following functions.
{3𝑥−4}
➢ 𝑓 𝑥 = + 3
5
While adding, subtracting or dividing by any non-zero constant, period does no
1
change. But, here coefficient of 𝑥 is 3 so period is 𝑇 =
3

cos2 (𝜋𝑥−3)
➢ 𝑓 𝑥 =
5
𝜋
Degree of 𝑐𝑜𝑠𝑖𝑛𝑒 is even and coefficient of 𝑥 is 𝜋 so period is 𝑇 = =1
𝜋
𝑥
➢ 𝑓 𝑥 = sin −1
2

Here square root does not affect the period.


1 2𝜋
Power of 𝑠𝑖𝑛𝑒 is odd and coefficient of 𝑥 is so period is 𝑇 = 1 = 4𝜋
2
2

© 2021, BYJU'S. All rights reserved


Illustration :Find the period of following functions.
𝑥−𝜋 𝑥 𝜋
➢ 𝑓 𝑥 = cos = 𝑐𝑜𝑠 −
3 3 3
𝜋
Here, − is the angle that does not affect the period of function.
3
1 2𝜋
Degree of 𝑐𝑜𝑠𝑖𝑛𝑒 is odd and coefficient of 𝑥 is so period is 𝑇 = 1 = 6𝜋
3
3
|𝑡𝑎𝑛 8𝑥+9 | 𝜋
➢ 𝑓 𝑥 = +
7 2
𝜋
Period of 𝑡𝑎𝑛𝑥 is 𝜋 and coefficient of 𝑥 is 8 so period is 𝑇 =
8
7 7
➢ 𝑓 𝑥 = =
𝜋 sin 𝑥
cos 2 −𝑥

Here, square root does not affect the period.


Degree of 𝑠𝑖𝑛𝑒 is odd so period is 𝑇 = 2𝜋
© 2021, BYJU'S. All rights reserved
When 2 or more functions are Added/Subtracted/multiplied

➢ When 2 or more functions are given,


ℎ 𝑥 = 𝑓 𝑥 ± 𝑔 𝑥 or 𝑓 𝑥 . 𝑔 𝑥 or combination of these ,
Where 𝑇1 , 𝑇2 be the period of 𝑓 𝑥 and 𝑔(𝑥) respectively.
Period of ℎ(𝑥) can be calculated by taking L.C.M of 𝑇1 & 𝑇2
𝑇 = L.C.M. 𝑇1 , 𝑇2 (Period T may or may not be fundamental period)

Note
L.C.M is only possible when 𝑇1 & 𝑇2 are of same type

© 2021, BYJU'S. All rights reserved


𝑥 𝑥
Illustration : If 𝑓 𝑥 = sin 2 + cos 22 then find period of 𝑓(𝑥).

Solution:
𝑥 𝑥
𝑓 𝑥 = sin + cos 2
2 2
𝑥 𝑥
⇒ 𝑓 𝑥 = sin + cos
2 4
𝑥 2𝜋
Period of sin is 𝑇1 = 1 = 4𝜋
2
2

𝑥 2𝜋
Period of cos is 𝑇2 = 1 = 8𝜋
4
4

Period of 𝑓(𝑥) is given by 𝑇 = L.C.M. 4𝜋, 8𝜋 = 8𝜋

© 2021, BYJU'S. All rights reserved


𝜋𝑥
tan 𝜋 𝑥 +sin 𝜋𝑥+cos2 then find Period of 𝑓 𝑥 .
Illustration : 𝑓 𝑥 = 2 4

([.] denote the greatest integer function)


Solution:
𝜋𝑥
tan 𝜋 𝑥 +sin 𝜋𝑥+cos2 4
𝑓 𝑥 = 2

𝑥 always gives integer value so we


can replace it by integer n Period of 𝑓 𝑥 is given by

tan 𝜋 𝑥 = tan 𝑛𝜋 = 0 𝑇 = L.C.M. (2,4) = 4

2𝜋
Period of sin 𝜋𝑥 → 𝑇1 = =2
𝜋

𝜋𝑥 𝜋
Period of cos 2 → 𝑇2 = 𝜋 =4
4
4
© 2021, BYJU'S. All rights reserved
Illustration : If 𝑓 𝑥 , 𝑔 𝑥 are two periodic functions having period 7 and 11
𝑥 𝑥
respectively, then find the period of ℎ 𝑥 = 𝑓 𝑥 𝑔 −𝑔 𝑥 𝑓 .
5 5
𝑥 𝑥
Solution: ℎ 𝑥 = 𝑓 𝑥 𝑔 −𝑔 𝑥 𝑓
5 5

Period of 𝑓 𝑥 is given by 𝑇1 = 7 Period of ℎ 𝑥 is given by


𝑥 11 ∴ 𝑇 = L.C.M (𝑇1 , 𝑇2 , 𝑇3 , 𝑇4 )
Period of 𝑔 is given by 𝑇2 = 1 = 55
5
5 ∴ 𝑇 = L.C.M (7, 55, 11, 35)
Period of 𝑔 𝑥 is given by 𝑇3 = 11
⇒ 𝑇 = 7 × 5 × 11= 385
𝑥 7
Period of 𝑓 is given by 𝑇4 = 1 = 35
5
5

© 2021, BYJU'S. All rights reserved


Illustration : If ℎ 𝑥 = sin 𝑥 + cosec 2 2𝑥
then find period of ℎ(𝑥).
3 5
Solution:
𝑥 2
2𝑥
ℎ 𝑥 = sin + cosec
3 5
𝑥 2𝜋
Period of 𝑠𝑖𝑛 3 → 𝑇1 = 1 = 6𝜋
3
2𝑥 𝜋 5𝜋
Period of 𝑐𝑜𝑠𝑒𝑐 2 5
→ 𝑇2 = 2 = 2
5

Period of ℎ 𝑥 is given by

5𝜋 L.C.M. 6𝜋, 5𝜋 𝑎 𝑐 L.C.M. 𝑎, 𝑐


𝑇 = L.C.M. 6𝜋, = ∵ L.C.M. , =
2 H.C.F. (1, 2) 𝑏 𝑑 H.C.F. (𝑏, 𝑑)

30𝜋
𝑇= ⟹ 𝑇 = 30𝜋
© 2021, BYJU'S. All rights reserved
1
Illustration : If 𝑓 𝑥 = sin2 3𝑥 + cos2 4𝑥 then find period of 𝑓(𝑥).
Solution:
𝑓 𝑥 = sin2 3𝑥 + cos2 4𝑥
𝜋
Period of 𝑠𝑖𝑛2 3𝑥 is 𝑇1 =
3
𝜋
Period of 𝑐𝑜𝑠 2 4𝑥 is 𝑇2 = 4

Period of 𝑓 𝑥 is given by
𝜋 𝜋 L.C.M. 𝜋, 𝜋 𝑎 𝑐 L.C.M. 𝑎,𝑐
𝑇 = L.C.M. , = ∵ L.C.M. , =
3 4 H.C.F. (3, 4) 𝑏 𝑑 H.C.F. (𝑏,𝑑)
𝜋
=
1
⇒𝑇=𝜋

© 2021, BYJU'S. All rights reserved


Illustration :Find period of the function ℎ 𝑥 = sec 𝑥 + 𝑥 .

Solution:
ℎ 𝑥 = sec 𝑥 + {𝑥}
Period of 𝑠𝑒𝑐 𝑥 is 2𝜋
Period of 𝑥 is 1
Period of ℎ 𝑥 is given by
𝑇 = L.C.M. 2𝜋, 1
2𝜋 is irrational number and 1 is rational number. So, the L.C.M does
not exist.

© 2021, BYJU'S. All rights reserved


Illustration : If ℎ 𝑥 = sec 𝑥 + 𝑥 then find period of ℎ 𝑥 .
Solution:
ℎ 𝑥 = sec 𝑥 + 𝑥

sec 𝑥 is a periodic function and period is 2𝜋


𝑥 is non periodic function
ℎ 𝑥 is sum of 𝑠𝑒𝑐 𝑥 (periodic) and 𝑥 (non periodic)
Periodic + Non Periodic = Non-Periodic
So, ℎ 𝑥 is non periodic function
⇒Period of the function is not defined.
Note
Sum of Periodic and non Periodic function is always non periodic
© 2021, BYJU'S. All rights reserved
Illustration : If 𝑓 𝑥 = max(sin 𝑥 , |𝑥|) then find period of 𝑓 𝑥 .

Solution: 𝑓 𝑥 = max(sin 𝑥 , |𝑥|)= |𝑥|


𝑦 = −𝑥 𝑦=𝑥
From graph we can see that
𝑚𝑎𝑥(𝑠𝑖𝑛 𝑥 , |𝑥|) = |𝑥|
𝑦 = sin 𝑥
So, 𝑓 𝑥 = |𝑥|
𝑥 is non periodic function
⇒ 𝑓 𝑥 is Non Periodic

© 2021, BYJU'S. All rights reserved


Illustration : If 𝑓 𝑥 = min(sin 𝑥 , |𝑥|) then find period of 𝑓 𝑥 .

Solution: 𝑓 𝑥 = min(sin 𝑥 , |𝑥|) = sin 𝑥


𝑦 = −𝑥 𝑦=𝑥
From graph you can see that
min(sin 𝑥 , |𝑥|) = sin 𝑥
𝑦 = sin 𝑥
So, 𝑓 𝑥 = 𝑠𝑖𝑛 𝑥
Period of sin 𝑥 is 2𝜋
𝑇 = 2𝜋

© 2021, BYJU'S. All rights reserved


1
Illustration : Find period of 𝑓 𝑥 =
1−cos 𝑥

Solution:
1
𝑓 𝑥 =
1−cos 𝑥

1
= 𝑥
2 sin2
2
1
Here degree of sin 𝑥 is even and coefficient of 𝑥 is
2
𝑥 𝜋
So Period of sin2 is = 1 = 2𝜋
2
2

𝜋
Period of 𝑓 𝑥 = 1 = 2𝜋
2

© 2021, BYJU'S. All rights reserved


sin 12𝑥
Illustration : Find period of 𝑓 𝑥 = 1+cos2 6𝑥
Solution:
sin 12𝑥 1+cos 12𝑥
𝑓 𝑥 = 1 1
∴ cos 2 6𝑥 =
1+2+ 2 cos 12𝑥 2

sin 12𝑥
=3 1
+ cos 12𝑥
2 2

2𝜋 𝜋
Period of sin 12𝑥 = =
12 6
2𝜋 𝜋
Period of cos 12𝑥 = =
12 6

𝜋 𝜋 𝜋
Period of 𝑓 𝑥 = L.C.M. , So, period =
6 6 6

© 2021, BYJU'S. All rights reserved


Illustration : Find period of 𝑓 𝑥 = sec 2 𝑥 + cosec 3 𝑥

Solution:

𝑓 𝑥 = sec 2 𝑥 + cosec 3 𝑥
1 1
= +
cos2 𝑥 sin3 𝑥

Here degree of cos 𝑥 is even. So, Period of cos 2 𝑥 is 𝜋


Degree of sin 𝑥 is odd. So, Period of sin3 𝑥 is 2𝜋
Period of 𝑓 𝑥 = L.C.M. 𝜋, 2𝜋
Period = 2𝜋

© 2021, BYJU'S. All rights reserved


sin2 𝑥 cos2 𝑥
Illustration : Find period of 𝑓 𝑥 = 1 − −
1 + cot 𝑥 1 + tan 𝑥
Solution:
sin3 𝑥 cos3 𝑥
𝑓 𝑥 =1− −
sin 𝑥 +cos 𝑥 cos 𝑥 +sin 𝑥

(sin3 𝑥+cos3 𝑥)
=1 −
𝑠𝑖𝑛𝑥+𝑐𝑜𝑠𝑥

(sin 𝑥+cos 𝑥)(sin2 𝑥+cos2 𝑥−sin 𝑥 cos 𝑥)


=1 −
(sin 𝑥+cos 𝑥)

= 1 − 1 + sin 𝑥 cos 𝑥
1
= sin 2𝑥
2
Degree of sin 2𝑥 is odd and coefficient of 𝑥 is 2 so Period of 𝑓 𝑥 is 𝜋

© 2021, BYJU'S. All rights reserved


Illustration : Find period of 𝑓 𝑥 = sin 2𝑥 + cos 3𝑥 + tan 5𝑥

Solution:
𝑓 𝑥 = sin 2𝑥 + cos 3𝑥 + tan 5𝑥
2𝜋
Period of sin 2𝑥 = =𝜋
2

2𝜋
Period of cos 3𝑥 =
3
𝜋
Period of tan 5𝑥 =
5

Period of 𝑓 𝑥 is given by
2𝜋 2𝜋 𝜋 L.C.M. 2𝜋, 2𝜋, 𝜋
𝑇 = L.C.M. , , = 𝑇 = 2𝜋
2 3 5 H.C.F. (2, 3, 5)
© 2021, BYJU'S. All rights reserved
Illustration : Find period of 𝑓 𝑥 = sin 𝜋𝑥 + {𝑥}.

Solution :
𝑓 𝑥 = sin 𝜋𝑥 + {𝑥}
2𝜋
Period of sin 𝜋𝑥 = =2
𝜋

Period of {𝑥} = 1

Period of 𝑓 𝑥 is given by

𝑇 = L.C.M. 2, 1

𝑇 =2
© 2021, BYJU'S. All rights reserved
Illustration : Find period of 𝑓 𝑥 = sin 3𝑥 + 𝑥 .

Solution :
2𝜋
Period of sin 3𝑥 =
3
Period of {𝑥} = 1
Period of 𝑓 𝑥 is given by
2𝜋
𝑇 = L.C.M. ,1
3
Also, L.C.M of an irrational number and a rational number is not defined.
⇒ 𝑓 𝑥 = sin 3𝑥 + 𝑥 is not periodic

© 2021, BYJU'S. All rights reserved


𝑥 𝑥
Illustration : Find period of 𝑓 𝑥 = sin2 4
+ cos 2
3
.

Solution :
1
Here degree of sine is even and coefficient of 𝑥 is
4
𝑥 𝜋
So, Period of sin2 = 1 = 4𝜋
4
4
1
Here degree of cosine is even and coefficient of 𝑥 is
3
𝑥 𝜋
So, Period of cos 2 = 1 = 3𝜋
3
3
Period of 𝑓 𝑥 is given by

𝑇 = L.C.M. 4𝜋, 3𝜋 ⟹ 𝑇 = 12𝜋

© 2021, BYJU'S. All rights reserved


Illustration : Find the period of 𝑓 𝑥 = 2 5𝜋{𝑥} + tan 𝜋 𝑥 , where [.]
represents G.I.F

Solution:

𝑓 𝑥 =2 5𝜋{𝑥} + tan 𝜋[𝑥]


tan 𝜋 𝑥 = tan 𝑛𝜋 = 0, 𝑛 ∈ ℤ, 𝑥 = 𝑛
Period of {𝑥} = 1
⇒ Period of 𝑓 𝑥 is 1
So, 𝑇 = 1

© 2021, BYJU'S. All rights reserved


sin 𝜋 𝑥
Illustration : Find the period of 𝑓 𝑥 = 𝑥
𝑌
Solution: 𝑓 𝑥 = sin 𝜋 𝑥 sin 𝜋 𝑥
𝑥 𝑓 𝑥 =
𝑥
Here 𝑥 ∈ 0,1 , 𝑥 ≠ 𝕀 −2 −1 0 1 2
𝑋

sin 𝜋 𝑥 = sin 𝑛𝜋 = 0 ∵ 𝑥 always gives integer value


𝑥 = 𝑛, 𝑛 ∈ 𝕀
⇒𝑓 𝑥 =0

Domain of 𝑥 is 𝑥 ∈ ℝ − 𝕀. (∵ 𝑥 = 0 for 𝑥 = 𝕀)

Domain of 𝑓 𝑥 ∈ ℝ − 𝕀.

So, period of 𝑓 𝑥 is 1.

© 2021, BYJU'S. All rights reserved


Illustration : Find the period of 𝑓 𝑥 = sin 𝑥 0
𝑌
𝑓 𝑥 = sin 𝑥 0
Solution: 𝑌=1

0 𝑋
𝑓 𝑥 = sin 𝑥 −2𝜋 −𝜋 0 𝜋 2𝜋

Here Domain of 𝑓 𝑥 is 𝑥 ∈ ℝ − 𝑛𝜋 .

For 𝑥 = 𝑛𝜋, 𝑓 𝑥 is of the form 00 (indeterminant form)

⇒ 𝑓 𝑥 = 1 ∀ 𝑥 ∈ ℝ − 𝑛𝜋 .
So, period of 𝑓 𝑥 is 𝜋.

© 2021, BYJU'S. All rights reserved


1 1
Illustration : 𝑓 𝑥 = 𝑥 + + 𝑥 − + 2 −𝑥 , then find period of 𝑓(𝑥).
2 2

Solution: 𝑓 𝑥 = 𝑥 + 12 + 𝑥 − 12 + 2 −𝑥
1 1 1 1
⇒ 𝑓 𝑥 = 𝑥 + 2 − 𝑥 + 2 + 𝑥 − 2 − 𝑥 − 2 + 2 −𝑥 − −𝑥
(∴ 𝑥 = 𝑥 + 𝑥 ⇒ 𝑥 = 𝑥 − 𝑥 )
1 1
⇒ 𝑓 𝑥 = 2𝑥 − 𝑥 + − 𝑥− − 2𝑥 − 2 −𝑥
2 2

1 1
⇒ 𝑓 𝑥 = − 𝑥 + 2 − 𝑥 − 2 − 2 −𝑥

𝑇1 = 1 𝑇2 = 1 1
𝑇3 = =1
−1

⇒ 𝑇 = L.C.M. 1, 1, 1 = 1

© 2021, BYJU'S. All rights reserved


1 2
Illustration : Find period of 𝑓 𝑥 = 𝑥 + 𝑥 + 3 + 𝑥 + 3 − 3 𝑥 + 10
Solution:
1 2 𝑛−1
We know, 𝑥 + 𝑥 + 𝑛 + 𝑥 + 𝑛 + ⋯+ 𝑥 + = 𝑛𝑥
𝑛

1 2
⇒ 𝑥 + 𝑥 + 3 + 𝑥 + 3 = 3𝑥

𝑓 𝑥 = 3𝑥 − 3 𝑥 + 10

= 3𝑥 − 3𝑥 − 3𝑥 + 3 𝑥 + 10

= − 3𝑥 + 3 𝑥 + 10

1
𝑇1 = 3 𝑇2 = 1
1
⇒ 𝑇 = L.C.M. ,1 =1
3
© 2021, BYJU'S. All rights reserved
When period of the trigonometric functions can not be determined directly

Then, apply following method :


𝜋 𝜋
➢ Step I : 𝑓 2
+ 𝑥 = 𝑓 𝑥 , If satisfies then period of 𝑓(𝑥) is 𝑇 = .
2

➢ Step II : 𝑓 𝜋 + 𝑥 = 𝑓 𝑥 , If satisfies then period of 𝑓(𝑥) is 𝑇 = 𝜋.

➢ If step I and step II does not satisfy then period of 𝑓(𝑥) is 𝑇 = 2𝜋.

Note : This method is applicable only for trigonometric functions.


If Step I dose not satisfy then only check step II

© 2021, BYJU'S. All rights reserved


Illustration : Find period of 𝑓 𝑥 = cos(cos 𝑥) + cos sin 𝑥

Solution:
Now, By using verification method,
𝜋 𝜋 𝜋
𝑓 + 𝑥 = cos cos +𝑥 + cos sin +𝑥
2 2 2

= cos − sin 𝑥 + cos cos 𝑥

= cos sin 𝑥 + cos cos 𝑥

=𝑓 𝑥
𝜋
So, period is .
2
© 2021, BYJU'S. All rights reserved
sin 𝑥 +| cos 𝑥|
Illustration : Find period of 𝑓 𝑥 = | sin 𝑥−cos 𝑥|
Solution:
Now, By using verification method,
𝜋 cos 𝑥 + |−sin 𝑥| cos 𝑥 + | sin 𝑥 |
𝑓 +𝑥 = = ≠𝑓 𝑥
2 | cos 𝑥 + sin 𝑥| | cos 𝑥 + sin 𝑥 |
− sin 𝑥 + |−cos 𝑥|
𝑓 𝜋+𝑥 =
| −sin 𝑥 + cos 𝑥|

sin 𝑥 + | cos 𝑥| sin 𝑥 + | cos 𝑥|


= =
|− sin 𝑥−cos 𝑥 | | sin 𝑥−cos 𝑥 |

=𝑓 𝑥

So, period of 𝑓 𝑥 is 𝜋.
© 2021, BYJU'S. All rights reserved
sin 𝑛𝑥
Illustration : If 𝑓 𝑥 = 𝑥 is periodic and period is 4𝜋, then find 𝑛.
sin 𝑛

sin 𝑛𝑥
Solution: 𝑓 𝑥 = 𝑥
sin 𝑛

2𝜋
Period of sin 𝑛𝑥 is .
𝑛
𝑥 2𝜋
Period of sin is 1 = 2𝑛𝜋 .
𝑛
𝑛
2𝜋
Period of 𝑓 𝑥 = L.C.M. , 2𝑛𝜋
𝑛
2𝜋 L.C.M. 2𝜋, 2𝑛𝜋
𝑇 = L.C.M. , 2𝑛𝜋 =
𝑛 H.C.F. (𝑛,1)
= 2𝑛𝜋

⇒ 2𝑛𝜋 = 4𝜋 ⇒ 𝑛 = ±2
© 2021, BYJU'S. All rights reserved
Period of a composite function
𝑓 𝑔 𝑥

Both 𝑓 and 𝑔 are 𝑓 is periodic and 𝑓 is non periodic Both 𝑓 and 𝑔 are
non periodic 𝑔 are non periodic and 𝑔 is periodic periodic
N.P(N.P) P(N.P) N.P(P) P(P)

Non periodic Non periodic Periodic Periodic


[ Period of inside
function i.e., 𝑔 𝑥 ]

Note : N.P means non periodic


P means periodic
© 2021, BYJU'S. All rights reserved
Illustration : Find period of 𝑓 𝑥 = sin 3𝑥 + cos 6𝑥

Solution: 𝑓 𝑥 = sin 3𝑥 + cos 6𝑥

For sin 3𝑥 : For cos 6𝑥 :


𝑔 𝑥 = . (greatest integer function) 𝑔 𝑥 = . (greatest integer function) is
is non periodic function non periodic function

ℎ 𝑥 = sin 3𝑥 is periodic function ℎ 𝑥 = cos 6𝑥 is periodic function


𝑔 ℎ 𝑥 = sin 3𝑥 = N.P(P) 𝑔 ℎ 𝑥 = cos 6𝑥 = N.P(P)

© 2021, BYJU'S. All rights reserved


2𝜋
Period of sin 3𝑥 =
3
2𝜋
Period of cos 6𝑥 =
6

2𝜋 2𝜋
Period of 𝑓 𝑥 = L.C.M. ,
3 6

L.C.M. 2𝜋, 2𝜋 2𝜋
= ⟹ Period of 𝑓 𝑥 =
H.C.F. (3,6) 3

© 2021, BYJU'S. All rights reserved


Illustration : Find period of 𝑓 𝑥 = sin(cos 𝑥)
Solution: 𝑓 𝑥 = sin(cos 𝑥)
For sin(cos 𝑥) :
𝑔 𝑥 = sin 𝑥 is periodic function
ℎ 𝑥 = cos 𝑥 is periodic function
𝑔 ℎ 𝑥 = sin(cos 𝑥) = P(P), so sin(cos 𝑥) is periodic
Now, by using verification method
𝜋
𝑓 + 𝑥 = sin − sin 𝑥 = −sin sin 𝑥 ≠ 𝑓 𝑥
2
𝑓 𝜋 + 𝑥 = sin − cos 𝑥 = −sin cos 𝑥 ≠ 𝑓 𝑥
𝑓 2𝜋 + 𝑥 = sin cos 𝑥 = sin cos 𝑥 = 𝑓 𝑥
So, period of 𝑓 𝑥 is 2𝜋.
© 2021, BYJU'S. All rights reserved
Illustration : Find period of 𝑓 𝑥 = sin 𝑥

Solution :
𝑓 𝑥 = sin 𝑥

P(N.P) gives N.P


For sin 𝑥 :
𝑔 𝑥 = sin 𝑥 is periodic function
ℎ 𝑥 = 𝑥 is non periodic function

𝑓 𝑥 = 𝑔 ℎ 𝑥 = sin 𝑥 = P(N.P)

So, sin 𝑥 is non periodic function


© 2021, BYJU'S. All rights reserved
Illustration : Find period of 𝑓 𝑥 = cos 𝑥 2 .

Solution :
𝑓 𝑥 = cos 𝑥 2

P(N.P) gives N.P


For cos 𝑥 2 :
𝑔 𝑥 = 𝑐𝑜𝑠𝑥 is periodic function
ℎ 𝑥 = 𝑥 2 is non periodic function

𝑓 𝑥 =𝑔 ℎ 𝑥 = cos 𝑥 2 = P(N.P)

So, cos 𝑥 2 is non periodic function


© 2021, BYJU'S. All rights reserved
Illustration : Find period of 𝑓 𝑥 = log cos 3𝑥

Solution :
𝑓 𝑥 = log cos 3𝑥

N.P(P) gives P
For log cos 3𝑥 :
𝑔 𝑥 = 𝑙𝑜𝑔𝑥 is non periodic function
ℎ 𝑥 = cos 3𝑥 is periodic function
𝑓 𝑥 =𝑔 ℎ 𝑥 = log cos 3𝑥 = N.P(P)
So, period of 𝑓 𝑥 = period of ℎ(𝑥)
2𝜋
𝑇=
3
© 2021, BYJU'S. All rights reserved
Illustration : Find period of 𝑓 𝑥 = sin 𝑥
Solution : 𝑓 𝑥 = sin 𝑥

P(P) gives P
For sin 𝑥 :
𝑔 𝑥 = 𝑠𝑖𝑛𝑥 is periodic function
ℎ 𝑥 = 𝑥 is periodic function
𝑓 𝑥 =𝑔 ℎ 𝑥 = sin 𝑥 = P(P)
Here 𝑥 is not trigonometric function
So, Period of 𝑓 𝑥 = period of 𝑥
𝑇=1

© 2021, BYJU'S. All rights reserved


One-One
(Injective)

Many-One
4 kinds of
Functions
Onto
(Surjective)

Into

© 2021, BYJU'S. All rights reserved


Notes :

A function can be of one of these four types :

❑ One-one, onto (injective and surjective ) also called as Bijective functions.

❑ One – one, into (injective but not surjective)

❑ Many – one, onto (surjective but not injective)

❑ Many – one, into (neither surjective nor injective)

© 2021, BYJU'S. All rights reserved


One – one function (Injective function/ Injective mapping) :

A function f: A → B is said to be a one-one function if different


elements of set A have different f images in set B

1 𝑎 𝑎
1
𝑏
2 𝑏 2 𝑐
𝑑
3 𝑐 3
𝑒

Here you can see that different elements of set 𝐴 have different images in 𝐵

© 2021, BYJU'S. All rights reserved


Many – one function :

A function f: A → B is said to be a many-one function if there exist at least two


or more elements of set A that have same f images in set B

1 𝑎 𝑎
1
𝑏
2 2 𝑐
𝑏
𝑑
3 3

In 1st figure , elements 1 & 2 of set 𝐴 have same image in set 𝐵


so the function is many-one.

© 2021, BYJU'S. All rights reserved


Onto function (surjective mapping):
If the function 𝑓: 𝐴 → 𝐵 is such that each element in 𝐵 (co-domain) has at
least one pre-image in 𝐴, then we say that 𝑓 is an‘onto’ function.

➢ Or, if range of 𝑓 = Co – domain of 𝑓 . 𝐴 𝐵 𝐴 𝐵


1 𝑎 1 𝑎
2 𝑏 2 𝑏
➢ 𝑓 ∶ 𝐴 → 𝐵 is surjective iff ∀ 𝑏 ∈ 𝐵, 3 𝑐 3 𝑐
4 𝑑
there exists some 𝑎 ∈ 𝐴 such that 4
𝑓 𝑎 = 𝑏. 𝑑 Onto
One to one, Many to one, Onto

➢ If co-domain is not given, then co-domain of function can


be taken as 𝑅.

© 2021, BYJU'S. All rights reserved


Into function:
If the function 𝑓: 𝐴 → 𝐵 is such that there exists at least one element in 𝐵 (co-domain)
which is not the image of any element in domain (𝐴), then 𝑓 is an ‘into’ function.

➢ Or, if range of 𝑓 ≠ Co – domain of 𝑓 .

𝐴 𝐵 𝐴 𝐵
1 𝑎 1 𝑎
𝑏 2 𝑏
2 𝑐 3 𝑐
3 𝑑 4 𝑑

One to one, Into Many to one, Into


If a function is onto, then it can not be into and vice – versa.
If function is one- one, then it can not be many-one and vice-versa
© 2021, BYJU'S. All rights reserved
Illustration : Identify the nature of the function 𝑓: 𝑁 → 𝑁 such that 𝑓 𝑥 = 𝑥 + 1.
Solution: 𝑓 𝑥 =𝑥+1
𝐴 𝐵
1 1 Here, element 1 in set 𝐵 has no pre-image in set 𝐴
2 2
3 3 So,range of 𝑓 ≠ Co – domain of 𝑓 ⇒ 𝑓 is Into
4 4
5
Different elements of set A have different images in set B
⋮ ⋮ So, function is one-one

⋮ ⋮ Hence, function is One-one, Into

∞ ∞

𝑁 𝑁
© 2021, BYJU'S. All rights reserved
Illustration : Identify the nature of the function 𝑓: 𝑍 → 𝑍 such that 𝑓 𝑥 = 𝑥 2 − 𝑥.
𝑓 0 =0 𝑓 1 =0 𝑓 2 =2
Solution: 𝐴 𝐵
−∞
For 𝑓 𝑥 = 1 ⇒ 𝑥 2 − 𝑥 = 1
−∞
⇒ 𝑥2 − 𝑥 − 1 = 0
⋮ ⋮
1± 1+4
−2 −2 ⇒ 𝑥2 =
2
−1 −1
1± 5
0 0 ⇒𝑥= ∉𝑧
2
1 1
2 2 So, element 1 in set 𝐵 has no pre-image in set 𝐴
range of 𝑓 ≠ Co – domain of 𝑓 ⇒ 𝑓 is Into
⋮ ⋮
Elements 0 & 1 of set A have same image in set B
∞ ∞ So, function is many-one

𝑍 𝑍 Hence, Function is Many-one, Into


© 2021, BYJU'S. All rights reserved
Illustration : Identify the nature of the function 𝑓: 𝑁 → 𝐼 such that
𝑛−1
𝑛 = odd
2
𝑓 𝑥 = −𝑛
𝑛 = even 𝑛=1 𝑓 𝑥 =0
Solution: 2
𝐴 𝐵 𝑛 = 2 𝑓 𝑥 = −1
1 −∞ 𝑛=3 𝑓 𝑥 =1
2 ⋮
−2 𝑛=4 𝑓 𝑥 = −2
3
4 −1 𝑛=5 𝑓 𝑥 =2
5 0
⋮ 1 So, range of 𝑓 = Co – domain of 𝑓 ⇒ 𝑓 is Onto
2
Different elements of set A have different
⋮ ⋮ image in set B so function is one-one.

∞ ∞ Function is One-one, Onto


© 2021, BYJU'S. All rights reserved 𝑁 𝐼
Summary Sheet
❖ When 2 or more functions are given like ℎ 𝑥 = 𝑓 𝑥 ± 𝑔 𝑥 or
ℎ 𝑥 = 𝑓 𝑥 . 𝑔 𝑥 or combination of these ,then period of ℎ 𝑥 is
𝑇 = L.C.M. 𝑇1 , 𝑇2 , where 𝑇1 is the period of 𝑓(x) and 𝑇2 is the
period of 𝑔(𝑥). 𝑇 need not be fundamental period.
❖ LCM(𝑇1 , 𝑇2 ) is only possible when 𝑇1 and 𝑇2 are of same type
❖ When period of the trigonometric functions can not be determined directly
Then, apply following method :
𝜋 𝜋
➢ Step I : 𝑓 2
+ 𝑥 = 𝑓 𝑥 , If satisfies then period of 𝑓(𝑥) is 𝑇 = .
2

➢ Step II : 𝑓 𝜋 + 𝑥 = 𝑓 𝑥 , If satisfies then period of 𝑓(𝑥) is 𝑇 = 𝜋.


➢ If step I and step II does not satisfy then period of 𝑓(𝑥) is 𝑇 = 2𝜋.
© 2021, BYJU'S. All rights reserved
Summary Sheet
For composite function (fog(x) type)

𝑓 𝑔 𝑥

N.P(N.P) P(N.P) N.P(P) P(P)

Non periodic Non periodic Periodic Periodic


[ Period of inside
function i.e., 𝑔 𝑥 ]

© 2021, BYJU'S. All rights reserved


Summary Sheet
One – one function (Injective function/ Injective mapping) :
A function f: A → B is said to be a one-one function if different
elements of set A have different f images in set B
Many – one function :
A function f: A → B is said to be a many-one function if there exist at least two
or more elements of set A that have same f images in set B
Onto function (surjective mapping):
If range of 𝑓 = Co – domain of 𝑓 then 𝑓(𝑥) is said to be Onto function .
Into function:
If range of 𝑓 ≠ Co – domain of 𝑓 then 𝑓(𝑥) is said to be Into function .
© 2021, BYJU'S. All rights reserved
Relations and Functions
Methods of finding nature of Functions

© 2021, BYJU'S. All rights reserved


Road Map

Number of Functions

Horizontal line Test

Methods to determine whether a


function is one - one or not

© 2021, BYJU'S. All rights reserved


Methods to determine whether a function is ONE-ONE or NOT:

By Graph: Use only when graph of function is known


Horizontal line test: If we draw a straight line parallel to 𝑋- axis , and it cuts the
graph of the function at exactly one point, then the function is ONE – ONE.

Let us consider a function 𝑓: A → B.

Then check the Range and Co-domain of 𝑓.

If Range < Co-domain ⇒ 𝑓 is Into


If Range = Co-domain ⇒ 𝑓 is Onto

© 2021, BYJU'S. All rights reserved


Graphical Test :

Let us consider two functions 𝑦 = 3𝑥 + 5 and 𝑦 = 𝑥 3 . A straight line parallel to the


X −axis cuts the graph of these functions at one point only. Hence the function is
ONE - ONE.

𝑌 𝑦 = 3𝑥 + 5 𝑌 𝑦 = 𝑥3

0
𝑋 0 𝑋
ONE - ONE ONE - ONE

© 2021, BYJU'S. All rights reserved


Example: Identify whether the given function is (i) one-one/many-one
(ii) onto/into
𝜋 𝜋
𝑖) 𝑓: − , → −1,1 , 𝑓 𝑥 = sin 𝑥 𝑖𝑖) 𝑓: 0, 𝜋 → −1,1 , 𝑓 𝑥 = sin 𝑥
2 2
𝑌 𝑦 = sin 𝑥 𝑌
𝑦 = sin 𝑥

𝜋 0 𝜋 𝑋
− 0 𝜋 𝑋
2 2
From the graph, a straight line parallel
From the graph, a straight line parallel to to the X −axis, cuts the graph at two
the X −axis, cuts the graph at one point points ⇒ 𝑓 𝑥 is many-one.
only ⇒ 𝑓 𝑥 is one-one.
Range : 0,1 Co-domain : −1,1
Range : −1,1 Co-domain : −1,1
Range ≠ Co-domain⇒Into function
Range = Co-domain ⇒ Onto function
© 2021, BYJU'S. All rights reserved
Find the nature of function 𝑓 : 0, ∞ → (0, ∞), such that
𝑖) 𝑓 𝑥 = 𝑥 2 𝑖𝑖) 𝑓 𝑥 = 4𝑥
Solution:
𝑌 𝑦 = 𝑥2
𝑌 𝑦 = 𝑎𝑥 ; 𝑎 > 0

1
0
𝑋 0
𝑋
From the graph, a straight line parallel From the graph, a straight line parallel to
to the X −axis, cuts the graph at one the X −axis, cuts the graph at one point
point only ⇒ 𝑓 𝑥 is one-one. only ⇒ 𝑓 𝑥 is one-one.
Range : (0, ∞) Co-domain : 0, ∞ Range : (1, ∞) Co-domain : 0, ∞
Range = Co-domain ⇒ Onto function Range ≠ Co-domain ⇒ Into function
© 2021, BYJU'S. All rights reserved
Methods to determine whether a function is ONE-ONE or NOT:

By derivative method
𝑑𝑦 𝑑𝑦
If > 0 or < 0 for its entire domain, then 𝑓 𝑥 is one-one Monotonous
𝑑𝑥 𝑑𝑥

𝑑𝑦 𝑑𝑦
If > 0 or < 0 under few conditions, then 𝑓 𝑥 is many-one for
𝑑𝑥 𝑑𝑥
its entire domain . Non-Monotonous

In other words, any function which is either monotonically increasing or


decreasing in it’s whole domain is one-one, otherwise many-one.

© 2021, BYJU'S. All rights reserved


Methods to determine whether a function is ONE-ONE or MANY-ONE:

𝑌 𝑓′ 𝑥 > 0 𝑌 𝑓′ 𝑥 < 0 𝑌 𝑓′ 𝑥 > 0


and
𝑓′ 𝑥 < 0
in [𝑥1 , 𝑥2 ]

𝑋
0 𝑥1 𝑥2 0 𝑥1 𝑥2 𝑋 0 𝑥1 𝑥2 𝑋
Monotonous (Increasing) Monotonous (Decreasing) Non-Monotonous

Note:
If Co-domain of function is not given, assume it as ℝ.
© 2021, BYJU'S. All rights reserved
Find the nature of following functions

𝑖)𝑓 𝑥 = 𝑥 3 + 𝑥 , 𝑓: ℝ → ℝ 𝑖𝑖) 𝑓 𝑥 = 𝑥 3 − 𝑥 , 𝑓: ℝ → ℝ

𝑓 ′ (𝑥) = 3𝑥 2 + 1 ≥ 1 ( Always) 𝑓 ′ 𝑥 = 3𝑥 2 − 1 ≥ −1

Hence, 𝑓 𝑥 is a ONE-ONE function ⇒ 𝑓 ′ 𝑥 > 0 (Not sure)


𝑓 𝑥 → odd degree polynomial Hence, 𝑓 𝑥 is a MANY-ONE function

Range :ℝ 𝑓 𝑥 → odd degree polynomial


Co-domain :ℝ
⇒ Range = Co-domain = ℝ
Range = Co-domain
⇒ 𝑓 is an onto function
⇒ 𝑓 is an onto function
© 2021, BYJU'S. All rights reserved
𝑖𝑖𝑖)𝑓 𝑥 = 2𝑥 + sin 𝑥

𝑓 ′ (𝑥) = 2 + cos 𝑥 ≥ 1 (∵ cos 𝑥 ∈ [−1,1])


Hence, 𝑓 𝑥 is a ONE-ONE function

Range = Co-domain = ℝ

⇒ 𝑓 is an onto function

© 2021, BYJU'S. All rights reserved


𝑣) 𝑓 𝑥 = 𝑥 3 + 𝑥 2 + 𝑥 + 1
𝑖𝑣)𝑓 𝑥 = sin 𝑥 − 5𝑥
𝑓 ′ 𝑥 = 3𝑥 2 + 2𝑥 + 1
𝑓 ′ 𝑥 = cos 𝑥 − 5 < 0
Discriminant of the above quadratic is
∵ cos 𝑥 ∈ −1,1
𝐷 = 4 − 12 < 0 also 𝑎 = 3 > 0
Hence 𝑓(𝑥) is always decreasing function.
⇒ 𝑓′(𝑥) > 0
Hence, 𝑓 𝑥 is a ONE-ONE function Hence, 𝑓(𝑥) is always increasing function.
Also, Range = Co-domain = ℝ Hence, 𝑓 𝑥 is a ONE-ONE function
Hence, 𝑓 𝑥 is a Onto function
Range = Co-domain = ℝ
Hence, 𝑓 𝑥 is a Onto function
© 2021, BYJU'S. All rights reserved
Identify the nature of the function 𝑓: −∞, −1 → 0, 𝑒 5 such that
3
𝑓 𝑥 = 𝑒 𝑥 −3𝑥+2
3 −3𝑥+2
Solution: Given, 𝑓 𝑥 = 𝑒 𝑥
3 −3𝑥+2
⇒ 𝑓′(𝑥) = 𝑒 𝑥 . (3𝑥 2 − 3) + − +
3 −3𝑥+2
⇒ 𝑓′ 𝑥 = 𝑒 𝑥 . 3(𝑥 − 1)(𝑥 + 1) −1 1
⇒ 𝑓 ′ 𝑥 > 0 in −∞, −1
Hence, function 𝑓(𝑥) is increasing throughout it’s domain
⇒ 𝑓 𝑥 is a one-one function.
𝑓 −1 = 𝑒 −1 3 +3+2 = 𝑒 4 , Also, 𝑥 → −∞, 𝑓(𝑥) →0
⇒ Range of 𝑓(𝑥) is (0, 𝑒 4 ] and Co-domain of 𝑓(𝑥) is 0, 𝑒 5
⇒ 𝑓 𝑥 is an into function
© 2021, BYJU'S. All rights reserved
Methods to determine whether a function is ONE-ONE or NOT:
Using basic definition :
Let f: A → 𝐵 be a function.
For 𝑥1 , 𝑥2 ∈ 𝐴 and 𝑓 𝑥1 , 𝑓 𝑥2 ∈ 𝐵
𝑓 𝑥1 = 𝑓 𝑥2 ⇔ 𝑥1 = 𝑥2 or 𝑥1 ≠ 𝑥2 ⇔ 𝑓 𝑥1 ≠ 𝑓 𝑥2

Example: (i) A function 𝑓: 𝑅 → 𝑅 such that 𝑓 𝑥 = 3𝑥 + 5


𝑓 𝑥1 = 3𝑥1 + 5 𝑓 𝑥2 = 3𝑥2 + 5
𝑓 𝑥1 = 𝑓 𝑥2 ⇒ 3𝑥1 + 5 = 3𝑥2 + 5
⇒ 𝑥1 = 𝑥2 ⇒ Function is one to one.
(ii) A function 𝑓: ℝ → ℝ such that 𝑓 𝑥 = 𝑥 2
𝑓 𝑥1 = 𝑥12 𝑓 𝑥2 = 𝑥22
𝑓 𝑥1 = 𝑓 𝑥2 ⇒ 𝑥12 = 𝑥22
⇒ 𝑥1 = ±𝑥2 ⇒ Function is many to one
© 2021, BYJU'S. All rights reserved
Find the nature of function 𝑓 : ℝ − 3 → ℝ − {1}, such that
𝑥−1
𝑓 𝑥 =
𝑥−3
Solution:
We know,
Let 𝑓 𝑥1 = 𝑓 𝑥2
𝑎𝑥−𝑏 𝑎
𝑥1 − 1 𝑥2 − 1 Range of 𝑓 𝑥 = is ℝ −
𝑐𝑥−𝑑 𝑐
⇒ =
𝑥1 − 3 𝑥2 − 3 ⇒ Range of 𝑓(𝑥) :ℝ − {1)
⇒ 𝑥1 𝑥2 − 3𝑥1 − 𝑥2 + 3 = 𝑥1 𝑥2 − 3𝑥2 − 𝑥1 + 3 Co-domain of 𝑓(𝑥) :ℝ − 1 (Given)
⇒ 2𝑥1 = 2𝑥2 ⇒ 𝑥1 = 𝑥2 Hence, Range = Co-domain

∴ 𝑓 𝑥 is one-one function Hence, 𝑓 𝑥 is an onto function

© 2021, BYJU'S. All rights reserved


𝑥
Identify the nature of the function : 𝑓 𝑥 =
1 + 𝑥2
𝑥
Solution: 𝑓 𝑥 =
1 + 𝑥2
Let 𝑓 𝑥1 = 𝑓 𝑥2
𝑥1 𝑥2
⇒ 2 =
1 + 𝑥1 1 + 𝑥22

⇒ 𝑥1 + 𝑥1 𝑥22 = 𝑥2 + 𝑥12 𝑥2

⇒ 𝑥1 − 𝑥2 − 𝑥1 𝑥2 𝑥1 − 𝑥2 = 0

⇒ 𝑥1 − 𝑥2 1 − 𝑥1 𝑥2 = 0
1
⇒ 𝑥1 = 𝑥2 , 𝑥1 = ∴ 𝑓 𝑥 is many-one function
𝑥2
© 2021, BYJU'S. All rights reserved
𝑥
For range : Let 𝑦 = ⇒ 𝑦 + 𝑥 2𝑦 = 𝑥 ⇒ 𝑦 + 𝑥 2𝑦 − 𝑥 = 0
1+𝑥 2
For real value of 𝑥, discriminant of the above quadratic equation should be
greater than or equal to 0
i.e., 𝐷 ≥ 0 ⇒ 1 − 4𝑦 2 ≥ 0
⇒ 4𝑦 2 − 1 ≤ 0
⇒ 2𝑦 − 1 2𝑦 + 1 ≤ 0
1 1
⇒− ≤𝑦≤
2 2
1 1
Range of 𝑓(𝑥): − , Co-domain of 𝑓(𝑥) : 𝑅 ⇒ 𝑓(𝑥) is Into function
2 2

© 2021, BYJU'S. All rights reserved


Identify the following functions as ONE – ONE or MANY – ONE :
1
−1
𝑓 𝑥 = 1−𝑒 𝑥

Solution:
Let 𝑓 𝑥1 = 𝑓 𝑥2

1 1
⇒ 1−𝑒 𝑥1
−1
= 1−𝑒 𝑥2−1

Squaring on both the sides, we get,


1 1
1 1
−1 −1 𝑒 𝑥1 𝑒 𝑥2
⇒𝑒 𝑥1 =𝑒 𝑥2 ⇒ =
𝑒 𝑒
1 1
⇒ 𝑒 𝑥1 = 𝑒 𝑥2
⇒ 𝑥1 = 𝑥2 ⇒ 𝑓(𝑥) is one-one function
© 2021, BYJU'S. All rights reserved
Methods to determine whether a function is ONE-ONE or NOT:

Miscellaneous

➢ If 𝑓 𝑥 is even, then it is always many−one function. 𝑌 𝑦 = 𝑥2

For example 𝑦 = 𝑥 2 is an even function which is


symmetric about the 𝑌-axis hence a line parallel to
X −axis will cut this line at two points.
0 𝑋
➢ If 𝑓 𝑥 is periodic within domain, then it is always
many−one function.
➢ Hit and Trial.

Note: Range of even function can never be ℝ because its graph must take a turn
to replicate graph of R.H.S in L.H.S.
© 2021, BYJU'S. All rights reserved
Identify the nature of the function 𝑓: ℝ → ℝ such that
2𝑥 2 − 𝑥 + 5
𝑓 𝑥 = 2
7𝑥 + 2𝑥 + 10
2𝑥 2 − 𝑥 + 5
Solution: Given function, 𝑓 𝑥 = 2
7𝑥 + 2𝑥 + 10
Numerator of the g 𝑥 = 2𝑥 2 − 𝑥 + 5
Discriminant of the numerator = 1 − 4 × 2 × 5 < 0 ⇒ 𝑔 𝑥 = +ve
Denominator of the h 𝑥 = 7𝑥 2 + 2𝑥 + 10
Discriminant of Denominator = 4 − 4 × 7 × 10 < 0 ⇒ ℎ 𝑥 = +ve
⇒ 𝑓 𝑥 = +ve
⇒ Range ≠ ℝ (only positive)
⇒ Range ≠ Co-domain
⇒ 𝑓 𝑥 is an Into function
© 2021, BYJU'S. All rights reserved
In such problems, always substitute 𝑥 = 0 and proceed as follows.
5 1
𝑓 0 = =
10 2
1
Now substitute 𝑓 𝑥 =
2
1 2𝑥 2 − 𝑥 + 5
⇒ = 2
2 7𝑥 + 2𝑥 + 10
⇒ 7𝑥 2 + 2𝑥 + 10 = 4𝑥 2 − 2𝑥 + 10
4
⇒ 3𝑥 2 + 4𝑥 = 0 ⇒ 𝑥 = 0, −
3
⇒ 𝑓 𝑥 is a Many − One function

© 2021, BYJU'S. All rights reserved


If 𝑓: ℝ → 𝐵, 𝑓 𝑥 = 2 sin 𝑥 − 2 3 cos 𝑥 + 1 is onto, and the interval
of 𝐵 is 𝑎, 𝑏 . Find the value of 𝑏 − 𝑎.

Solution: Function is onto ⇒ Range = Co-domain

𝑓 𝑥 = 2 sin 𝑥 − 2 3 cos 𝑥 + 1

∵ 𝐴 sin 𝑥 + 𝐵 cos 𝑥 + 𝐶 ∈ 𝐶 − 𝐴2 + 𝐵2 , 𝐶 + 𝐴2 + 𝐵2

2 2
⇒ 𝑓(𝑥) ∈ 1 − 22 + 2 3 ,1 + 22 + 2 3
⇒ 𝑓 𝑥 ∈ −3, 5

Thus 𝐵 ∈ −3, 5
𝑏−𝑎 = 5+3=8
© 2021, BYJU'S. All rights reserved
𝑥2
If the function 𝑓: ℝ − −1 , 1 → 𝐴 , defined by 𝑓 𝑥 = is surjective ,
1−𝑥 2
then 𝐴 is equal to :

ℝ − −1 , 0 ℝ − (−1 , 0)

ℝ − {−1} 0 ,∞ JEE Main April 2019

Solution:
∴ 𝐴 = ℝ − −1 , 0
⇒ 𝑦 − 𝑦𝑥 2 = 𝑥 2
𝑦 Range of 𝑓 𝑥 = Co-domain of 𝑓 𝑥 = 𝐴
⇒ 𝑥2 = ∵ 𝑥2 > 0
1+𝑦


𝑦
≥0 Hence, option (𝑎) is the correct answer.
1+𝑦

⇒ 𝑦 ∈ −∞ , −1 ∪ 0 , ∞
© 2021, BYJU'S. All rights reserved
Permutation Based Number of Functions

One-One
(Injective) + Many-One

Total
Functions

Onto + Into
(Surjective)

© 2021, BYJU'S. All rights reserved


Permutation Based Number of Functions

Case 1 ∶ When both the sets 𝐴 and 𝐵 contain equal number of elements.
𝐴 𝐵
Let a function be 𝑓: 𝐴 → 𝐵, such that
1 𝑎
𝐴 = {1, 2, 3, 4} and 𝐵 = {𝑎, 𝑏, 𝑐, 𝑑}
2 𝑏
Here, 𝑛 𝐴 = 𝑛 𝐵 = 4 3 𝑐
4
Each element of 𝐴 can choose one element out of 4 𝑑
element of 𝐵

∴ Total number of functions = 4 × 4 × 4 × 4 = 44 = 256


In general if 𝑛 𝐴 = 𝑛 𝐵 = 𝑚.
Then total number of function from 𝐴 to 𝐵 = mm
© 2021, BYJU'S. All rights reserved
Now, for one-one functions,
𝐴 𝐵
Element 1 can choose one out of four elements of 𝐵
1 𝑎
2 can choose one out of remaining three elements of 𝐵 and so on 2 𝑏
∴ Number of one-one functions = 4 × 3 × 2 × 1= 4! = 24 3 𝑐
In general if 𝑛 𝐴 = 𝑛 𝐵 = 𝑚. 4 𝑑

Then total number of one-one function from 𝐴 to 𝐵 = m!


Number of Many-one functions = Number of function - Number of one-one function
= 256 − 24
Number of onto functions = 4 × 3 × 2 × 1 = 4! = 24
Number of into functions =Number of functions – Number of onto functions
= 256 − 24
© 2021, BYJU'S. All rights reserved
Permutation Based Number of Functions
Case 2: When number of elements in 𝐴 (domain) is more than 𝐵 (Co-domain).
𝐴 𝐵
Let a function be 𝑓: 𝐴 → 𝐵, such that
1 𝑎
𝐴 = {1, 2, 3, 4, 5} and 𝐵 = {𝑎, 𝑏, 𝑐, 𝑑} 2 𝑏
3
Here, 𝑛 𝐴 > 𝑛 𝐵 𝑐
4
Each element of 𝐴 can choose one element out of 5 5 𝑑
element of 𝐵

Total number of functions = 4 × 4 × 4 × 4 × 4 = 45

In general if 𝑛 𝐴 = 𝑛 and 𝑛 𝐵 = 𝑚.

Then total number of functions from 𝐴 to 𝐵 = mn


© 2021, BYJU'S. All rights reserved
Now, for one-one functions,

For one-one function each element of 𝐴 must be mapped to one element of 𝐵


𝐴 𝐵
Which is not possible here
1 𝑎
So, Number of one-one functions = 0 2 𝑏
3
𝑐
4
5 𝑑

Number of many-one functions = Number of function - Number of one-one function

= 45 −0
= 45

© 2021, BYJU'S. All rights reserved


Number of onto functions:

First divide 5 elements of 𝐴 into 4 groups and


divide this 4 into 4 candidates in 𝐵. 𝐴 𝐵
1 𝑎
𝐴 Division of 4 objects 2 𝑏
1 1 among 4 candidates 3
𝑐
2 4
=5 1
3 5! 1 5 𝑑
1 = × × 4!
4 1! .1! .1! .2! 3!
5
2 Formation of groups

Number of into functions = Total functions− number of onto functions


© 2021, BYJU'S. All rights reserved
Permutation Based Number of Functions

Case 3: When number of elements in 𝐴 (domain) is less than 𝐵 (Co-domain).


𝐴 𝐵
Let a function be 𝑓: 𝐴 → 𝐵, such that
1 𝑎
𝐴 = {1, 2, 3, 4, } and 𝐵 = {𝑎, 𝑏, 𝑐, 𝑑, 𝑒} 2 𝑏
Here, 𝑛 𝐴 < 𝑛 𝐵 3 𝑐
4 𝑑
Each element of 𝐴 can choose one element out of 4 𝑒
element of 𝐵
4
Total number of functions = 5 × 5 × 5 × 5 = 5
In general if 𝑛 𝐴 = 𝑛 and 𝑛 𝐵 = 𝑚.

Then total number of functions from 𝐴 to 𝐵 = mn


© 2021, BYJU'S. All rights reserved
Now, for one-one functions,
1 can choose one out of five elements of 𝐵
2 can choose one out of remaining four elements of 𝐵 and so on
Number of one-one functions = 5 × 4 × 3 × 2 = 120
Number of many-one functions =Number of functions – number of one one functions
= 54 − 120
Number of onto functions = 0 ( because 𝑛 𝐴 < 𝑛 𝐵 )
Number of into functions = Number of functions – Number of onto functions
= 54

© 2021, BYJU'S. All rights reserved


If 𝐴 = 1,2,3,4 , then the number of functions on set 𝐴,
which are not one – one , is :

240 248 232 256 JEE Main April 2020

Solution:
Number of many – one function
= (Total number of function) − (number of one – one function)

Total function : 44
One-one functions: 4 × 3 × 2 × 1 = 24
Number of many – one function = 256 − 24
= 232
© 2021, BYJU'S. All rights reserved
Summary Sheet
• Horizontal line test: If we draw a straight line parallel to the 𝑋 - axis and
cuts the graph of the function at exactly one point, then the function is a
one to one function.

• If range < codomain, then it is an into function.

• If range = codomain, then it is an onto function.

• If for function 𝑦 = 𝑓(𝑥), 𝑓′(𝑥) > 0 or 𝑓′(𝑥) < 0, it is a monotonous


function. If not, then it is a many to one function.

• If 𝑓(𝑥) is even, then it is always a many to one function.

© 2021, BYJU'S. All rights reserved


Summary Sheet
• If 𝑓(𝑥) is periodic within a domain, then it is always a many to one
function.

• If 𝑓: 𝐴 → 𝐵, is a one-one function, then maximum number of one-one


functions possible are 𝑛𝑃𝑚 (𝑛 ≥ 𝑚) where 𝑛 is the number of elements
in 𝐵 and 𝑚 is the number of elements in 𝐴.

• If 𝐴 and 𝐵 are two sets having 𝑚 and 𝑛 elements respectively such that
1 ≤ 𝑛 ≤ 𝑚, then number of onto functions from 𝐴 to 𝐵 is
𝑛
n−r
෍ −1 𝑛𝐶𝑟 ⋅ 𝑟 𝑚
𝑟=1

• Number of bijective functions = 𝑛!, where 𝑛 is the number of elemetns


in set 𝐴 and set 𝐵.
© 2021, BYJU'S. All rights reserved
Relations and Functions
Composite and Inverse Functions

© 2021, BYJU'S. All rights reserved


Road Map

Procedure to find inverse function

Inverse functions

Condition for existence of composite


function

Composite function
© 2021, BYJU'S. All rights reserved
Composite Functions :
Definition : Let 𝑓: 𝐴 → 𝐵 and 𝑔: 𝐵 → 𝐶 be two functions
then, 𝑔𝑜𝑓: 𝐴 → 𝐶 is defined by 𝑔𝑜𝑓 𝑥 = 𝑔 𝑓 𝑥

𝑓 𝑔 𝑔 𝑓 𝑥
𝑥 𝑓(𝑥) Here, we give 𝑥 as input to
function 𝑓 then output is 𝑓(𝑥)
𝑔𝑜𝑓 𝑥 = 𝑔 𝑓 𝑥 Now, 𝑓(𝑥) is input for function 𝑔
and then its output is 𝑔 𝑓 𝑥
Condition for existence of 𝑔𝑜𝑓 𝑥 :

𝑓 𝑥 must be subset of domain of 𝑔.

© 2021, BYJU'S. All rights reserved


Composite Functions :

➢ If 𝑓 𝑥 , 𝑔 𝑥 & ℎ 𝑥 are three functions and their composition of mapping is


defined, then, :

𝑓𝑜𝑔 𝑥 = 𝑓 𝑔 𝑥 𝑔𝑜𝑓 𝑥 = 𝑔 𝑓 𝑥
𝑓𝑜𝑓𝑜𝑓 𝑥 = 𝑓 𝑓 𝑓 𝑥
𝑔𝑜𝑔 𝑥 = 𝑔 𝑔 𝑥 𝑓𝑜𝑔𝑜ℎ 𝑥 = 𝑓 𝑔 ℎ 𝑥

➢ 𝑓+𝑔 𝑥 =𝑓 𝑥 +𝑔 𝑥

𝑓 𝑓 𝑥
➢ 𝑔
𝑥=
𝑔 𝑥
, 𝑔 𝑥 ≠0

© 2021, BYJU'S. All rights reserved


1−𝑥
Illustration : If 𝑓 𝑥 = , then find 𝑓 𝑓 sin 𝜃
1+𝑥

Solution:
1 − sin 𝜃
𝑓 sin 𝜃 =
1 + sin 𝜃

1 − 𝑓 (sin 𝜃)
𝑓 𝑓 sin 𝜃 =
1 + 𝑓 sin 𝜃

1 − sin 𝜃
1 −
= 1 + sin 𝜃
1 − sin 𝜃
1 +
1 + sin 𝜃

1 + sin 𝜃 − (1 − sin 𝜃)
=
1 + sin 𝜃 + (1 − sin 𝜃)

2 sin 𝜃
= ⇒ 𝑓 𝑓 sin 𝜃 = sin 𝜃
© 2021, BYJU'S. All rights reserved
2
1
Illustration : If 𝑓 𝑥 = 1 − 𝑥 , then find 𝑓𝑜𝑓𝑜𝑓 𝑥

1
Solution: 𝑓 𝑓 𝑥 =
1 − 𝑓(𝑥)
1 1−𝑥 𝑥−1
= = =
1 1−𝑥−1 𝑥
1 −1 − 𝑥

1
𝑓 𝑓 𝑓 𝑥 =
1 − 𝑓(𝑓 𝑥 )
1 𝑥
= = = 𝑥
𝑥−1 𝑥 − 𝑥 + 1
1 −
𝑥

⇒ 𝑓𝑜𝑓𝑜𝑓 𝑥 = 𝑥
© 2021, BYJU'S. All rights reserved
Illustration : If 𝑓 𝑥 = 𝑥 3 − 𝑥 & 𝑔 𝑥 = sin 𝑥 then find 𝑓𝑜𝑔 𝑥 and 𝑔𝑜𝑓 𝑥

Solution:
3
𝑓𝑜𝑔 𝑥 = 𝑓 𝑔 𝑥 = 𝑔 𝑥 −𝑔 𝑥

= sin3 𝑥 − sin 𝑥

𝑔𝑜𝑓 𝑥 =𝑔 𝑓 𝑥 = sin 𝑓 𝑥

= sin 𝑥 3 − 𝑥

© 2021, BYJU'S. All rights reserved


1
Illustration : If 𝑓 𝑥 = 𝑎 − 𝑥𝑛 𝑛 , then find 𝑓𝑜𝑓 𝑥

Solution:

𝑓𝑜𝑓 𝑥 = 𝑓 𝑓 𝑥
1
𝑛 𝑛
= 𝑎− 𝑓 𝑥
1
𝑛 𝑛
1
𝑛
= 𝑎− 𝑎− 𝑥 𝑛

1 1
= 𝑎−𝑎+ 𝑛
𝑥 𝑛 = 𝑛
𝑥 𝑛

=𝑥
© 2021, BYJU'S. All rights reserved
1
1 7
Illustration : If 𝑔 𝑥 = 4 cos 4 𝑥 − 2 cos 2𝑥 − cos 4𝑥 − 𝑥7 , then the
2
value of 𝑔 𝑔 100 is equal to.

−1 0 1 100

Solution:
1
1
Let, 𝑔 𝑥 = ℎ 𝑥 7
where ℎ 𝑥 = 4 cos 4 𝑥 − 2 cos 2𝑥 − 2 cos 4𝑥 − 𝑥 7
Now, we know that cos 2𝑥 = 2 cos2 𝑥 − 1 , cos 4𝑥 = 2 cos 2 2𝑥 − 1
1
⟹ ℎ 𝑥 = 4 cos 4 𝑥 − 2(2 cos2 𝑥 − 1) − 2 (2 cos 2 2𝑥 − 1) − 𝑥 7
1
⟹ ℎ 𝑥 = 4 cos 4 𝑥 − 4 cos 2 𝑥 + 2 − cos2 2𝑥 + − 𝑥 7
2
Now , cos 2𝑥 2 = 2 cos 2 𝑥 − 1 2 = 4 cos 4 𝑥 − 4 cos 2 𝑥 + 1

© 2021, BYJU'S. All rights reserved


1
= 4 cos4 𝑥 − 4 cos2 𝑥 + 2 − (4 cos 4 𝑥 − 4 cos 2 𝑥 + 1) + − 𝑥 7
2

3
= − 𝑥7
2
1
3 3 7
∴ ℎ 𝑥 = − 𝑥7 ∴ 𝑔 𝑥 = − 𝑥7
2 2
1
1 1
3 7 3 3 7
⇒𝑔 𝑔 𝑥 = − 𝑔 𝑥
7
= − − 𝑥7 = 𝑥7 7 =𝑥
2 2 2

⇒ 𝑔 𝑔 100 = 𝑥 = 100
Hence, option (𝑑)is the correct answer

© 2021, BYJU'S. All rights reserved


1−𝑥 2𝑥
Illustration : If 𝑓 𝑥 = log 𝑒 , 𝑥 <1, then 𝑓 is equal to :
1+𝑥 1+𝑥 2
2
2𝑓(𝑥) 𝑓 𝑥 2𝑓(𝑥 2 ) −2𝑓(𝑥)

Solution: JEE Main April 2019


1−𝑥 2𝑥
𝑓 𝑥 = log 𝑒 , Let 𝑔 𝑥 =
1+𝑥 1 + 𝑥2

1 − 𝑔(𝑥)
Then 𝑓 𝑔(𝑥) = log 𝑒
1 + 𝑔(𝑥)

2𝑥 2
1− 1−𝑥 2 1−𝑥
= log 𝑒 1 + 𝑥2 = log 𝑒 = log 𝑒
2𝑥 1+𝑥 2 1+𝑥
1+
1 + 𝑥2

1−𝑥
∴ 𝑓 𝑔(𝑥) = 2𝑙𝑜𝑔𝑒 1+𝑥
= 2𝑓 𝑥 Hence, option (𝑎) is the correct answer
© 2021, BYJU'S. All rights reserved
Illustration : If 𝑓 𝑔 𝑥 = 3𝑥 + 4 and 𝑔 𝑥 = 5𝑥 + 4. then find 𝑓 𝑥

Solution:
𝑓 𝑔 𝑥 = 3𝑥 + 4
𝑡 −4
𝑓 5𝑥 + 4 = 3𝑥 + 4 Let 5𝑥 + 4 = 𝑡 ⟹ 𝑥 = 5

𝑡−4
⇒𝑓 𝑡 =3 +4
5

3𝑡 12
⇒𝑓 𝑡 = − +4
5 5

3𝑡 8 3𝑥 + 8
⇒𝑓 𝑡 = + ⇒𝑓 𝑥 =
5 5 5
© 2021, BYJU'S. All rights reserved
Remark

➢ 𝑓: 𝐴 → 𝐵 and 𝑔: 𝐵 → 𝐶 then ➢ 𝑓: 𝐴 → 𝐵, 𝑔: 𝐶 → 𝐷 then

𝑓: 𝐴 → 𝐵 𝑔: 𝐶 → 𝐷
𝑔𝑜𝑓: 𝐴 → 𝐶 exists 𝑓𝑜𝑔: 𝐶 → 𝐵 will exist
𝑔: 𝐵 → 𝐶 𝑓: 𝐴 → 𝐵

Here, For any of the three following conditions


(Range of 𝑓 = 𝐵) ⊆ (Domain of 𝑔 = 𝐵)
1. 𝐷 must be matched to A
So, 𝑔𝑜𝑓: 𝐴 → 𝐶 exists or
2. D Should be a subset of 𝐴
or
3. Range of 𝑔 ⊂ Domain of 𝑓

© 2021, BYJU'S. All rights reserved


Illustration :
3𝑥
If 𝑓: 𝑁 → ℝ ; 𝑓 𝑥 = 2𝑥 + 1 & 𝑔: 𝑍 → ℝ, 𝑔 𝑥 = then find 𝑔𝑜𝑓 𝑥
𝑥 2 +1
Solution:
In the first look, 𝑔𝑜𝑓 is not possible as codomain of 𝑓 doesn’t match
the domain of 𝑔
Let’s check its existence
We will check another condition
i.e (Range of 𝑓 ⊆ Domain of 𝑔)
As Range of 𝑓 Is odd natural numbers ≥ 3 ⊂ Domain of 𝑔
Hence, 𝑔𝑜𝑓 𝑥 exist.
Now 𝑔𝑜𝑓 𝑥 = 𝑔 𝑓 𝑥
3 2𝑥+1 6𝑥+3
⇒ 𝑔 2𝑥 + 1 ⇒ ⇒
2𝑥+1 2 +1 4𝑥 2 +4𝑥+2
© 2021, BYJU'S. All rights reserved
Illustration :
If 𝑓: ℝ → ℝ; 𝑓 𝑥 = 𝑥 2 & 𝑔: 0, ∞ → ℝ, 𝑔 𝑥 = 2𝑥 + 1 then find 𝑔𝑜𝑓 𝑥
Solution:
In the first look, 𝑔𝑜𝑓 is not possible as codomain of 𝑓 𝑓 𝑥 = 𝑥2
doesn’t match the domain of 𝑔
Let’s check its existence
So, we will check other condition
i.e (Range of 𝑓 ⊆ Domain of 𝑔)
As 𝑅𝑓 = Range of 𝑥 2 = [0,∞) ⊆ domain of 𝑔 Range of 𝑓 𝑥 = 0, ∞

Hence, 𝑔𝑜𝑓 𝑥 exist


Now 𝑔𝑜𝑓 𝑥 = 𝑔 𝑓 𝑥

⇒ 𝑔 𝑥 2 = 𝑔(𝑓 𝑥 ) = 2𝑥 2 + 1
© 2021, BYJU'S. All rights reserved
Illustration :
If 𝑓: ℝ → ℝ; 𝑓 𝑥 = 𝑥 2 & 𝑔: 0, ∞ → ℝ, 𝑔 𝑥 = 2𝑥 + 1 then find 𝑓𝑜𝑔 𝑥
Solution:
In the first look, 𝑔𝑜𝑓 is not possible as codomain of 𝑓 doesn’t match
the domain of 𝑔
Let’s check its existence
Let’s check another condition i.e.,Range of 𝑔 ⊆ Domain of 𝑓
Let’s find the range of 𝑔 𝑥
Domain of 𝑔 𝑥 = 0, ∞ and 𝑔 𝑥 is strictly increasing function
For 𝑥 = 0,
𝑓 𝑥 = 1 which is minimum value
For 𝑥 → ∞,
𝑓 𝑥 → ∞ which is maximum value
Hence range of 𝑔 𝑥 = 1, ∞
© 2021, BYJU'S. All rights reserved
𝑅𝑔 = Range of 2𝑥 + 1 = [1,∞) ⊂ domain of 𝑓 (𝑅)

Hence, 𝑓𝑜𝑔 𝑥 exist

⇒ 𝑓𝑜𝑔 𝑥 = 𝑓 𝑔 𝑥

⇒ 𝑓 2𝑥 + 1

⇒ 2𝑥 + 1 2

⇒𝑓 𝑔 𝑥 = 4𝑥 2 + 4𝑥 + 1

© 2021, BYJU'S. All rights reserved


Illustration : Let 𝑓 𝑥 = 𝑥 & 𝑔 𝑥 = 2 − 𝑥 , find the domain of
1 𝑓𝑜𝑔(𝑥) 2 𝑔𝑜𝑓 𝑥
Solution:
𝑓: 0, ∞ → 0, ∞ 𝑔: (−∞, 2]→ 0, ∞
𝑔 𝑥 = 2−𝑥
𝑓 𝑥 = 𝑥
2

Domain of 𝑓 𝑥 = 𝑥 Domain of 𝑔(𝑥) = 2 − 𝑥


𝑥 is in square root so, 𝑥 ≥ 0 2 − 𝑥 is in square root so, 2 − 𝑥 ≥ 0 ⇒ 𝑥 ≤ 2

Range of 𝑓 𝑥 = 𝑥 Range of 𝑔 𝑥 = 2 − 𝑥
𝑥 ≥ 0, so range is 0, ∞ 2 − 𝑥 ≥ 0, so range is 0, ∞

© 2021, BYJU'S. All rights reserved


1. For 𝑓𝑜𝑔 to exist
𝑔: (−∞, 2]→ 0, ∞
𝑓: 0, ∞ → 0, ∞
As 𝑅𝑔 = Range of 𝑥 2 = [0,∞) ⊆ domain of 𝑓 i. e. , [0, ∞)
Hence, 𝑓𝑜𝑔 exist
⇒ 𝑓𝑜𝑔:(−∞, 2] → 0, ∞
Domain of 𝑓𝑜𝑔 is (−∞, 2]
⇒ 𝐷𝑓𝑜𝑔 = (−∞, 2]

© 2021, BYJU'S. All rights reserved


2. For 𝑔𝑜𝑓 to exist
Only 0,2 is common
𝑓: 0, ∞ → 0, ∞
𝑔: (−∞, 2]→ 0, ∞

Here, Range of 𝑓 = 0, ∞ ⊄ Domain of 𝑔 (−∞, 2]


So, 𝑔𝑜𝑓 will be only defined in common portion 0, ∞ ⋂ −∞, 2 = [0, 2]
𝑓 𝑥 is increasing function. Range of 𝑓 𝑥 is 0, 2
To find Domain of 𝑓 𝑥 if 𝑓 𝑥 ∈ 0,2
When 𝑓 𝑥 = 0 ⇒ 𝑥 = 0 For 𝑥 = 0, 𝑓 𝑥 is minimum
When 𝑓 𝑥 = 2 ⇒ 𝑥 = 2 For 𝑥 = 4, 𝑓 𝑥 is maximum

⇒ 𝑔𝑜𝑓: 0 , 4 → 0, ∞ ⇒ 𝐷𝑔𝑜𝑓 = 0 , 4
© 2021, BYJU'S. All rights reserved
Illustration : Let 𝑓 𝑥 = 𝑥 & 𝑔 𝑥 = 2 − 𝑥 , find the domain of 𝑓𝑜𝑓

Solution:
3 For 𝑓𝑜𝑓 to exist

𝑓 ∶ 0, ∞ → 0, ∞
⇒ Perfectly matched
𝑓 ∶ 0, ∞ → 0, ∞

As 𝑅𝑓 = Range of 𝑓 = [0,∞) ⊆ domain of f = [0, ∞)


So, 3rd condition satisfied
Hence,𝑓𝑜𝑓 is exist
𝑓𝑜𝑓 ∶ 0, ∞ → 0, ∞
⇒ 𝐷𝑓𝑜𝑓 = 0, ∞
© 2021, BYJU'S. All rights reserved
Illustration : Let 𝑓 𝑥 = 𝑥 & 𝑔 𝑥 = 2 − 𝑥 , find the domain of 𝑔𝑜𝑔

Solution:
4 For 𝑔𝑜𝑔 to exist
𝑔 ∶ −∞, 2 → 0, ∞
𝑔 ∶ −∞, 2 → 0, ∞ ⇒ Only 0, 2 is common portion

Here, Range of 𝑔 𝑥 = 0, ∞ ⊄ Domain of 𝑔 𝑥 = (−∞, 2]


So, 𝑔𝑜𝑓 will be only defined for 0, ∞ ⋂ −∞, 2 = [0, 2]
For 𝑔 𝑥 = 2 − 𝑥 range should be 0, 2
For 𝑔 𝑥 = 0 ⇒ 2−𝑥 =0 ⇒𝑥 =2
For 𝑔 𝑥 = 2 ⇒ 2 − 𝑥 = 2 ⇒ 𝑥 = −2
𝑔𝑜𝑔 ∶ −2, 2 → 0, ∞ ⇒ 𝐷𝑔𝑜𝑔 = −2, 2
© 2021, BYJU'S. All rights reserved
Inverse of a function

Let 𝑓 ∶ 𝐴 → 𝐵 be one - one and onto function, then there exist a unique
function 𝑔 ∶ 𝐵 → 𝐴, such that 𝑓 𝑥 = 𝑦 ⇔ 𝑔 𝑦 = 𝑥 ∀ 𝑥 ∈ 𝐴, 𝑦 ∈ 𝐵.
Then 𝑔 is said to be inverse of 𝑓 and is represented as 𝑔 𝑥 = 𝑓 −1 (𝑥)

➢ Inverse of a Bijective function is also a Bijective function.

➢ Inverse of a Bijective function is unique.

© 2021, BYJU'S. All rights reserved


➢ If 𝑓 & g are two Bijective functions and 𝑔𝑜𝑓 exists, then 𝑔𝑜𝑓 will be Bijective
function and 𝑔𝑜𝑓 −1 = 𝑓 −1 𝑜𝑔−1

➢ 𝑓 ∶ 𝐴 → 𝐵 and 𝑓 −1 ∶ 𝐵 → 𝐴

𝑓 −1 ∶ 𝐵 → 𝐴
𝑓𝑜𝑓 −1 ∶ 𝐵 → 𝐵 = 𝐼𝐵
𝑓∶𝐴→𝐵

𝑓∶𝐴→𝐵
𝑓 −1 𝑜𝑓 ∶ 𝐴 → 𝐴 = 𝐼𝐴
𝑓 −1 ∶ 𝐵 → 𝐴

© 2021, BYJU'S. All rights reserved


Inverse of a function 𝑓 and 𝑓 −1 are mirror images of
each other w.r.t 𝑦 = 𝑥
𝑓(𝑥) = 𝑒 𝑥
𝑌
From figure, we can see that image of 𝑓(𝑥) = 𝑒 𝑥
𝑓 −1 (𝑥) = ln 𝑥
(𝛼, 𝛽) with respect to line 𝑦 = 𝑥 is 𝑓 −1 (𝑥) = ln 𝑥
(0,1) We can observe that image of any point (𝛼, 𝛽)
(𝛽, 𝛼)
𝑋 with respect to line 𝑦 = 𝑥 is (𝛽, 𝛼) so
(1,0)
𝛼 replace by 𝛽 and 𝛽 replace by 𝛼

𝑦=𝑥
In general, we can say that
𝑥 co-ordinate replace with 𝑦 co-ordinate (𝑥 → 𝑦)
Here 𝑓(𝑥) and 𝑓 −1 (𝑥) 𝑦 co-ordinate replace with 𝑥 co-ordinate (𝑦 → 𝑥)
do not Intersect
Graph of 𝑓(𝑥) and 𝑓 −1 (𝑥) may or may not
have point of intersection

© 2021, BYJU'S. All rights reserved


𝑓 and 𝑓 −1 are mirror images of
Inverse of a function each other in 𝑦 = 𝑥

𝑓(𝑥) = 𝑒 −𝑥 𝑦 = 𝑥2 + 1
𝑌
𝑌

𝑦 = 𝑥−1
(0,1)
(0,1)
(1,0) 𝑋
𝑋
(1,0)

𝑦=𝑥
𝑓 −1 (𝑥) = − ln 𝑥 𝑦=𝑥

𝑓(𝑥) and 𝑓 −1 (𝑥) have 𝑓(𝑥) and 𝑓 −1 (𝑥) have


One Intersection point no Intersection point

© 2021, BYJU'S. All rights reserved


𝑌
𝑓 𝑥 =1−𝑥

𝑦=𝑥

𝑓 −1 (𝑥) = 1 − 𝑥

𝑓 𝑥 =1−𝑥
⇒ 𝑓 −1 (𝑥) = 1 − 𝑥
Function and its Inverse function have same graph

Hence,𝑓(𝑥) and 𝑓 −1 (𝑥) have Infinite points of intersection


© 2021, BYJU'S. All rights reserved
Procedure to find Inverse of a Function:
If the given function is bijective, then these steps are followed
➢ Step 1 : Find 𝑥 = 𝑓(𝑦) if possible ⇒ 𝑥 in terms of 𝑦 if possible
−1
➢ Step 2 : replace 𝑥 with 𝑓 (𝑥) and replace 𝑦 with 𝑥 𝑥 → 𝑓 −1 (𝑥) , 𝑦→𝑥

Illustration :
𝑎 𝑓(𝑥) = 𝑒 −𝑥 , then find 𝑓 −1
Solution:
𝑦 = 𝑒 −𝑥

ln 𝑦 = ln 𝑒 −𝑥

ln 𝑦 = −𝑥
⇒ 𝑥 = − ln 𝑦 ∴ 𝑓 −1 𝑥 = − ln 𝑥
© 2021, BYJU'S. All rights reserved
Illustration :
𝑏 𝑓(𝑥) = 𝑥 2 + 1, then find 𝑓 −1 (𝑥) for 𝑥 ∈ [0, ∞)
Solution:
𝑓(𝑥) = 𝑥 2 + 1

𝑓(𝑥) = 𝑥 2 + 1
𝑥 =± 𝑦−1
𝑌 Now, Image of function is in
1st quadrant so we consider
𝑓 −1 (𝑥) = 𝑥 − 1 𝑥 = + 𝑦 − 1 (∵ 𝑥 ≥ 0)

𝑋 𝑥 → 𝑓 −1 (𝑥) 𝑦→𝑥

⇒ 𝑓 −1 (𝑥) = 𝑥 − 1
© 2021, BYJU'S. All rights reserved
Illustration : Find the inverse of the following functions
𝐴. ) 𝑓: ℝ → ℝ, 𝑓 𝑥 = 3𝑥 − 5
𝑦+5 −1
𝑥+5
Solution: Let 𝑦 = 3𝑥 − 5 ⇒ 𝑥 = ⇒𝑓 𝑥 =
3 3
𝑥
𝐵. )𝑓: 4,6 → 6,8 , 𝑓 𝑥 = 𝑥 + , where [.] denotes G.I.F
2
Solution:
𝑥 𝑥
Let 𝑦 = 𝑥 + 2 ⇒ 𝑥 = 𝑦 − 2 ⇒ 𝑓 −1 𝑥 = 𝑥 − 2 ∵ 𝑥 ∈ 4,6 ⇒ ∈ 2,3 ⇒ =2
2 2
1
5 7
𝐶. ) 𝑓 𝑥 = 1 − 𝑥 − 4
1
Solution: Let 𝑦 = 1 − 𝑥 − 4 5 7 ⇒ 𝑦7 = 1 − 𝑥 − 4 5

1 1
5 𝑦7
⇒ 𝑥−4 =1− ⇒𝑥 =4+ 1− 7
𝑦 5 ⇒ 𝑓 −1 (𝑥) =4+ 1− 7
𝑥 5
© 2021, BYJU'S. All rights reserved
10𝑥 +10−𝑥
𝐷. )𝑓 𝑥 = log 𝑒 (𝑥 + 𝑥 2 + 1) 𝐸. ) 𝑓 𝑥 =
10𝑥 −10−𝑥
+2
Solution: Solution:
Let 𝑦 = log 𝑒 (𝑥 + 𝑥 2 + 1) 102𝑥 +1
Let 𝑦 = +2
102𝑥 −1
⇒ 𝑒𝑦 = 𝑥 + 𝑥2 + 1 ⇒
102𝑥 +1
𝑦 − 2 = 2𝑥
10 −1
⇒ 𝑒𝑦 − 𝑥 = 𝑥2 + 1 ⇒ 102𝑥 𝑦 − 102𝑥 2 − 𝑦 + 2 = 102𝑥 + 1
⇒ 𝑒 2𝑦 − 2𝑒 𝑦 𝑥 + 𝑥 2 = 𝑥 2 + 1 ⇒ 102𝑥 𝑦 − 3 = 𝑦 − 1
⇒ 𝑒 2𝑦 − 1 = 2𝑒 𝑦 𝑥 ⇒ 102𝑥 =
𝑦 −1
𝑦 −3
𝑒 2𝑦 − 1 𝑒 𝑦 − 𝑒 −𝑦 𝑦 −1
⇒𝑥= ⇒𝑥= ⇒ 2𝑥 = log10
2𝑒 𝑦 2 𝑦 −3
𝑒 𝑥 − 𝑒 −𝑥 1 𝑥 −1
−1
∴ 𝑓 (𝑥) = ∴ 𝑓 −1 (𝑥) = log10
2 2 𝑥 −3
© 2021, BYJU'S. All rights reserved
Remark

⇒ If 𝑔 be the inverse of 𝑓 then we can say that 𝑓 be the inverse of 𝑔

𝑓 𝑔 𝑥 =𝑥

⇒ 𝑔 𝑥 = 𝑓 −1 𝑥

Also, 𝑓 𝑥 = 𝑔−1 𝑥

𝑔 𝑓 𝑥 =𝑥

© 2021, BYJU'S. All rights reserved


𝑒 𝑥 −𝑒 −𝑥 𝑒 1002 −1
Illustration : If 𝑓 𝑥 = &𝑓 𝑔 𝑥 = 𝑥, then find 𝑔
2 2𝑒 501

𝑒 1002 − 1 𝑒 501 − 𝑒 −501


Solution: 𝑔 =𝑔 = 𝑔 𝑓 501
2𝑒 501 2

𝑓 𝑔 𝑥 =𝑥

⇒ 𝑔 𝑥 = 𝑓 −1 𝑥

⇒ 𝑓 𝑥 = 𝑔−1 𝑥

⇒𝑔 𝑓 𝑥 =𝑥

⇒ 𝑔 𝑓 501 = 501
© 2021, BYJU'S. All rights reserved
Illustration : 𝑓: −∞, 1 → −∞, 1 & 𝑓 𝑥 = 2𝑥 − 𝑥 2 , then find 𝑓 −1 𝑥

Solution: 𝑦 = 2𝑥 − 𝑥 2

⇒ 𝑥 2 − 2𝑥 + 𝑦 = 0

2 ± 4 − 4𝑦
⇒𝑥=
2
⇒𝑥 =1± 1−𝑦 ≤1

⇒𝑥 =1− 1−𝑦 (∴ 𝑥 𝜖 −∞, 1 or 𝑥 ≤ 1)

⇒ 𝑓 −1 𝑥 = 1 − 1 − 𝑥

© 2021, BYJU'S. All rights reserved


1
Illustration : 𝑓: 1, ∞ → 2, ∞ & 𝑓 𝑥 = 𝑥 + , then find 𝑓 −1 𝑥
𝑥

1
Solution: 𝑦 = 𝑥 +
𝑥
⇒ 𝑥 2 − 𝑥𝑦 + 1 = 0

𝑦 ± 𝑦2 − 4
⇒𝑥=
2
𝑥 ∈ 1, ∞ , 𝑦 ∈ 2, ∞
So, only + sign is accepted

𝑥 + 𝑥2 − 4
∴ 𝑓 −1 𝑥 =
2

© 2021, BYJU'S. All rights reserved


Illustration : 𝑓 𝑥 = 2𝑥 + 1 , 𝑔 𝑥 = 𝑥 3 then find 𝑔𝑜𝑓 −1 (64) =?

Solution: Let 𝑡 = 𝑔𝑜𝑓 −1 Alternate solution


(64)
−1
𝑔𝑜𝑓 𝑥 = 𝑓 −1 𝑜𝑔−1 𝑥
𝑔𝑜𝑓 𝑡 = 64 1
𝑔−1 𝑥 = 𝑥3
𝑓 𝑡 = 2𝑡 + 1 is given, 𝑥−1
𝑓 −1 𝑥 =
𝑔 2𝑡 + 1 = 64 2
3
2𝑡 + 1 = 3 43
⇒𝑡= 𝑓 −1 𝑜𝑔−1 64
2 1
3 = 𝑓 −1 643 = 𝑓 −1 4
−1
⇒ 𝑔𝑜𝑓 (64) =
2 4−1 3
= =
2 2

© 2021, BYJU'S. All rights reserved


Illustration : Let 𝑓, 𝑔: ℝ → ℝ defined by 𝑓 𝑥 = 3𝑥 − 1 + 2𝑥 + 1 and
1
𝑔 𝑥 = 3𝑥 + 5 − 2𝑥 + 5 , then
5
𝑓𝑜𝑔 = 𝑔𝑜𝑓 𝑓𝑜𝑓 −1 = 𝑔𝑜𝑔

𝑓𝑜𝑔𝑜𝑓𝑜𝑔 … 𝑓𝑜𝑔 (5) = 5 None

Solution: 1 1
For 𝑥 ≤ − : For 𝑥 > − :
2 2

2𝑥 + 1 < 0 2𝑥 + 1 > 0
2𝑥 + 1 = −(2𝑥 + 1) 2𝑥 + 1 = (2𝑥 + 1)

𝑓 𝑥 = 3𝑥 − 1 − (2𝑥 + 1) 𝑓 𝑥 = 3𝑥 − 1 + (2𝑥 + 1)
𝑓 𝑥 =𝑥−2 𝑓 𝑥 = 5𝑥
© 2021, BYJU'S. All rights reserved
5 5
For 𝑥 ≤ − : For 𝑥 > − :
2 2

2𝑥 + 5 < 0 2𝑥 + 5 > 0

2𝑥 + 5 = −(2𝑥 + 5) 2𝑥 + 5 = (2𝑥 + 5)
1 1
𝑔 𝑥 = 3𝑥 + 5 + (2𝑥 + 5) 𝑔 𝑥 = 3𝑥 + 5 − (2𝑥 + 5)
5 5
𝑥
𝑔 𝑥 =𝑥+2 𝑔 𝑥 =
5

1 1
For 𝑥 ≤ − : For 𝑥 > − : :
2 2
𝑦
𝑦 =𝑥−2⟹𝑥 =𝑦+2 𝑦 = 5𝑥 ⟹ 𝑥 =
5
𝑥
⇒ 𝑓 −1 𝑥 = 𝑥 + 2 ⇒ 𝑓 −1 𝑥 =
5
© 2021, BYJU'S. All rights reserved
1 Now let’s check options
𝑥−2, 𝑥 ≤−
2
𝑓 𝑥 = 1
5𝑥, 𝑥> −
2 𝐴. ) 𝑓 = 𝑔−1 ⇒ 𝑔 𝑓 𝑥 =𝑥
𝑓𝑜𝑔 = 𝑔𝑜𝑓
5 𝑔= 𝑓 −1 ⇒𝑓 𝑔 𝑥 =𝑥
𝑥+2, 𝑥 ≤−
2
𝑔 𝑥 = 𝑥 5 −1
5
, 𝑥> −
2
𝐵. ) 𝑓𝑜𝑓 = 𝑓 −1 𝑜𝑓 −1 = 𝑔𝑜𝑔
(∴ 𝑔 = 𝑓 −1 )
𝐶. ) 𝑓𝑜𝑔 𝑥 = 𝑥
We can observe that 𝑓 −1 𝑥 = 𝑔(𝑥)
𝑓𝑜𝑔 5 = 5

So, 𝑓 and 𝑔 are inverse of each other. 𝑓𝑜𝑔𝑜𝑓𝑜𝑔 … 𝑓𝑜𝑔 5 = 5


Hence (𝑎, 𝑏, 𝑐)are the correct options
© 2021, BYJU'S. All rights reserved
Summary Sheet

𝑓 𝑔 𝑔 𝑓 𝑥 𝑔 𝑓 𝑓 𝑔 𝑥
𝑥 𝑓(𝑥) 𝑥 𝑔(𝑥)

𝑔𝑜𝑓 𝑥 = 𝑔 𝑓 𝑥 𝑓𝑜𝑔(𝑥) = 𝑓 𝑔 𝑥

Condition for existence of 𝑔𝑜𝑓 𝑥 :


𝑓 𝑥 must be subset of domain of 𝑔.
Here, we give 𝑥 as input to function 𝑓 then output is 𝑓(𝑥)

Now, 𝑓(𝑥) is input for function 𝑔 and then its output is 𝑔 𝑓 𝑥


© 2021, BYJU'S. All rights reserved
Summary Sheet

• To Find inverse function find 𝑥 = 𝑓(𝑦) if possible ⇒ 𝑥 in terms of 𝑦 if possible


then replace 𝑥 with 𝑓 −1 𝑥 and 𝑦 with 𝑥 (𝑥 → 𝑓 −1 𝑥 , 𝑦 → 𝑥)

• Inverse of a bijective function is also a bijective function and Inverse of a


Bijective function is unique.

• If 𝑓 & 𝑔 are two Bijective function and 𝑔𝑜𝑓 exists, then 𝑔𝑜𝑓 will be Bijective
function and 𝑔𝑜𝑓 −1 = 𝑓 −1 𝑜𝑔−1

© 2021, BYJU'S. All rights reserved


Summary Sheet
• 𝑓 and 𝑓 −1 are mirror images of each other w.r.t 𝑦 = 𝑥

• Graph of 𝑓 and 𝑓 −1 may or may not have point of intersection

• When 𝑔 be the inverse function of 𝑓 means 𝑔 𝑥 = 𝑓 −1 𝑥 then


following results are true
𝑓 𝑔 𝑥 =𝑥
⇒ 𝑔 𝑥 = 𝑓 −1 𝑥
Also, 𝑓 𝑥 = 𝑔−1 𝑥

∴𝑔 𝑓 𝑥 =𝑥

© 2021, BYJU'S. All rights reserved

You might also like